You are on page 1of 146
ACE Engineering Academy SINCE 1995 SINCE 1995 Hyderabad | New Delhi} Bengalueu | Bhubaneswar { Vijayawada | Visakhapatnam | Tirupati | Pune | Chennai Power Systems (Theory with worked out Examples) PSC | PSUs - Electrical Engineering ACE is the leading institute for coaching in IES, GATE & PSU’s HO: 204, II Floor, Rahman Plaza, Opp. Methodist School, Abids, Hyderabad-500001, Ph: 040-24750242, 24750263, 24750437, 24752469. CONSISTENTLY TOP RANKS IN IES Ail INDIA 157 RANK 26 TIMES IN GATE Copyright © ACE Engineering Academy 2014 All rights reserved. No part of this publication may be reproduced, stored in a retrieval system, or transmitted, in any form or by any means, electronic, mechanical, photocopying, digital, recording or otherwise, without the prior permission of the publishers. Published at : ACE Engineering Academy 2nd Floor, Rahman Plaza, Opp. Methodist School, Near Taj Mahal Hotel, Abids, Hyderabad - 500 001 Phones : 040-24750242 / 24750263 www.aceenggacademy.com email : ace.gateguru@gmail.com Authors : Subject experts of ACE Engineering Academy, Hyderabad S.No, Name of the Chapter Page No. 01 | Generating Stations | 1-13 02 | Transmission & Distribution | 14-62 a Rer-Unit system, symmetrical Components | 64 _ 95 ] 04 | Power System Stability 83-98 | 05 | Load Flow ae | 99- 105 | 06 | Economics of Power So 106 - 113 07 | Circuit Breakers 114 -127 | 08 | Protective Relays : 128 - 142 | COACHING OFFERED ¢ GATE e IES e PSUs > Class Room Coaching > Postal Coaching > Online Test Series > Postal Test Series > Interview Guidance Generating Stations Power Generation +, Non Conventional, Conventional Energy Sources Energy Sources Non Conventional Energy Sources Conventional Energy Sources 1. Smail capacity power Generation over al 1. Bulk capacity power Generation over a shorter interval of time Eg:- Wind, Solar,|. longer period. Eg:- Steam (or) Thermal, | Tidal, Geothermal, Diesel, Biomass and| . Hydro, Nuclear and Gas. |__MED, She |2. Most of the plants are located néar to | 2. Most of the plants are located at remote consumer premises i.e, there is no constraint | places i.e., these plants are set-up based on in setting up of these plants __geographical constraint 3. The operation of these plants are based on 3. There are no weather constraints while day-to-day weather constraints operating these plants. 4. Most of these plants having less fixed cost | 4. Most of these plants.are having high fixed and high running cost. cost and less running cost. 5. Suitable to meet peak load demands except | 5. Suitable to meet base load demand except Geothermal plants. Gas plants 6. Asynchronous Generators are employed — 6. Synchronous Generations are used Ex:- DC Generator, AC Generator and Induction Generator i 7. Distribution network is sufficient 7. Both the transmission and distribution networks are employed. | 8. Generated voltage will be at 1 KV. 8. Generated ‘voltage’ will be 33, 66, 11, 13.2and 18.2 kV. ‘A generating station which converts heat energy of coal combustion into electrical energy is known asa steam power station. 1. Selection of site for steam power stations: [ESE | New Dat Bengt | Bhbncorar | Viavada | Vikapatam | Tropa (a) The site should be nearest to the coal mines so that Transport cost of fuel is minimum. (b) Large quantity of cooling (for the condenser) water should be available. (©) The land should not be rocky and marshy. (@) The chimney of the plant, do not obstruct the flying aero planes, i.e, the site should be away from the airport, (©) The site should not be surrounded by residential building to avoid nuisance of smoke, noise etc. (® If the station is located, near the load center, the distribution cost reduces. (g) Facility for the disposal of ash. 2, Schematic Arrangement of Steam Power Stations: Induced maater of t Ho air ‘@) T. Coal Cal = Handi Storage Plant | ee LH tinting storage] eat Foresd deat fon Bxcier Finure: Schematic diaaram of Steam Power stations 3. The main and auxiliary equipment in a thermal power plant are. (a) Coal and ash handling plant (b) Steam generator or Boiler (©) Super heater (d) Economizer (©) Airpreheater. (f) Steam Turbine (g) Condenser (h) Cooling towers () Feed water heater () A.C. Generator (or) Alternator (k Exciter () Electro static precipitator (a) Coal and Ash handling plant: ‘The coal is transported (o the power station by toad ‘or’ rail and is stored in the coal storage plant. “The coal is delivered to the coal handling plant. Where it is pulverized (ie., crushed into small pieces) in order to increase its surface exposure. Thus promoting rapid combustion without using large quantity of excess air. The pulverized coal is fed to the boiler by belt conveyors. Advantages of coal pulverization: 1. Gives more surface area for a given mass of coal. 2. Fast combustion as more area is exposed to heat and oxygen. Boiler can be started quickly. Fast response to load changes. ay (ES ct [mr Wo | Ventas [Tp Te |G ACE sf Engineering Academy The coal is burnt in the boiler and the ash produced after the complete combustion of coal is removed (0 the ash handling plant and the delivered to the ash storage plant for disposal Indian coal has ash content of 20 to 40%. In thermal power plant bituminous coal is used because, it has low ash content. Eg: Semi — bituminous coal is more preferable. (b) Steam Generator (or) Boile Boiler is a device. Where in water is converted into steam by utilizing the heat of combustion This is two types: (i) Water tube boiler (ii) Fire tube boiler () Water tube boiler; ‘The waicr flows throxgh the tubes and hot combustion gases flow over these tube, water tube boiler are used universally for such plant, Features of water tube boiler. i) Method of water circulation ii) Improved method of heating iii) Easy removal of scale from inside the tube. (ii) Fire Tube Boil ‘The hot combustion gases flow through the tube. Water is surrounded by the fire tube, (c) Super Heater: A super heater is a device which raises the temperature of the steam~much above the boiling point of water. Super heater heat is taken from flue gases. Normal radiant type of heat transfer method is used. It is placed between boiler and turbine. It is improved efficiency of plant. ‘The use of Super heater: (@) Reduce the steam consumption per given output. (b) Reduce the condensation loss. (©) Eliminates the erosion of steam turbine blades due to the absence of moisture, (@) Improves the overall efficiency. Generating Stations (4) Economizer: Economizer mainly consists of closely spaced parallel tubes through which feed water on its way to boiler flows and the fuel gases flows outsides the tubes, Some of the heat energy of the fuel gases is recovered by heating the feed water hear saturation but, not converted into steam this results saving in fuel (10- 25%) and raising boiler efficiency. (© Air preheater: ‘An Airpreheater increases the temperature of the air supplied for coal burning by deriving heat from fuel gases. Air is drawn from the atmosphere, By a forced draught fan and is passed through air preheated before supplying to the boiler furnace. Air pre heaters are placed in between economizer and chimney. The efficiency of boiler is increased. (f) Steam Turbine: A steam turbine may be defined as a rotating machine, which converts the energy contained in steam into mechanical energy ot rotary energy. Basically there are two types of Turbines i) Impulse Turbine ii) Reaction turbine iii) Impulse Reaction Turbine (this turbine is a combination of impulse and re: turbines) (i) Impulse Turbine: The steam expands completely in the stationary nozzles (or fixed blades), the pressure over the moving blades remaining constant, so the steam attains a high velocity and impinges against the moving blades this results in the impulsive force on the moving blades which sets the rotor rotating. tion Gi) Reaction turbine: The steam is partially expanded in the stationary nozzles, the remaining expansion takes place during its flow over the moving blades, and the result is that the momentum of the steam causes a reaction force on the moving blades which sets the rotor in motion. (Ea ea en | eT Vann | Vp Ta] | ACE ) ae Power Systems (g) Condenser: ‘A condenser is a device which condenses the steam at the exhaust of Turbine. It serves two important functions. (1) it ereates a very low pressure at the exhaust of Turbine, thus permitting expansion of the steam in the prime mover to a very low pressure. This helps in converting heat energy of steam ‘nto ‘Mechanical energy in the prime mover.(2) fhe condensed steam can be used as feed water to the boiler. It is improves power plant efficiency, (h) Cooling Towers: A condenser is a device in which exhaust steam from steam turbine is condensed and the heat energy given by the steam during condensation is taken up by the cooling tower. Different types of cooling Towers are: 1, Natural draught cooling towers 2, Forced draught cooling towers 3, Induced draught cooling towers. (i) Feed Water Heater: The steam coming out of the turbine is condensed and the condensate is fed back to the boiler as feed water. It is necessary to heat the feed water before feeding it to the boiler for the. following reasons: 1. Feed water heating'improves the overall efficiency of the plant. 2. The dissolved oxygen and carbon dioxide are removed in the feed water heater. 3. The quantity of steam produced by the boiler is increased. 4, Thermal stresses due to cola water entering the boiler are eliminated, @ Generator: © Each altemator is coupled to a steam turbine and converts mechanical energy of the turbine into electrical energy. © Alternator used in thermal power plants are of usually 2 — pole (or 4 — pole) running at 3000 rpm (or 1500 rpm) for 90 Hz operation. (k) Exciter: * Exciters are nothing but the D.C. Generators their main function is to supply dc power to the field system (rotor). ‘+ Exciters are mounted on the same shaft of the alternator. © The capacity of the exciter is about 0.5% to 3% of the main altemator capacity. * In some cases a pilot exciter may be used to excite the main exciter itself. () Electvo static precipitator: The use of electrostatic precipitator is to remove fine, dust particles from flue gas, It is connected to high D.C. voltage about 30 kV. It is placed between combustion chamber and chimney 4, Advantages of Steam Power Plants: (a) Power stations can be located near the load centers, (b) It requires less: space as compared to the hydro — electrie power station. (©) Requires less transmission distribution, (@ Long summer will not affect the power generation like hydel power. and 5, Disadvantages of Steam Power Plants: @) It pollutes the atmosphere due to the production of large amount of smoke and fumes (b) Its costlier in running cost as compared to hydro-electric plant. 6. Efficiency of Steam Power Station: The overall efficiency of a steam power is quite low (about 29%) due to mainly two reasons. (@) Huge amount of heat is lost in the condenser. (b) Heat losses occur at various stages of the plant, [GSE aetna | Now Dal | Benes | Bhubaneswar | Vanvorade | Woaapaam | Tia | Pune | Chena ACE Engineering Academy Thermal efficiency: The ratio of heat equivalent of mechanical energy transmitted to the turbine shaft to the heat of combustion of coal is known as thermal efficiency of steam power station. Thermal efficiency n thermal __ heat equivalentof mech energy transmitted oturbine shaft heat of coal combustion Overall efficiency: The ratio of heat equivalent of electrical output to the heat of combustion of coat is known as overall efficiency of steam power station, Overall efficiency __ heat equivalentof electrical output ~— heatof combustion of coal Overall efficiency efficiency thermal efficiency x electrical Generating Stations 1.2 Hydro Electric Power Station A generating station which utilizes the potential energy of water at a high level for the generation of electrical energy is known as a hydroelectric power station, 1, Selection of site for Hydro Electric Statio (a) Sufficient quantity of water at a reasonable head should be available. (b) Mote area is required for reservoir and dam, (©) It should allow strong foundation with low cost. (@) There should not be any leakage of water in future, (©) The selected site should have large catchments area, to maintain a certain water level. ( Distance of power station site from load centers must not be longer. This is to minimize to transmission cost (g) Construction materials shall be available very nearby. (i) Submergence area shall be minimum 2 Schematic Arrangement of Hydroelectric Power Station: Main elements in hydro plant 1) Hydraulic Structure 2) Water turbine 3) Altemator Generator Head Race Dam Trost] (EST saad | Ney Da | Benga | Biabancvar | Viera | Viskhapatars | Tipe | Pune | Obes Surge Tank Anchor block Tail Turbine Draft Tube ® vnc Sots 1. Hydraulic Structure: Hydraulic structures in a hydro electric power station include as. (a) Water Reservoir (b) Dam (© Trash Rack (W) Fore bay (©) Surge tank (f) Penstock (g) Spill way (a) Water Reservoir: * It is a basic requirement of a hydro - electric plant. Its purpose is to store water which may be utilized to run the prime mover to produce electrical power. A reservoir stores water during the rainy season and supplies the same during the dry season. {b) Dam: ‘The function of dam is to provide a head of ‘water to be utilized in the water turbine. ‘© Ithelps to increase the working head of the power plant. Intake Structure: It is an arrangement to supply water into a water conductor system, {&) Trash Rac * The purpose of providing a trash rack is t0 prevent entering of debris which might damage the wicket gates and turbines runners. «Trash rack is made of steel bars and is placed across the intake at the beginning of water conductor system (ie, at the beginning of penstock). (@ Fore Bay: © It is an enlarged body of water just upstream of the penstocks. © The fore bay serves as a regulatory reservoir storing water temporarily when oad on the plant is reduced and to supply extra water required when load increases suddenly, (g) Spill Way Power Systems © It distributes river water to various penstocks. It is also known as head pond. (e) Surge Tank: © Surge tanks are large cylindrical tanks provided near the downstream end of the Jong penstock. ‘© Usually provided in high head and medium head plants, between dam and power house. Function of surge tank: + To provide better regulation of flow of water when the load on the system fluctuates. i.e., stores water when the load decreases suddenly and supplies extra water to the penstock when load increases suddenly, © Surge tank controls the pressure variations resulting from rapid variations in water flow in penstock and thereby prevents water — hammer effects. (0) Penstock: The pipe between surge tank and turbine is called penstock. © supplies water to the turbine from surge tank or fore bay. ‘* Its made of steel or concrete and designed as a pressure flow pipe. © Itis supported on anchor blocks «This may be considered as a safety valve for a dam. + A spill way serves to discharge excess water from the reservoir. 2, Water Turbines: ‘Water turbines are used to convert the energy of falling water into mechanical energy. Classification based on direction of flow: 1. Axial flow 2. Tangential flow 3. Radial flow (not used) a ee ‘Bhubaneswar | Visjawada | Viakhapatam | Tiwpatl| Pune | Chennai . ACE. Generating Stations 4, Mixed Flow (or) radial in ward ow N,=(1 165) WP p.m, He! Axial flow turbine: In this turbine, flow is parallel | to the axis of rotation of the runner, Eg: Propeller and Kaplan turbines Tangential flow turbine: In this, flow is tangential to the path of rotation of the runner. Eg: Pelton turbine. Mixed Flow turbine: In this, water enters the runner at the outer periphery in the radial direction and leaves it at the centre in axial direction. Eg: Modern Francis turbine. Classification based on action of water: 1. Impulse turbine 2. Reaction turbine Impulse Turbine: In this turbine, all the available energy of water is converted into kinetic energy or velocity head by passing through a nozzle provided at the end of penstock. Eg: Pelton wheel turbine. Reaction turbine: In this turbine, available energy of water remains mostly as pressure energy. As the water flows through the runner, the pressure energy gradually decreases. Therefore, the runner must be enclosed in an air tight casing. Eg; Francis, Propeller, kaplan turbines. Specific speed of turbines: It is defined as hypothetical speed of the turbine at which it produces 1 H.P of power under head of 1 meter. It is an important parameter in the design of turbines. Specific speed, NYP. Ht P. Where, N = Actual rotation speed in rpm, H=head in meters, P= power in HP(= 735.5 watts) Where P = power expressed in kW Selection of Turbines: It depends on head, specific speed, efficiency, part oad operation ‘Pelton | Francis | Propeller Wheel | and { _ Kaplan Slow speed | Medium | High speed (12-70) | speed (80-_| (310-1000) | 420) | Lowhead Less than | Propeller - im or Kaplan | Medium [15-10m | Kaplan or | ancis |Highhead [71-250m | Francis or pelton | | Very high | 250 mand_ | Pelton | | head above wheel Parts of a Francis Turbine: Spiral casing: The purpose of the casing is to provide an even distribution of water around the circumference of the turbine runner. + The cross-sectional area of the casing is gradually decreased to keep the velocity of water constant throughout its path. [GREET dated | New Det Benesura | Bhobaneswar | Visyaweda | Viakhapainam | Thupal | Pe | Chennai ACE, @ eta headeny 18: Power Systems Speed ring or Stay ring: The purpose of ring is to direct the water from the spiral casing to the guide vanes or wicket gates, Guide Vanes or Wieket Gates: The function isto | regulate the quantity of the water supplied to the | runner and to direct water on to the runner at an angle appropriate to the design. The Guide Vanes are operated by a Governor. Rumner: it consists of a series of curved vanes around the circumference to change he direction of flow of water when it passes through the runner. The runner is keyed to a shaft. Draft Tube: It is a pipe or passage of gradually increasing cross-sectional area which connects the runner exit to the tail race. The tower end must be submerged below the level of water in the tail race. Function of Draft Tube: (It permits a negative ex suction head to be established at the runner exit, thus making it possible to install the turbine above the tail race level without loss of head. (i) It converts @ large proportion of velocity energy rejected from the runner into useful pressure energy i.e, it acts as a recuperation of pressure energy. GOVERNING OF TURBINES: The operation of regulation of speed of turbine runner is known as Governing of Turbines. It is done automatically by means of Governor. Runaway Speed : This is the maximum speed at which « turbine wheel would run under the worst conditions of operation with all gates open so as to allow all possible water inflow under maximum head 3. Alternator: Hydroelectric generators are low speed ‘machines of the salient pole type, having a large number of poles, a large diameter and a short rotor (in length). The speed of a hydroelectric generator depends on the speed of the turbine driving it, which in tum depends on the specific speed of the particular type of turbine. Generally, Generation of voltages:3.3 KV, 5 KV and 11 KV at 50 Hz 4. Classification of Hydro Electric Power Stations: . Classification based on load characteristics: 1. Base-load plants 2. Peak-load plants. B. Classific characteristics: 1. Run-off-tiver plants 2. Run-off river plants with pondage. 3. Storage plants 4, Pumped-storage plants Based on storage Classification based on head 1. Low head plants 2. Medium head plants 3. Hiigh head plants 4.Nery High Head Plants Classification based on plant capacity: 3. Small-Hiydel plants 4, Medium Capacity plants 5. High Capacity plants 6. Super Hydro plants . Classification based on load characteristics: 1. Base Load Plants: * {tis plant designed to take care of based foad of the grid + The plant operated on the large portion of the load curve, ‘It supplies continuous power to the grid through out the year. © Generally itis of large capacity, # Its load factor is high, EGRET ain [New Daj ener | Biber | Vinraveda | VeaBopatam | Tira | Pane | Chea ACE 29: 2, Peak Load Plants: ‘© Itis a plant designed to take care of peak Joad of the grid system. ‘© It operates only during the period of peak load, Hence short period of total. operating time © Pumped storage plants are usually designed as peak load plants. B. On the basis of hydraulic or storage consideration: 1. Run - off the river plant: ‘+ The plant uses the water as it comes in the river. This type of power plants has no control over the river flow. It does. not store the water. ‘* During rainy season high water flow is available and if the power plant is not able to use this large ‘low of water some quantity of water is allowed to flow over dam spillways as waste. ‘© During dry season, the power produced by such plants will be low, due to low flow rates. The utility of these plants is very less compared to other plants due to uncertainty of power supply. 2. Run - off river plants with pondage: ‘The usefulness of the run — off river plant is increased by providing a pond (small storage) at the plant. The pond permits to store water during off peak hours and suses. during peak hours of the same day. To create pondage a barrage or a small dam may be constructed across the river. 3. Storage Plants: * A dam is constructed across the river to create a large reservoir for storing water during rainy season and the water is released when ever required. © The advantages of this plant is that the power generated by the plant during dry season will not be affected, Ex: Nagarjunasagar Hydro electric plant, Srisailam Hydro electric plant, Sileru Hydro plant ete. Generating Stations 4, Pumped storage plant: Pumped storage plant in combination with hydroelectric power plant is used for supplying the sudden peak load of short duration. During the off -peak period, water is pumped back to the head water pool from the tail water pool. During the peak demand the water is drawn from the head water pool to generate power to meet the peak demand. Pumped storage plant uses turbine called “reversible turbine” Eg: Nagarjunasagar v Hydro electric plant and Stisailam left bank power project. C. Classification based on Head: 1, Low head plants: * When the operating head is less than 15 meter the plant is named as low head plant. Run-off river plants are usually low head plants. © This type of plant uses Kaplan Turbine. 2, Medium head plants: © When the operating head of water is from 15 to 70 meter © This type of plant uses Francis turbine 3. High head plants: ‘* When the head of water from 71 to 250 meter the plant is known as high head power plant, ‘© This type of plant uses Francis or Pelton turbines. 4. Very High Head plants: ‘© When head is more than 250 meter. ‘* This type of plant uses Pelton turbines. Another classification Low head < 30m Medium head 30 ~250 m High head > 250m [ERR Ne To Waker | Vien | Vea | Tp [Re [Cha ACE 1 @ruAG eae 210: Power Systems 5. Calculation of Hydro Electric Potential: Water head: The difference of water level is called the water head. Gross head: The total head difference between the water levels in head race (upstream side) and tailrace (downstream side) is called as gross head or total head. Net head or effective head: Gross head - Head loss in the Conveyor system from Head race to the entrance of turbine due to friction ‘* It is equal to the difference of total head at the point of entry and at the point of exit of the turbine. Rated head: Head utilized in doing work on the turbine is called the rated head. Rated head = Net head ~ Loss in guide passage and entrance of the turbine Water Power equation (ar) output equation: Output Power, P=w.Q. Hn kW where, w= specific weight of water 9.81 kim’, H.= net head of water in meter on the turbine Q= quantity of water in m’/see. ‘1 = over all efficiency of the system. Output Power, P= ain hp. where, w = specific weight of water 000 kg/m Hydrograph: It is a graph between discharge and time in chronological order. Mass curve: It is a plot of cumulative volume of ‘water that can be stored from stream flow versus time. Flow duration curve: it is a graph between the discharge and the percent of time the discharge equaled or exceeded. Power duration curve: The power ~ duration curve is similar to that of flow duration curve but it represents the power developed in KW or HP against the percentage of time. Advantages of Hydroelectric power stations: 1) No cost of fuel 2} Low maintenance cost 3) High plant efficiency 4) Plant is free from pollution 5) Used as multi-purpose projects (irrigation, flood control etc) 6) Cost per unit is less 7) Suitable for variable heads and to act as. peak load plant. Disadvantages of Hydroclectrie power station: 1. The area required is more 2. High initial cost. 3, Located in remote area and require more cost on Transmission lines, 13 Nuclear Power Plants NUCLEAR POWER PLANTS: A generating station in which nuclear energy is_—_converted into electrical energy is known as a nuclear power station. 1, Selection of site for nuclear power station: 1) Availability of Water: As sufficient water is required for cooling purposes. 2) Disposal of Waste: The waste produced by fission in a nuclear power station is generally radio active which must be disposed off properly 10 avoid health hazards. 3) Distance from populated areas: The site selected for a nuclear power station should be quite away from the populated areas as there is a danger of presence of radio activity in the atmosphere near the plant. (ERAN had Nev Dam | Ben | Bhar | Wierda | Visibpatom | Tropa] ane | Chea Engin ACE My ichematic arrangement of Nuclear Power Statio The Nuclear Power Station mainly consists of the following: 1. Nuclear Reactor 2. Heat Exchanger 3. Steam Turbine 4. Condenser 5. Alterator Generating Stations Cod metal NUCLEAR REACTOR: It is an apparatus in which nuclear fuel (U") is subjected to nuclear fission. It controls the chain reaction that starts once the fission is done. If the chain reaction is not controlled, the result will be an explosion due to the fast increase in the energy released. Classification of Reactors: Reactor of a nuclear power similar to the furnace of a steam power plant. The heat liberated in the reactor due to the nuclear fission of the fuel is taken up by the coolant circulating through the reactor core. The nuclear reactor mainly consists of Fuel, core, moderator shield, reflector, reactor vessel, control rods and coolant. @ Types of Fuel Used: (a) Natural Uranium: Natural uranium is used as fuel and heavy water or graphite is nsed as moderator. () Enriched Uranium: The uranium contains 5 to 10% of Uzss and ordinary water can be used as moderator. (i) Type of Moderator used : (a) Graphite reactors (b) Beryllium reactors (©) Water (ordinary or heavy water) reactors ii) Type of Coolant used : (a) Gas cooled reactors (Hydrogen, CO>, air) (b) Water (ordinary or heavy water) cooled reactors, (©) Liquid metal cooled reactors. (® Organic liquid cooled reactors. (iv) Types of Core: (a) Homogeneous: In this reactor fuel and moderator represent a uniform mixture such as an aqueous solution of uranium salt, (b) Heterogeneous Reactor: In such reactor fuel rods are inserted in moderator. The fuel elements are generally arranged in some regular order forming a lattice. ‘Moderator: Moderator slows down neutrons before they bombard the fuel rods. [ERR a 0c Now DAT He [Bhan | Vogal | Vialhapanan | Trap | Pune | Chena @ vet Ficsen 12: Power Systems Control rods: The control rods are of cadmium or | 2, HEAT EXCHANGER: The coolant gives up heat boron and are inserted into the reactor. Cadmium to the heat exchanger which is utilized in or Boron are strong neutron absorbers and thus raising the steam, After giving up heat, the regulates the supply of neutrons for fission. coolant is again fed to the reactor. Reflector: This completely surrounds the reactor core and helps to bounce escaping neutrons back | 3, sre,yj ryRpINE: The steam produced in the into the core. Reactor Core: This contains a number of fuel rods made of fissile material. ‘Types of Nuclear Reactors: _ Name of Cooient | [sn] ghameet, [ Moderator [ Soler | rust | pT) Advanced 1 | 82294 | Graphite | CO, | Uranium (AGR) — Pressurised | Fr 3 | water | Pressurised. | Presursed | [rncted reactor | water | water | Unio | ewe | Boing ~~ Ordinary | water | water” Ordinary 3 ‘Uranium reactor | water Bariched Oride | GWR) ast 4 | Breeder No. Liquid | Uns & | Reactor | modertor.| sodium | Ury | BR) | Fast Breeder Reactor: A fast breeder reactor is a small vessel in which the necessary quantity (corresponding to critical mass) of enriched uranium or plutonium is kept without moderator. The vessel is surrounded by a fairly thick blanket of depleted fertile uranium The fertile absorbs neutrons from the fissile material and gets converted into fissile material. .The reactor core is cooled by liquid metal. U2as can be converted to Punw, the latter can be used in other thermal or fast breeder reactor. Breeding ratio: Ratio of number of secondary fuel atoms formed to the number of primary fuel atoms consumed. Breeding ratio equal to or ‘greater than unity, heat exchanger is led to the steam turbine through a valve. After doing a useful work in the turbine, the steam is exhausted to condenser. 4, CONDENSER: The condenser condenses the steam which is fed to the heat exchanger through feed water pump. 5. ALTERNATOR: The steam turbine drives the alternator which converts mechanical energy into electrical energy the output from the alternator is delivered to the bus~ bars through transformer, circuit breakers and isolators. 3. Fuels used 01690 + 10°C Boiling point 3000°C Electrical resistivity 18 © - cm. Fission: The breaking up of nuclear of heavy atoms into two nearly equal parts with release of hhuge amount of energy is known as nuclear fission, The release of huge amount of energy during fission is due to mass defect i., the mass of the: final product comes out to be less than initial product. This may defect is converted into heat energy according to Einstein relation of E = me’, Where, E = energy; m = mass considered (say 1 kg) ; c= velocity of light = 3 x 10° m/sec Fusion: The process of combination of two light nuclei to from a relatively heavier nucleus is known as fusion. The common between the two nuclear processes is that they release very large amounts of energy. But, there are many differences in the mechanism of the two processes. GSR TIN tad | Now Das | Benga | Bhabancovar | Viavada | Vnihapainan | Tiapt| Pe | Cheam ACE sb Girt Generating Stations sion involves breaking up of a heavy nucleus into lighter nuclei. Fusion, on the other hand, involves combining of two lighter nuclei into one heavy nucleus. The links of the fission process are neutrons while the licks of a fusion process are protons. Fission precedes best with thermal neutrons where thermal means room temperature, Fusion precedes best with thermal particles where thermal means temperatures of millions of °K, Advantages of Nuclear Power Plants: 1 The amount of fuel required is quite small. Therefore, there is a considerable the cost of fuel transportation. A nuclear power plant requires less space as compared to any other type of the same size. Hyderabad | NewD 3. Ithas low running charges as a small amount of fuel is used for producing bulk electrical energy. 4, Running costs are less. 5. Reliable and economical for bulk generation. Disadvantages of Nuclear Power Plants: 1. Nuclear power plants are not suitable for varying loads, as reactors cannot be easily controlled. 2. Ibis difficult to make the casing of the reactor, such as high temperature, neutron vomibardment, saving in | 3. The disposal of the products which are radioactive is. a major_—_ problem. ‘Bengaluru | Bhubaneswar | Viiyamada | Veakhapamam | Tirupati | Pune | Chennai > Transmission & Distribution 2.1 Basic Concepts and Line Constants in Transmission Necessity of Transmission Lines:~ Bulk amount of powers Generation could able to produce on economical basis by employing synchronous Generators at remote locations. The Bulk amount of power from the remote Generating station could able to be carried out to the load center by using suitable network and the network is called transmission network (or) transmission lines. Transmission line: It is a carier between the remote Generating station to the load center and it is made by conducting material (e.g. Al, Cu). load “Transmission -2 po -O- eat load i pen Tens ogg Level of voltages; Low voltage: 220V (1-phase,2-wire), 415V G-phase,4-wire). High Voltage : LIKV, 33kV. Extra High voltage: 66kV, 132kV, 220kV. Modem Extra High Voltage : 400kV. Ultra High voltage: 765kV and above. Most of power generation in India at LIKV. Maximum AC transmission Voltage in India is T65KV. (e.g, Sipat- Seoni 765KV line). Maximum DC transmission Voltage in India is S00KV (e.g. Talcher- Kolar Bipolar HVDC link). ‘Transmission - | Necessity of Extra High Voltages for transmission system: 1. The size of the conductor is reduced so that the cost of the conductor is reduced. 2. The transmission line loss will be reduced, 3. The transmission efficiency will increase n=1-k/vcosp I Selection of conductor size for different Voltage levels: Feeder: it is a conductor having constant current Density. The size of feeder designed based on current carrying capacity, Example for feeder is transmission line on which no tapping are taken to supply any local load, For V < 220 KV, selection of conductor done based on current carrying capacity For V > 220 kY, selection of conductor is done based on concept of corona (or) electric field Intensity. Distributor: It is a conductor having variable ccurreat density. Size of distributor designed based on voltage drop. a $220kV-- 400kV and above. - Conductors Bundle conductors ‘Types of conductors: 1. Solid Conductor: S Solid (®) High Cost i) High Tensile Strength (ii)Diffieut to string the conductors. (iv)High skin effect while using on ac systems SSSR) cada | New Deh | engtuns | Bhubaneswar | Vngawade | Veakhapamam | Tirupati | Pune | Cheam _ ACE, Engine Academy 215: ‘Transmission & Distribution 2. Stranded Conductor; It consists of two or ACSR: smaller cross sectional strands (or) filaments 1. Steel reinforced Aluminum conductor which are twisted together to get the required 2. Aluminum conductor and steel reinforced strength and running in parallel to increase the a current carrying capacity for the given The oves all Diameter of ACSR conductor will operating voltage. be D=(Qn-l)d n= no of layers in the stranded conductor. = diameter of each strand in em. aa “The number of strands in each layer will be Sin 1#6412 +18 +24 +30 436 Homogenous Stranded Conductor:~ It is made up of all the strands by one material | Epi: An ACSR conductor consists of 30 only ie., Aluminum, For AAC (All Aluminum Conductors): (i) Reduced Tensile strength due to strands when compared to solid conductor. (ii) Easy to string the conductors (iii)Reduced skin effect when compared to solid. But the cost of the conductor is not reduced to greater extent even though the strands are employed. Due to low tensile strength for conductor, mechanical sag increases and ground clearance of conductor reduces, safety of the system reduces. In order to improve the tensile strength and reduce the cost of transmission system composite stranded conductors are employed (.¢) two (or) more conducting materials used to make stranded conductor. 3. Composite Stranded Conductor: AL Fe In the composite stranded conductor the outer strands are baving high conductivity and low tensile strength ie; Aluminium and central strands are having low conductivity and high tensile strength i.e, steel are used which is known as ACSR conductor. Aluminum strands and 7 still strands will be represented as 30/7. In general for a given ACSR conductor the number of Al strands are more in number when compared to steel strands, he ACSR conductor represented as 1/6 It of one steel and six Aluminum strands. Eg3: An ACSR conductor consists of 7 steel strands and 54 Aluminum strands will be represented as 7/54, Skin effect: The non uniform distribution of current through the given cross sectional area of the conductor when it is operated on alternating current system is called skin effect. The main reason for the skin effect is non-uniform distribution of flux linkages. The skin effect will result in increased effective resistance (Rac) Vue In case of de system, there is no rate of change of current. So the current is uniformly distributed throughout the cross sectional area of the conductor. So the entire area will be considered while calculating the de resistance offered. Rat a 2g idea | New Deli | Beagalra | Bi rnc | Viayawada | Visakhapatnam | Tirupal | Pave | Chemist ACE 16: Meeks se = Jn case of ac system, there is a rate of change of current. Whenever the switch is closed, the system. is under sub/transient behavior. So the flux produced in the outer strands will link with the inner strands due to low reluctance offered, where as the flux produced by inner strands could only link with those strands and not with the external strands. So the internal flux linkages increases which will increases the internal inductance. Inductance will oppose the flow of current, so that the most of the current will concentrated on outer strands and very less current will be allowed through the inner strands. While calculating the effective resistance, the area in which the’ current concentrated is more only considered than that of entire area of cross section of the conductor. So the ac (or) effective resistance Rg = 2! (al Rac [-. Rac = K Reci K= 1.6] In case of ACSR conductor, the current distribution is non-uniform and the materials are also non-uniform. So the entire area will be considered while calculating the effective resistance. a= Entire area of cross section (or) High current is concentrated on the surface. So the conductivity of aluminum is extracted effectively. Less current is concentrated at the inner strands. So the conductivity of steel will also be éffectively utilized. Hence the entire cross-sectional area will be considered, while calculating the ac resistance. + As the diameter (or) Area of cross section of the conductor increases the skin effect will increase i.e., Rye will increase. * As the permeability of the material will increase, the skin effect will increase i¢., Rec will increase. As the operating frequency will increases, the skin effect will increase ie, Ry will increase. But it is more effective in communication circuits than that of power circuits, because the operating frequency is only 50 Ha, Skin effect (Rex) 2 Apt Skin effect (Rye) o d? pf Bundle conductors: Whenever the operating voltages beyond 270ky, it is preferable to use more than one sub conductor / phase which is known as bundle conductor. Bundle conductor is one which consists of two (or) more sub conductor / phase which are running in parallel in which the spacing between sub conductors is very high when compared to radius of each sub conductor; where as the distance of separation between the two phases will be very high when compared to the spacing between sub conductors. The each sub conductor is again composite stranded conductor, R Lata em 7 ae rel.Sem G 8 sm"IP, y 206m hor 400 KV line: 7400 kV tine” Byndle conductor Single conductor ‘Due to sub conductors the mutual distance is same We) 8m, The self distance will increase. Advantages of Bundle Conductor system: G@ Voltage gradient (or) field intensity will v reduced g GMD GMR In| —— care GMD is same (mutual distance) GMR is increased (self distance) Hyderabad | ‘Benglurs | Bhubanesar | Viayawada | Viakapatam | Tipai | Pune | Chennai @ ACE aay 17 ‘Transmission & Distribution (i) Due to reduced field Intensity the critical disruptive voltage will increase, So the corona loss is reduced. The reduced corona loss will reduce the communication interference with the adjoining communication lines, (ii) (iv) Due to increased GMR, Inductance/phase will reduced and capacitance / phase will increase. (¥) Characteristic impedance will reduced. (vi) Characteristic impedance loading will increase. If the single conductor is converted in to number of sub conductors then the radius of each sub conductor will be calculated as follows. D=O0O-O y Bundle Conductor PEP te ate If all sub conductors are identical Pan Radius of each sub conductor, 7, =~). for 2—sub conductors, 7, | 5 for 3—sub conductors, 5, Selection of Insulation level: ‘* In modem EHV lines the level of Insulation is designed based on switched voltages. ‘© The intemal voltages (switched voltages) will bbe more severe to the Insulation. * The insulation will be designed for peak switched voltages. * Transmission line will be protected from lightning surges by placing a ground wire or shield wire or earth wire on top of the tower. * Power system network will be protected from switching surges with Surge diverter or Surge Arrester (Non- linear resistor). Geometric Mean Radius (GMR): The transmission line conductor having only one conductor per phase, the GMR is the distance between the center to the circumference i.e. it is equal to radius of the conductor. However due to intemal flux linkages it is equal to | @ Itis also known as self distance, GMR or self distance wsed to calculate L of transmission line Self GMD is same as GMR and it will be used as a replacement for GMR in multi conductor system, MR 7188 1 Geometric Mean Distance (GMD): If a forward conductor ‘p’ is surrounding to the ‘n’ number of return conductors then OO! aoa 2 P dp Forward : a: fad, nd, SOn GMD is used in Land C calculations. return Example 2.1: P ce Find GMD and GMR of the following systems. @ Soca RGAE A ert | New Des Bene | Bhubaneswar | Viwawada | ViaKiapatam | Tropa | Pane | Cheam Sol: (i) GMR =3/GMR, GMR, GMR, GMR, (All three conductors are identical) = = 0.7788 * 3 em = 2.34 cm GMD = yD,,D,.D, =V4x4x4=4m Gi) GMR = 1 = 0.7788 «3 GMD = YD,,.D,.D,, = V4x4x8=5.04 m Observation: GMD of unsymmetrical spacing is more compared to its equivalent symmetrical spacing, Example 2:2: Find GMD and Self GMD of the following Bundle conduetor system in which r; =2 em, $=20 em, all the three phases are identical to each other, b Sol: Self GMD = i{se//GMD, SelfGMD, SejGMD, = Self GMD, (Ce All three phases are identical) = rixs = 0.7788x2%20 = 5.58 cm GMD =4/6x6x12 =7.56 m (By neglecting conductors) spacing between sub Power Systems Example 2,3: Find Self. GMD of the following . stranded conductors @ i) : ste COO es AL: Each strand radius. is rem ® @00 Self GMD = al 2r ar) x (rx Br xr) x (Parr) = fe x(0.7788) xBxax8 (0.7788) x8x4x8 =175 rem ic) solo © 3 ‘The effect of steel strand will be neglected in self GMD calculation. Self GMD of all four strands is same, Self GMD, = 4f7'x 87 x4rxJ8r =r 0.778% V8 x4x VB = 0.7788 V8 x 4x8 =223 rem So, self GMD of conductor also. ‘Transposition of lines It is employed to eliminate the communication interference when transmission line is running close to the communication line. The communication interference is magnetic effect. Normally in transmission lines there are two conductor configurations, a ° b © ° ° (Symmetrical untransposed ) SRR a | New Dati Bengaars | Batanesrar | Visvavada | Veakapatnn [Tip | Pane | Cheol ACE i a b ¢ Oo 0 0 (Unsymmetrical untransposed) S| Line “J Inductan | Voltage | Interference | | No | Configu- | cephase | /phase | | ration_| __| Untransp [Same | Same | Does not exist | 1 | osed because 0. | symmetri \ cal fine | Untransp | Unequal | Unequa | Exists because 2 | osed 1 #0. symmetri cal line | ‘Transpos | Equal Equal | Eliminated 3 Je | [because @,=0. creme i | |__| trical tine [| ‘The concept of transposition of lines is considered if the load is balance. However it is @ old concept and not suitable for modem power system. The inductance/phase of untransposed unsymmetrical will be the average inductances of the three phases. Definition: It is defined as change in position of power conductor at regular intervals at equal distances throughout the line length, so that the original position of power conductor will be replaced by its successive power conductor. Untransposed unsymmetrical 3-6 transmission line R : R y YY Power network $5 communication network 4x20 + byZ120+ $m 224040 4, at circulating current = aR R = resistance of communication network ® Bagiazeag Academy 119: ‘Transmission & Distribution The circulating current will provide the noise in the communication network which is known as communication interference. w LB 1B po Ce y sa Xe inZi00 XC teZ120 p22 Ky 62240) 2240 r= Gy 20+ Gy 2120 + 2240+ by 20 + dy 2120 + by 2240+ by 20+ Gy 2120+ dx 2240 = 0. The transposition of lines means the shi position of power conductor by 120°, Example 2.1.42 Find GMD of each conductors and the system for the following conductors confignration without ‘Transposition and with Transposition. 8 b £ Os eO O. }-——6m = 6m——4 Sols (i) Without Transposition, GMD,= ¥6x12 =8.48 m GMD; = ¥6x6 =6m GMD, = J6x12 =8.48 m GMD,, = GMD, GMD, .GMD, =7.56m i) with transposition, GMD, = GMD, = GMD. = 4/6x6x12 GMD .,=7.56m Finderatad | New Deli | Beagalars | Bhubaneovar | Viayawada | Viakapatnam | Tirupati | Pune | Chennai 2) ACE 13 0: Power Systems ‘The Transmission line consists of a series combination of resistance, Inductance and a parallel combination of capacitance conductance. | abi 54 c= G In general the conductance of transmission line is assumed to be zero. The conductance is the one which will be offered for the flow of leakage current through the insulation to the ground. This is only possible at tower location. In between two towers there will not be any leakage. Resistance Calculation: Iti expressed as R=" ikem. a Tn case of transmission line, the distance is in kilometers, So the parameters will be calculated based on per km length, p= specific resistance (0. km) 1 = length (km) a=area of cross section (cm’); The resistance is independent on the distance between two wites and it depends on the size of the conductor. The resistance is also independent on symmetrical J unsymmetrical configuration, Inductance Calculation: Magnetic fluxlinkages Inductance: (Current through conductor Hkm It will produce magnetic field and the energy stored in the induetor Ey = pur. 2.2 Transmission Line Parameters Inductance of 1-9, 2- wire system: 1 ey The circuit cannot be closed unless there is second wire. The second wire is necessary to maintain potential difference and also closed path for current. System or loop or total or circuit Inductance Lota = La + Le Leotat = 214 (Both the conductors are similar in size) Inductance/conductor (Ls) = Lint + Lex d d= Distance of separation in m r= radius of conductor inom For external Inductance py = 1.0 and for internal Inductance p, > | except for Cu and Ai conductors ts Bal Sim 8x Qn \r L =0.05pr+0.2 wf ‘) mH/km In case of Cu or Al conductors p= 1.0 1,=02 (2) mE/km r!= imaginary (or) fictitious radius due to addition of internal linkages to external linkages r= 0.77881] If both conductors are identical then Leowt = 214 Lega = O.1pr+ 0.4 u(2) mH/km SERRE ec ctat Da Sn [Brabenr Viera | Vala | Tiss | Pe | hea ® Engineering: ACE tens Ql: ‘Transmission & Distribution d Leu = 04 of i ntti, Induetance of 1-4, 1-wire system: Some times earth will also be considered as return conductor instead of conductor return. But there are certain disadvantages when it is used an a.c system. ail h eo Earth Inductance of conductor-a i 14=02 (8) miVkm r) Ly=0 , as the radius of earth is very large So Liat = La + Ly = La. Inductance of 3-phase 3-wire Transmission line: For any 3-phase conductor configuration with transposition, Inductance of each conductor will be same and Liph is L of any conductor. Liph = 0.2 uf on) mH/km, GMR Induetive reactance per phase, X;./ph = 2nfL./ ph Reactive power absorbed by line Q./ph=Px, /ph Q, 3~ph)=31?x, /ph Example 2.2.1: ‘A yo ‘conductor 1 - @ line operate at S0Mz. "The diameter of each conductor is 20 mm and the spacing between conductors is 3m: inductive reactance of each conductor. “OO OC FI L, ~oan() mH/km r 2n{—_3__| 0.788x10x107 ) = 1.19 mH/km Inductive reactance X, 374 Ohm 2nfl, Capacitance Calculatiot Capacitance= Hlectri The importance of capacitor is to develop static field and also to store the energy Be =" cv Capacitance of 1-phase 2-wire system (Isolated earth plane): The two-wires will act as two plates with a potential difference V or. the applied voltage between the two conduetors is V. a a a r b r d canbe Bi 4) r = 1.0 for surrounding dielectric medium (air) Fim , I= VoCw Alkm Capacitance of each conductor to neutral: There is a zero potential between the two wires at mid point so that each power conductor will act individually and deliver 50% of the total power. ma YY 2 a | oa | J 4 2 im | GRR) ert [New Det | Benga | Bhubaneswar | Vigawada | Visakhapatnam | Tiropat | Pane | Grane ACE mn E c 122: Power Systems. Con = In case of the capacitance calculation, the actual radius of the conductor is considered, where as in case of inductance calculation imaginary radius is, used due to internal magnetic flux linkages. ire Transmission Line Capacitance of 1-6 2 with the effect of earth: Itcan be calculated by using Kelvin’s images law. b | ‘\ $ Beuintenta plan (=) LN istpeenaipie 1 va? +4n7 h a4 The electric flux lines which are terminated on infinite plane it will not induce any charge on the infinite plane. However if it is terminated on finite plane, it will induce a charge which is opposite to that of the original charge of the conductor. The no of electric flux lines which are eliminated from the surface of the conductor are equal to the amount of charge which is deposited. Each electric flux line will induce a partial amount of charge on the finite plane. So there is a uniform distribution of charges on the finite plane, It is very difficult to evaluate the capacitance due io uniform distribution of charges. The uniform distribution of charges are replaced by single equivalent charge. In order to make the earth as zero potential plane the equivalent charge is placed below the earth at a depth equal to the height of the conductor so that ce —— d ——. Sol: it will be a mirror image to the original power conductor. Qnty Cae ag =m : 2h In Example 2.2.2: What is the capacitance of neutraV/im of a1 = @ phase line composed of number 2 single strand conductor radius is 0.328 em, spaced 3 m apart and 7.5 m above the ground? Neglecting earth effect, (9)0.817 pF/km (b) 0.00817 wrk (©) 0.0212 wF/m (@) 0.516 wR/km With earth effect mere, Earth effect can be neglected, if d? <<(2h)’ ord << 2h +. Without earth effect 2mx8,854x10°? x1 3 In) 2 0.328x10 17 x 10°? Fim, = 0,00817 uF/km (EGS tN as [Het | tenn | Ward | Vipin |Top | Pe [Ge @ Bay ACE 298: ‘Transmission & Distribution Capacitance of 3-phase 3-wire Transmission | Sol: Inter line capacitances in 3 phase ling configuration connected in A to calculate For any conductor configuration if the conductors capacitance are each conductor to neutral it are transposed then capacitance of each conductor should be converted into “Y is same such that 2dr SoC =3. C/ph=—™0__ Fy m. aoe Cow in (2) In this problem Cy, = 0.004 ,F/km r ' S0, Cag = 0.012 pFrkm Capacitive reactance perphase 1 Example 2.2.4: Xe/ ph = Q-kn e e en Ciph ‘The inductive & capacitive reactance’s perphase of a 3.- 4, 100km length of line are 500 and Capacitive susceptance, B, / ph = @C/ ph mho- 10000 respectively. What will be the inductive : e and capacitive reactance of the same line with Charging current, 1, /ph = V,,@C/ph A/kin ee Reactive power delivered by transmission line Sol: Inductive reactance Xo ! Wo 1 Oh ph =(Vqs).9C/ ph Capacitive reactane Ke x Q.G- ph) = (Va, oC/ph 00km., X_ = 50Q, Xe =10002 1Okm, Xi, = 252, X 1000. Example 2.2.3: In. a. 3. phase * symmetrical _ configuration transmission line has inter line capacitance of 0.004,F/km, What is ‘the capacitance of each conductor to neutral S.No | Line Deseription | Rj L Xi ci Xe 1. | Length increases | Increases | Increases | Increases | Increases | Decreases [2 | Distance of No Increases | Increases | Decreases | Increases separation increases | change 3. _ | Radius of conductor | Decrease | Decreases | Decreases | nereases | Decreases increases s = _ 4, _ | Symmetrical Does not | Decreases | Decreases | Increases | Decreases spacing depend | 3. | Unsymmetrical | Does not | Increases | Increases | Decreases | Increases spacing __| depend _ 6. {Effect of earth is} No | No No change | Increases | Decreases __| taken into account | change | change Jo 7. [Height of the{ No [No No change | Decreases | Increases conductor increases | change _| change SRT) si | Nee Das | Dents | tubancavar | Vinawad | Viahapaam | Tirupad | Pe | Chemai ® ‘Engineering, ne 124: Power Systems 2.3 Steady State Analysis of Transmission Lines There are two performance indices for any power apparatus ~ Power Efficiency 1. Efficiency { Energy Efficiency Voltage Regulation 2. Repsain| Speed Regulation The transmission line is & static device. So the performance of the transmission line is analysized by considering the power efficiency and voltage regulation. ‘Transmission line Sending end Receiving end (Ps, Qs, Vs, Is) (Pi, Qs Ver T) Power Efficiency: Efficiency = eet x 100% ut put + losses —__ 100% P+3PR The losses in transmission line generally very less. So the efficiency will be very high i.e. (95-99%). So there is no much concern about the efficiency. Voltage Regulation: It is defined as the change in magnitude of receiving end voltage from no load to full load while keeping the sending and voltage and the frequency of supply remains same. 8=|Veol - [Ve [Yeo] =[¥e|>100 %e= vi Vro= no load receiving voltage V;. = full load receiving voltage Regulation means voltage drop. Percentage regulation is having good meaning when compared to regulation. Regulation depends o the quantity of load placed at receiving end and powr factor of the load. Representation of transmission lines: The transmission can be represented based on the | length in which the power is carried out. A) short transmission lines:- less than 80 km. B) medium transmission line: 80 km to 160 km. C) long transmission line: Above 160 km, Short transmission line: Generally charging current is very less in short transmission line so capacitance can be removed from the network. So it will be represented as RL Jumped parameter model. Medium Transmission lines; As the length is ‘medium, charging is appreciable so capacitance will be considered in the network. So it will be represented as RLC lumped model. Load end Capacitance model: The total shunt capacitance is placed at the receiving end only. (or) the total charging current will be allowed till the end of transmission line Sending and Capacitance model: The total shunt capacitance is at the sending end. There is no charging current in the transmission line. Nominal — T model: The shunt capacitance is placed at the middle (or) the total charging current will be allowed upto middle of the transmission line. Nominal — m model: The shunt capacitance is placed on both sides with equal magnitudes (or) the charging current of 50 percent magnitude is allowed tll the end of transmission line, [EERE STEIN porting [New Dati | Bengaara | Bhubaneswar | Vijpawada | Visakhapatnam | Tirupati | Pune | Obeanai _ ACE ® Engg heady 195: ‘Transmission & Distribution Long Transmission line: It will be represented in RLCG distributed parameter model. Uniform Distributed Parameters-These parameters are physical and electrically not separable. ‘Uniform distributed parameters are considered to evaluate the transient behavior of long transmission lines. ie. switch closed condition. Exact mathematical solution is considered to evaluate the sending end voltage and current. Lumped parameters are considered to evaluate the steady state behavior of long transmission. lines. ‘The two possible network configurations are, (i Equivalent -T Gi) Equivalent - x ‘The most and effective way of representing the transmission line is using. two port network configuration port means pair of terminals, ABCD parameters or Generalized circuit constants: Is I ree 8 t Vu Ve Oca : Vs, Is_ are dependent values Vr, I, are independent values The dependent values are expressed in terms of independent values. With certain parameters and those parameters are called Transmission parameters or ABCD parameters. V.= AV, + Bl, «... (1) 1, = CV; +Dk -...(2) V,) (A BYV, 1.) tc DJ, Equation (1) and (2) are standard transmission line equations, Parameters will be calculated by the following procedure. Cased: Open circuit at the receiving end (1,= 0) Vs=AVo, Is=CV 0. Reverse voltage gain under OC (unitless quantity) C==, transCet admittance under OC (mho) , OC input impedance (Zoc) Case 2: Short cireuit at the receiving end (V,=0) Vs= Bleoy “Ts=Didse , transfer impedance under SC (©) , Reverse current goin under SC(unitless quantity) 2 Fl D ZL: Seiaetimnedance Zc Characteristic impedance of transmission line & cD Fora symmetrical transmission line A= D For a reciprocal transmission line AD-BC = 1 Zeelea lay it % For symmetrical line Z, = £ (ER a [New Demi | Wecgatre | Bhabanevar | Viravac | Vana | Trap | Pane | Cheol Sol: The given equations can be expressed as Vv.) (A B *) 1) \c pj) v,\_(A BY'(V, iJ \c vp) Uy . D -BIY, AD-BC|-c A |, _[D -BIVs “[-c AU, The output impendence of transmission fine is v,__DV,-Bl, I, -CV,+Al, Transmission lines in series (or) Cascade (or) Tandem: 14 Bi pata v sa, F cD, vy Mo mn Ao Bo j Co Do VY [4 Bl (h (* Bo fe oe By] cy val |c, ville, D2) The resultant ABCD parameters of cascade combination of two networks is product of two ABCD vectors. a lose RTI eta | New Det | Bengal | Bhabancsvar | Viayawada | Viakhapatnam | Tirupati | Pu | Chea 6: Power Systems ‘Two Transmission lines in parallel:- a Ar Bi [ ettan to Az Bz C, Dr AB, + A,B, B+B, * (A-A)D; ~D,) =(6.26)+A-WO If two lines are identical transmission lines Ao=A, Bo=B/2, -Co=2C and Do=D Example 23.2: ‘Two identical lines having A = 093.2 8 ands Ae C= 10°29" and D = 093212 are connected in Parallel, The equivalent transmission Tine parameters @ ag CE ates ‘Transmission & Distribution Sol: The resultant combination A=D=093 21.2° B= 66272°ohm C= 2x10°290° mho ABCD of this parallel Short transmission line: Series combination of resistance and Inductance. x ke x] 10V, + Zk L=0.V, +101 AB) (IZ Ee Al + i) A=D, network is symmetrical AD—BC= I, network is reciprocal Regulation: Woul W009 al on no-load = sv Vs= AVey In case of short transmission line |A |= 1.0 [Vr0|=| Vs | IYAII¥L 199 IV, 1 The sending end voltage magnitude of short transmission line will be represented as [Wsl = [Vi] + Roos ty Xsing %e= PACE En “4° stands for lagging p.f ‘-* stands for leading p.f %e= %Reosd, + %Xsind, For lagging power regulation is positive For leading power factor the regulation will be either positive or negative or zero. ‘The regulation will be maximum (positive) at 4,-tan"! (4) R 4, = (impedance angle of transmission line) ‘© The regulation will be zero at leading power factor, factor always voltage Bs . ‘A distribution Sine having series impedance in per unit 0.08: + 40.05. What is the voltage regulation ‘witha’ load power factor of 0.8 Jagging at half pull load condition Sol: at FL, %Voltage regulation =%R cosh, + %Xsindy = (8x0.815%0,6)% =94% At Ys FL, %Voltage regulation = tx 9.4% a4T% Medium Transmission line Receiving end capacitor model: k Z i: T Vs ¥] Ve Pp Hirabed | New Delt | Bengdara | Bhubaneswar | Vinyowada | Veakhapatamn | Twpad | Pune | Chensal Oved Fiore 228: Power Systems Ve\_(1 21 0) (ve Nominal - = model: I) \o i,y/2 UY I I, - | | A=1+ZY,B=Z, C=Y and D=1.0 ra Ly A#D, network is unsymmetrical AD ~ BC = | network is reciprocal Sending end capacitor model i = CG 005 A= 10, B=Z,C=Y and D=1+ZY A#D, network is unsymmetrical AD - BC = | network is reciprocal Nominal — T Network: ZI Z/2 PT ‘ Vs Ve M4 {Tt (i) rue Ve) (1 Z/21 OV zi2Vv, i) lo tty alo ihe ZY A=D, network is symmetrical (1% (1-24) Le AD - BC = 1, network is reciprocal. (4 By 2 Is (ut) ts 2 A=D, network is symmetrical AD BC= 1, network is reciprocal For all the networks the preferable voltage WA regulation formula is %e VV, ‘The Nominal T Network is a symmetric and reciprocal network. Sending end capacitance method is a uncompensated method. Receiving end capacitor method is a over compensated method. So the medium transmission lines are represented either by nominal ~ r (or) nominal = T methods. However the nominal ~ 1 is a preferred on according to load conditions. ‘A medium line with parameters A, B, C, D. ‘extended t ‘by ‘connecting ; ‘a short tine of impedance Zin ‘series. The overall, ABCD. genre of the: PRET cect | New Dati | Beneaurw | Bhubaneswar | Vieyavada | Viakbapaiam | Trpat | Pune | Chenal ® si? EBay 199: ‘Transmission & Distribution Sol: Medium line and short line are cascaded IeX2 together the resultant ABCD parameters as le Vs, 4, By) [4 By. Zz te R/2 follows) 5 l=le plo 4 ood med Vio ‘A AZ+B Example 2.3.6: : acne ‘The ABCD constants of 9 3 phase transmission line are A = D = 0821’, B = 170280'Q, Example 23.5: ‘A 50Hz 3-phase transmission line of length 100 ‘km has a capacitance of 0.03/n pF per km. it is represented as a x model. The shunt admittance a each end of the transmission line will be (@) 150x106 “290° mho (b) 10010290" mho (©) 50x10°290° mho LOE oa @ a5 gon mho ‘otal shunt admittance, Y=joC/km. 1 =j2nx 50x22 <100 nS = 3002 90° ns In the nominal x model half of the admittance placed at sending end side and reaming half placed at receiving end side. The shunt admittance at each end of transmission line is z =150x10290°mho Ferranti Effect: When the transmission line operating at no-load (or) light load condition, the receiving end voltage is more than the sending voltage. This phenomena is called ferranti effect. Itis more severe in long transmission line. R22 vO) od r i R22 ROSES) ew Da Bese | Wt] Vinee | Vintage [Tit] ane | Coa C = 0.002290.4"mho.The sending end voltage is 400KV. ‘The receiving end voltage under no Toad condition is. {APGENCO) (a) 400 kV (b) 500 kV. ‘ (©)320kV. (@4ITKV Sol: No load receiving end voltage 400 08 = 500KV Long Transmission line or Distributed xl x0 Series impedance per km, 2=r+ joL Okm Shunt admittance per km, y= g + jooC mho/km ‘At any point x on the line voltage and currents will be given as V, =(coshyx)V, + (Z,sinkyx)1, sink |, + (coshy).L, % “y called as propagation constant, = [zy ‘Zc! called as characteristic impedance, Z, = E y Power Systems At the sending end side (x =), Ve=Vs and =I, So the sending end voltage and currents given as V, = (coshyl)¥, +(Z, sini), ) 1 inky |P, + (cosh). (Zen 7+ (cashryl).1, | Cosby Z,Sinhyl]-y Fl sitet Coshyl x ‘These are the exact ABCD parameters (or) ABCD parameters of distributed parameter model (or) (or) practical transmission line model. Here A = D = Distributed mode is symmetrical AD-BC = 1 = network is reciprocal Approximate ABCD parameters of long line: coshyt=t4 2" OO" 5 2 a! (0 sink yee cosh 21+ 0 2! yee 3 «wo? =(Jauy =2Y, (Z=21, ¥=yl) sinh A=D = cosh yé : oy B=Zcsinhy! 2 =Z, fs go" | 6 . Gee 7 a-7[1-2| 6 Conclusion on ABCD parameters: ABCD parameters will be represented as, A=|Al Zo, B= BIZ, cally, D=|DIZa Generally © |AJSL, very close to 1, a= 0 to 10° * Angle B is equivalent to impedance angle of line such that f = 60° to 90°. |B] value depends on length of the line * Angle y ~ 90°. [C| depends on length of the line © {D| <1, very close to 1, A=0 to 10° If length of transmission line increases then e IAlL,at © BIt, B almost constant © [C| 1, y almost constant DL,ad Static Power Transfer Equations in terms of ABCD parameters V, =|Vl48 v= fe beers] Te pl Vv, \Z0 ‘The angular difference between sending end and receiving end bus voltages is known as load angle of transmission line . The receiving end power transfer S= Vil" (Always complex power taken as VI* provided that inductive toad absorbs reactive power and capacitive load delivers reactive power) (ESAT) desta | New Dai | Bengal | Bhubaneswar | Viayavads | Viskbapamam | Tirupa | Pune | Chenal ‘Transmission & Distribution Lolo 8) aE cos(8) (\v28-lalZolv,.20)" NAMA IvP S,=|V,|20, BB zy Sne-8)- iz| —+-sin(0) IVIL, | 8) Aly 26a) Prax occurs for 8 = 0 Bi a LYAIIV IM cog(@) eee -()) mu 12Z| iz Reactive power during Prax, Ne lA fl 2 iBi IN sing 8). cial sin(p-a) ~( vio The loci of the above equations are circle. For | Approximate Power transfer equations fixed receiving end voltages they will be concentric circles, «For long transmission line R< . Voltage in the line V = See Lv —() ‘Delhi | Bengals | Bhubaneswar | Visyaweds | Viakapenam | Tirapal | Pune | Cheam Power Systems 1 ieee NMoboE,. * For over head line system, surroundings dielectric medium air ¢,= 1 for air Hobe v=3x10'm/s 3x10* km/s (for lossless OH line) © Forattenuated over head line, v=2.5x10%km/s to 2.8x10°km/s © For Underground cable [4x10 x8,854%10" Wheeee, &, >I for cable dielectric material (UG cable) > v(OH line) Example 2.41: ‘What is the velocity of wave. i propeaition i in a under ground cable having relative permittivity of its diglectric material 4. 3x10" ve Where €, =4. 50, v= 1.5 x 10° mis ‘Example 2.4.2: ‘The time taken for a surge to travel a 600. km long overhead transmission line is _ “TAS-05 (a)6s (bis: S (c) 0.02 s (a) 0.002 s Sol: The time taken by the wave to reach end of ength Velocity Sol: in UG cable v= the line is T = _ 600 3x10° = 0,002 s Propagation constant (7) : It represents changes happening in V,I waves during wave propagation such as physical displacement and decrement in magnitude of wave. yay z=r+ joL@/km y=2+joC U/km + Forattenuated Tr line = V+ joL)(E+ jo) y=a+ jp is attenuation constant, represents decrement is VI wave magnitude (nepers/ km) B is quadrature component, represents physical displacement of wave (radians/ km) For loss less line (r= 0, g= 0) y=joVLC @.=0, no attenuation B=oVLC rad/km B=1.047x10° rad/km © For loss line or B>1.047x107 rad/km attenuated line Wave length of line (2): Wave length of transmission line is distance to be traveled by the wave to get one wave repetition or 2 m radians or 360° With B rad/ km physical displacement (CGE) oetad Nev Dats [Berea | Babancoar | Vinyard | Veakapamam | Tirupad | Pune | Cheat Oued. ‘Transmission & Distribution Sol: If the propagation constant is a:* i then the wave length will be Surge impedance / characteristic impedance (Ze) Natural impedance : It is impedance offered by apparatus for surges or traveling waves or transients lz Vy © For attenuanated line (r+0, g*0) _—- _ [Priek Ver joc Z,+|Z,|Z-0 (capacitive) © For loss less line: (r=0, = 0) i io — (pure resistive in nature) ve Its pure resistance in nature but it will not create any losses in line. Surge impedance or different apparatus © For OH line Z, = 200 ~ 500.2 Typically for OH line Z.= 400 * For UG cable Z,= 4080 Typically for UG cable Z,~ 40 0 Example 2.44: | The Generalised cireut constants of 132, tine are A=D= 0.98203" “ B=82.5276'2 0005290" mho What is surge impedance of ine Surge Impedance Loading (SIL): It is also known as characteristic impedance loading or ideal loading or ideal power transfer capability . It is the loading on the line when ever the load impedance is exactly matching to surge impedance of transmission line . So at SIL condition Z1,= Zc Z, (cesistive) K V, & Pare resistive load @ « ttpeltev 2° 2 © Transmission line will act as neither sink nor source of reactive power (Q) © Load angle 6 = Be ‘© Both sending end and receiving end power factors are unity Loading > SIL + 2 i © Load angle 8>Bé (ERATED) oer | New Det | Bengore | Bhubaneswar | Vinyowada | Vikapatam | Tirupa | Pune | Cheam . ACE ® * Receiving end power factor is unity but sending end power factor is lagging Loading < SIL © Z>Z6¢ © tip ctev 2 * Transmission line acts as source of react ve power ° |V,|>[¥,| (errantieffect) ° iid SIL . Economical loading on turider ground cable is loading < SIL Whenever the traveling wave reaches (0a discontinuity or if the wave faces a change in medium in its propagation then there will be some reflections experienced by the wave such that the resultant wave at that point will be sum of incident and reflected waves. From basis wave equation , voltage at any point x, at any time tis given as Where fi is forward wave and f is backward wave. Refracted wave = Incident wave + reflected wave =2085.4MW. Example 2.4.6: ‘A.200km long lossless over head line operated at SIL condition: What will be the iad oe of ‘transmission fine 2 Sok: at SIL condition, toad angle 6=/ radians 8=1.047x10°x200 = 0.2094 radians Incident V, 1) 2 ceiving end Ss e Refracted (Wash) Reflection Wh) Surface of discontinuity If incident voltage is V then the incident current willbe 1= 2, Zo Reffacted quantities are , ESSER TIN) erat | New Deki [Beni] Bianca | Vinyovada | Visakhapatnam | Tirupati | Pane | Cheam ‘A step voltage of magnitude V initiated on transmission line t=O instant such that a current wave of I also propagated towards end of the line. In open circuit line always refracted current is zero at receiving end side where as voltage at sending end side is maintained as constant at V . Time taken by the wave for one propagation ‘Transmission & Distribution After four propagations the first propagation will be repeated. Short circuit line wave propagation ac A step voltage of magnitude V initiated on transmission line t=0 instant such that a current wave of I also propagated towards end of the line. In short circuit line always refracted voltage is zero at receiving end side where as voltage at sending end side is maintained as constant at V. Time taken by the wave for one propagation through the line is T= length / velocity. through the line is T = length’/ velocity Voltage Current Voltage Current wo VE ~ T ual t=T v I te v I Ti Tt fav ty Tr Vv T a or Veo. tor 2 1-0 5 a — y ore 31 teat Ty Os a it v 4 3 13T v 1 1337" ae an 137 v Ie a av 31 tar v=o [RATED ering | Nov Dati | Bengaluru | Bhubaneswar | Vieavada | Vinkhapatam | Tirupati] Pune | Chennai 4 pee) a Power Systems In each propagation the current in line will be increased by a value I such that in the fourth propagation the current in line becomes 4I Example 2.5.1: ‘An overhead line 600 km long and. having surge impedance at S00. ‘The. line short circuited at the receiving end and a voltage of ‘6000 v is applied to the other end. Calculate the sending end voltage and curcent after 0.008 Sol: Incident voltage, V = 6000 V V _ 9000 pa Incident current, 1 = 500 Z, 1 6oo0v sc 1+ 600km Time taken to reach 600 km _ 600 © 3x10° After 4T are 0.008s voltage and currents at sending end side are V. «000V I= 51= 60A = 2 msec Electrical equivalent circuit for the calcuiation of refracted quantities at discontinuity Incident waves are V, I Reflected waves are Vi, 11 Refracted waves are V2, Ir The lumped electrical equivalent circuit to find reffacted quantities at T” is as follows Equivalent circuit Reflected (Visi) refracted (V2,l)~ incident wD Zz, © Refracted voltage V, =2V—“" Z+Z, Coefficent of voltage refraction Vz 2% Veefaction = Ve =F Vo Z,4Ze «Reflected voltage V, cd Coetlicient of voltage reflection ee aM | 20-2. stain P| 7D a Z,+Z, Z. a Z+Ze Co-efficient of current refraction i ok 2e tn TD Reflected current 1, =I, -I 122-1 tZe # Refracted current 1, = ppEiderted | Now Delhi] Benga | Bhubaneswar | Vinyavada | Viakhepatnam | Tirupati | Pane | Ch i 289: ‘Transmission & Distribution { i E a Al ye 2enZe 2,42, | Co-efficient of current reflection =| 2-2 ee ze For open circuit line (Z,=0): Venton = 2 Vreflection = 1 ireraction = 0 =-1 iretestion For short circuit line (Zz = 0) Veeacton = 0 Vreteto jreiacion = 2 ietection = 1 Example 2.5.2: Actrayeling wave due to lightning with an incident voltage V travels through the overhead line of sure impedance of 400 Q and enters a cable of surge. impedance of 400. What is the voltage centering the cable at the junction? ES-06) @Va ay /i1 (2/11 @yv Sok: Reffacted voltage into the cable is, z V, =2V ine + Z csbte ,=2v 0 400+40 2 u Hinderabad | New Delhi | Bengalra | Bhubanesvar | Example 2.5.3: ‘Match List I (oad) with List I (relationship) iw respect of forward. current i, and the reflected current j, at the receiving end of the transmission line. . Select the correct answer using the codes given below the Lists: (IAS-07) List ‘List 1 A. Open-circuit Lie 0 B. Short- 2. = Ar GR, 3.Partial reflection D.R, —_| kv ‘ 22,4 ZZ, 42,2, 400%40 (00% 400) + (400% 40)+ 40400) = aun =3.333 KV V, _ 3.331000 400 3.333x1000 334, = 83,33A, What is he fest impressed surge voltage and currents into line-3 from following network, Z,= 3002 Z, @ KY, 155.8 kA. (0) 15584 KV, 623. (©) T792KV,31ATA. (d). 311.68 kV,1246.8A eo 1 (ELE YN ete nc | Vigo | Voom [Trp | Choa ACE @» E cadey sl: “Transmission & Distribution V=200kV 200KV Line-l Line-2__Line-3 qe 2500 Impressed surge voltage on L-2 300 700. Impressed surge voltage on L-3 ATL sany| 22] 550 a 22004 |-vrsaey = 155.83 KV Current in L-3 83KV SON $623.3 250 2.6 Voltage Contro} axv | 7,42, | The performance of load equipment is satisfactory, if it is supplied with quality of electricity. It means that the required real power is, delivered with the terminal voltage is within the regulation levels. The system should not experience either over voltage or under voltage. Due to over voltages insulation failure will occur where as due to under voltages losses in the system is more and power factor will be very less . There are two methods of voltage control (1)Internal method (or) Excitation Control:- Controlling the voltage of any transmission line by controlling the excitation of alternator will also changes the voltages of other lines also. So the voltage control by controlling the excitation of alternator is not acceptable. (2) External Method In this method the voltages at the required buses will be controlled by placing separate compensating devices in the system such that the other bus voltages will not be affected. eae! Generally the magnitude of voltage will be controlled by doing reactive power compensation in power system network: The different compensating equipment are, (i Shunt Capacitor i) Stunt Inductor (ii) Series Capacitor (iv) Synchronous Capacitor (¥) Synchronous Inductor (vi) Synchronous Phase Modifier or Phase advancer. In the above methods, the voltage will be controlled by controlling the net reactive power of the system. Sign Convention for reactive power: hi I Case (1): Synchronous Generator (machine) connected to Inductive load. ‘The synchronous machine will deliver the required reactive power to the load in a clockwise direction of current. . ; pirat | New Debi | Bengaluru | Blancoar | Vinyard | Visihapainan | Tropa] Pune [Chena] @ edits Power Systems ‘The reactive power absorbs by the Inductor is positive. The reactive power of synchronous machine will also positive and it will act as the synchronous capacitor. Case (2): The Synchronous Machine is connected in parallel with shunt capacitor. ‘The shunt capacitor will deliver the required reactive power to the synchronous machine, The reactive power absorbed by shunt capacitor is negative. That is capacitor delivers reactive power and it will be absorbed by synchronous machine so it act as synchronous Inductor. Electrical equipments - Reactive power nature: Induction motor — absorption (sink) Transformer — absorption (sink) Transtnission line ~ Neither sink nor source. (loading = SIL) Sink (loading > SIL) = Source (loading Nel NAMA = sin 90° ~~ sin(90+ 8) any x 8i0600+8) Ml x Assuming that the system is more stable cos 3 approximately equal to 1. .|-[vloos8) 0, = Mliy iv) ‘The magnitude of receiving end voltage x Ivl=lvJ-2 Ml=Nl Ww ie Where Q,_ is nothing but net reactive power in the system which will be modified in order to change magnitude of receiving end voltage. Shunt Capacitor: «It will modify current carried by the line Delhi | Bengauns | Bhakencavar | Viayarada | Viekbapatnam | Tapa | Pane | Chennai * [twill control under voltage in the system + Itwill inject reactive power into the system without shunt capacitor : IM Wal Tra Wal=W- FQ WVal<|Ml With shunt capacitor : iv Weal [Par QWicap 2 Qn Qin Wal=W- jy Om Qace) Mal > al but it is economical to have [V,.| < |V, ‘Whenever |V,, is very high so that the rating of the shunt capacitor will increase which in then increase the cost of supply. The shunt capacitor will also improve the power factor of the load apart from improvement in voltage...However_ the improvement in power factor is high when compared to improvement in voltage. = |V\|, the reactive power required So practically shunt capacitor will be treated as power factor correction device ‘Transmission & Distribution The improved power factor will reduce the KVA demand of the consumer so that the fixed tariff will reduced. Due to shunt capacitor there is no change in real power demand, Qa A geo 0862 > cos i The reactive power supplied by the shunt Capacitor = Qi-Qr Qieap = Si sings ~ Sz sing Quicap = P (tang; = tangz) IfP is 3~ phase real power consumed by the load then Quiap is the reactive power require to be injected by capacitor bank . During operation, At the designing stage, Quo (3=#) aio Normally the shunt capacitors are connected in delta configuration across the supply. Ciph = Seer 8) SAEED) erated | Now Dewi | Benpur | Bhutaneswar | Visyewad | Viapatam | Toupa | Pune | Cheanat Engicering Academy Power Systems Example 2.6. At an industrial sub — station with a 4 mw load, a capacitor of 2 MVAR is installed to maintain the load power factor. at 0.97 lag. If the capacitor bank is out of service, the load power factor will be (a) 08 lag (©) 0.9 lag (0) 0.75 lag (@) 0.85 lag Sol: Reactive power supplied = P{tan;-tango] 2=4 [tan $ ~ tan (cos”'(0.97))] tan 4; = 0.7506 1 = tan”(0.7506) = 36.86 Power factor = cos § = cos (36.86) = 0.8 lag Shunt Inductor : ‘When the transmission line is operating at no-load (or) light load, the receiving end voltage is more than the sending end voltage. In order to reduce the receiving end voltage a suitable shunt inductor is connected at the load bus to absorb excess reactive power from the system. without shunt inductor IM Wal © Piowd~JQioad Voss Toad With shunt inductor : Iv Waal 7 ate Quis | Quad + Qaring — VV W-jy CQ Qa) Na x V1 (ne ~Qas yy! ; Nal < [Val but itis economical to have [Vl > [V I h bh Va 4 : Yan Ih h coséy Power factor will be improved by placing shunt ‘eactor only when ever the load is leading which is not practical so shunt reactor is not a practical power factor correction device Practical shunt reactor will be called as Ferranti effect control device. ‘The reactive power absorbed by shunt Inductor Qurina = P (tandy - tandi) During operation — Quins /Ph = X,/ph Quiet’ 4) =3 ve Qurioa 9 — f At the designing stage , 3V jn L/ph=——* Quin RSET ad |New Dat | Ba | Bsbnevar | Vina | Vilas | Tia | Pe | Chea Oat Tinsen ‘Transmission & Distribution Example 2.6.2: In a 400 kV power network, 360 KV is recorded at a 400 kV bus. The reactive power absorbed by a shunt reacor rated for 50 MVAR, 400 kV connected at the bus is Sol: At rated voltage (400 kV) — Quactor SOMVAR Queactor «© V At Vi = 400 KV, Queactor! = 50 MVAR AtV2 = 300 kV, Vi We 360" 400° =40.5 MVAR Quescoe2 = Quer * = 50x Series Capacitor: The voltage control can be reactance of transmission line . done by changing without series capacitor AV, = IR cos, + IX, sind, With series capacitor AVe= Xe sin by Due to series capacitor © Maximum power transfer capability and stability of system increases . Fault level in the system increases During fault a large voltage appears across series capacitor such that the capacitor may be damaged. So the series capacitor will be protected by vsing spark gap . * There is a subsyncbronous resonance occurs in the system such that shaft of alternator will be damaged Example 2.6.3: “The steady state stability limit of a transmission Tine is 400MW. If 30% series compensation is added to the line then the new SSSL of transmission line willbe (Resistance & capacitance of line are neglected, voltages at both ends of line are remains unchanged) (a) 400 MW (») $714 MW (©) 280 MW (@) 13333 MW | sot: P,,, = Well¥el x, I | Ke = 400 MW [Msi Iv IX | RX: Iva Weal Psst Quat cos ¢, With series compensation -iKe AV2= IR cosh, + L(Xi- Xo) sing, AV2 [Yul The series compensation will be considered if (QR) is very high The percentage compensation is Fex100 The amount of reactive power of series capacitor will be Quem/ph = PX¢ /ph The voltage boost produced by series capacitor Is) Let Ml = LNeIIY, | mem) = 7X MW = 571.5 MW 07 Example 2.6.4: If aline is 100% series compensated it may result into series resonance at (a) power frequency (S0Hz) (b) 100 Hz (@} 150 Hz (@) none Ans: (a) Differences between shunt and series capacitors: Shunt Capacitor: (i) The voltage produced by shunt capacitor is more uniformly distributed throughout its length. (i) There is no change in stability. ii) There is an improvement in load power factor. (iv) For the same voltage boost, the reactive power required is high. Series Capacitor: (i) The voltage produced by series capacitor is sudden and boost at the points where it, is connected. Gi) Itwill improve the stability (iii) No improvement in load power factor. (iv) For the same voltage boost, the reactive power required is less. Synchronous Condensor or capacitor: It is an over excited synchronous motor operating at no load condition . At no load condition real power drawn by the synchronous motor is zero where as the reactive power injected by the synchronous motor into the bus given as OnuCph)=(e-y) ; Power Systems Synchronous I ‘Mor ae During over excitation field current given to rotor field winding is very high so the flux produced is more such that |E|>{V) The synchronous condensor is similar w shunt capacitor so this can also be called as power factor correction device . The rating of synchronous condensor for power factor correction is Quyncond = P (tangy ~ tanga) Power factor of synchronous condenser is ZPF lead, Synchronous Coil or Inductor : Itis an under excited synchronous motor operating at no load condition . At no load condition real power drawn by the synchronous motor is zero where as the reactive power will be absorbed by the synchronous motor from the bus which is given as Ogu ets)= My) s During under excitation field current given to rotor field winding is less so the flux produced is low such that [E| <[V| The synchronous coil is similar to shunt inductor 50 this can also be called as Ferranti effect contro} device Power factor of synchronous coil is ZPF lag. Differences between shunt capacitor and synchronous condenser: Shunt Capacitor: (1) There is a step control in reactive power. (2) The cost is less. (3) There is no consumption of real power. (GREET esta No | ena | hater | Viola | Veiga | Trop ne | Gma ] ACE agin eademy HT: Transmission & Distribution (4) There is no stability problem even large variation at load voltages. (5) There is no effect on fault currents. Quwce= Quod Qn Synchronous Capacitor: ee ; (1) There is smooth control over the reactive Pue= FM ns power. 7 (2) The cost is high 3) Due to mechanical loss, there is some real Mista Qn = [Elc0s5~|V]] power loss even though itis under no-load. X, (4) The synchronous motor will get asynchronized if there is large variation in voltages. (5) It will rise the fault levels during short circuit fault, Example 2.6.5: Ani induction Motor operating at 0.8 pf lag consuming 300 kw. A. zero. real power consuming syrichronous motor is connected across the induction motor to raise the p.f. to 0.92 lagging. The reactive power drawn by the synchrenous motor is (@) 97.2. kVAr lag (036 KVAr ag (b) 97.2-KVAr lead (@) 36 kVAr lead Sol: Reactive power supplied=P(tanp:~ tand:) = 300{tan(cos' 0.8)-tan(cos"'0.92)] =97.2kVAr Lagging reactive power supplied — means leading reactive power absorption <. Synchronous motor will draw 97.2 kVAr lead. Synchronous Phase Modifie The loaded synchronous motor with over excitation is called synchronous phase modifier. ivi Powe Qn IV] Pras iQ Polo The phase modifier will inject reactive power only for [Eleos 8 > |V| The power factor of phase modifier > v Lb it b> or cosb2> cos power factor got improved . ES eee RY f ‘ je Si J Qa Lt The reactive power supplied by synchronous phase modifier 1 Q = $1 sini ~ Sz sings tad, — Pe tan Hyderabad | New Deli | Bengalans | Bhubaneswar | Visyamaa | Viakhapainam | Tirupati | Pune | Chennai ACE 22 Fngineering Academy 248: ‘Power Systems Example 2.6.6: In the following system, before connections part B, part C power factor is 0.6 lag. After connections part B, part C power factor is 0.8 | lag. ‘Then which of the following option is correct. The combination of conductor and its insulator is, called cable. Alll electrical cables consists of three essential points: i) The conductor for transmitting electrical power, ii) The insulation, an electrical insulating medium, needed to insulate the conductor from direct contact with earth or other object. iii) Extemal protection against mechanical damage, chemical or electro chemical attacks, fire or any other dangerous effects external to | the cables. © Copper conductor has. extensively been used for cable but of aluminium is being used toa considerable extent. # A stranded conductor is expressed as Joy where the first number stancs for the number of strands used and the second number corresponds to the diameter of each strand in mm. Sometimes the second number given corresponds to the gauge of the strand used e.g, 3/20 which means a cable with three strands each of 20 SWG. The main requirements of the insulating materials used for cable are: i) high insulation resistance ii) high dielectric strength iii) good mechanica! stoperties (@) Part -A is an under excited synchronous motor, part - B is a shunt eapavitor (b) Part -A is an over excited synchronous motor, part —B is a shunt eapacitor (©) Part -A is an over excited synchronous motor, part - B is a shunt inductor (@ Part ~A is an under excited synchronous motor, part —B is a shunt inductor. Ans: (a) 2.7 Under Ground Cables Various insulations: i) Vulcanized rubber Dielectric strength is 10 kVimm to 20kV/mm, ii) Butyl rubber iii) Silicon rubber Dielectric strength is 1.2 kV/mm to 3kV/mm. iv) Neoprene rubber v) Polyvinyl chloride Dielectric strength is 17 kV/mm. vi) Impregnated paper. Dielectric strength 30kV/mm, is 20 kV/imm to Under Ground Cables are used in the place of overhead lines to have following advantages and disadvantages: Advantages: 1. Ina densely populated circuits where overhead lines are not possible. ‘Under ground cables provides better regulation The chances of accidents in under ground system ate very low compared to over head lines. ‘AS the cables are laid under ground with better insulation, the chances of failure or fault are less compared to over head lines. 2, 3, SRR ad | New Dani [Benes | Bhs | Vind | Vinitapame | Tia | Pe | Ciena ‘Transmission & Distribution Disadvantages: 1. Under ground cables are very costly as compared to overhead lines, 2. Practically, identification of cable faults is difficult than in case of faults in the overhead lines. 3. Jointing of cables is difficult. Hence tapping for loads and service mains is not convenient in under ground system, Classification of Cables: 1. According to number of conductors. It can be classified as single, two, three ot four core cables. 2. According to the voltage i) Low—tension (L.T) cables upto 1000 ii) High — tension (H.T) cables upto 11,000 iii) Super — tension (S.T) cables upto 22kV to 33kV iv) Extra high — tension (B.H.T) cables from 33KV to 66kV v) Extra super voltage (E.8.V) cables ~ beyond 132kV. 3. According to the nature of the application of insulation and lead sheath. i) Belted type ii) H-type iii) S.Ltype iv) HSL, 4. According to the methods of improving the dielectric stress i) Solid type upto 33 kV ii) Oil filled type upto SOOKY. iii) Gas pressure type. upto 235kV Construction of Cables: a) Cores or conductors: The conductors are made of tinned, copper or aluminum and are usually stranded in order to provide flexibility to the cable. b) Insulation: - Each conductor is provided with a suitable thickness of insulation, depending upon the voltage the commonly used materials for insulation are impregnated paper, vamished cambric or rubber mineral compound. ¢) Metallic Sheath: - It is provided to avoid the entry of moisture content from earth surface to insulator. ‘The material used for sheath is Al (ot) lead 4) Bedding: It consists of a fibrous material like jute or hessian. ‘The purpose of bedding is to protect the metal sheath against corrosion and from mechanical injury due to Armouring. 4) Armouring: It consists of one or two layers of galvanized steel wire or steel tape. Its purpose is to protect the cable from mechanical injury while laying it and during the course of handling, ©) Serving: - In order to protect Armouring from atmospheric conditions, a layer of fibrous material (like jute) similar to bedding is provided over the Armouring this is known as serving. Armouring. Lead eat Conductor Insulation Bedding Serving ‘Types of cables: 1) Single core 2) Three core. 1. Single core: It consists of stranded copper conductor a belt of insulation of impregnated paper and lead sheath over it. The sheath is protected by covering it with hessian tapes or jute which is soaked in some preservative compound of bituminous nature. D D 2. Three core cable: This cable is used for the 3 phase power transmission in which all the three cores will be placed inside a single cable. Fijderabad | New Delhi | Bengals | Bhubaneswar Al agg hey 250: Power Systems Insulation resistance of single core cable: Let ‘I* be the length of the cable in meter. “p” be the resistivity of insulator in © - meter ‘r be the radius of single core cable of conductor. ‘R’ be the internal sheath radius = 23 (8) Ont Ar The insulation resistance. of a cable is inversely proportional to its length, Insulation resistance, R, Capacitance of a single core cable : Let ‘R’ be the internal sheath radius ‘be the conductor radius E> bo fr, is the permittivity of free space £55 8.854 x 10-7 F /m ‘er’ is the relative permittivity of the insulation. “2? be the charge per meter axial length of the cable in coulombs. Potential difference between the conductor and sheath. Capacitance of the cable is 2 PRE Pi i @ Electro static stress distribution in cable cross sectio ‘V = potential difference between core and sheath Electric field intensity at any point (x) in cable ‘cross section is h ~2mex E, =—Y—kviem xn(®) r On surface of conductor (x=), Es Vv r wn(®) t On surface of sheath (x=R), By =v kv em nf) ‘The electrical stress is maximum at the surface of conductor and the stress is minimum on the surface of sheath. By- distributing the stress uniformly, the breakdown of insulation can be avoided this may be Two methods. (a) by using metallic inter sheaths (b) by using insulating materials of different dielectric constants. kV/cm Inter sheath grading: The inter sheaths made of metallic cylinders one or more are inserted in the dielectric between the conductor and lead sheath to fix up the potentials at that distance from the surface of core in the insulation. ‘The inter sheaths do not carry any part of working current, but carries the current which is the difference between (EERE I tern | New Dats | Bengar | Bhobancowar | Viavavad | Visahapatam | Tirwpa | Pane | Gheaal ‘Transmission & Distribution the charging currents taken by section on each Ses ‘Two inter sheath are inserted between cable throughout length in = V— Vile In(dy/d) Bowax= Vi—Valnin(do/d) Samer = Vo! reba Dids) The stress can be made to vary between the same maximum and minimum value by choosing di and dh such that = dof = Dida = o = Did; a= YD/d di=ad = od the max stress are required to be made equal, we have imax = Samax = 3max V2 = Vul(l +1/a) = V/ (1+ Lat Wo?) One intersheath dyd = Did, = a @ =Did a= Did di = ad Vi = (14/a)V (1+1/a+1/02) Soa With two intersheath /gmx with out intersheath d/2én(D/d) a <10 tata intersheath /gnax intersheath = 2( 14a) The new max stress with two intersheath is only ‘gnax with one with out 3/ (+a. +07) Times that of stress with intersheath. out any Capacitance Gradi The insulation may be made of dielectrics of different permittivity such a cable is known as graded cable and the arrangement results in a more uniform stress in the dielectric. Lot di be the diameter of the dielectric having permittivity ki and D the diameter of the dielectric having permittivity ko 2n 1 1 a : én(a, /d)+5-€,(D/4,) Bimax = Q/ kid *Y intimes of V ov ‘Alena, /d)+k, 7k, ée(Did,)} Sime = Most economical radius of core(r): The most economical radius of core (1) will be designed in order to minimize ‘E” on the surface of core by keeping operating voltage(V) and radius of cable(R) as constant is =2.718 = 0.368 The most economical radius of core is r= 0.368R, Example 2, : A single-core length 5 km having diameter of -core as 2 ¢m and diameter of cable as 4-cm. if the © resistivity of dielectric material is 1.6% 1 Sol: With most economical radius of core, R= Pe 2nt wa) a ew HT ears | Dever | Vga | Wasa | Tra] Rs | Ob Q net Fcsen Power Systems _ 16x10" 2nx5000 = 50.93MQ Power factor in cables: The capacitance effect of the dielectric and the intersheath, a charging current flows from "ve supply the value of this charging current be ag V/VoC = @CV Where o=2nf ‘The charging current leads the voltage V by 90° and the total current drawn from the supply is the Phasor sum of currents V/Ri and @CV shown in below fig. ‘The total current I leads the voltage by an angle 6 where cos } gives the power factor of the cable. Cos $= cos(n/2 - 8) = sin 5 = (since 3 is small) In other words power factor of the cable = 8 ‘When 8 is small, sin 5 = tan 8 = 3 Therefore, 8=V/Ri . 1/aC V ~ WoC Ri Where !/ Rs = G = conductance of the cable. ‘The power factor of the cable is G/oC. Capacitance in 3-core eable: The three-core cable has capacitance between the cores and each core has capacitance with sheath is shown below fig. ‘Capacitances Ce t0 the care are in delta and can be replaced by an equivalent star attangement shown in below fig. 2 CG ‘The impedance between core | and the stat point rust be equal to (1/3) times the impedance of each branch of the delta, this gives Vol, = 130@Ce (or) C= 3Co. ‘The star may be assumed to be at zero potential and if sheath is also at zero potential the capacitance of each conductor to neutral is Co= Cy + Cs = 3Ce HC5 Methods of calculation of Cs & Ce: Method 1: Let conductors 2 and 3 be connected to the sheath .Capacities Co between conductors 2 and 3 and Cs of conductors 2 and 3 with respect to the sheath are eliminated, he o\, Capacitances Ce and Cs are now in parallel across core one and the sheath and they add up Measure the capacitance between core one and the sheath, is Cy = C5 #2Ce o Method 2: All the conductors are connected together and capacitance C, is measured between them and sheath, = 3Cs or Cs = 13 CG @ Since 2Ce = C:~Cy Co = 4(C1- 18) 16 (3C1 - C2) Cy =C per phase From Land? C,=C,+3C.=3/2C - C/6 3 ‘Transmission & Distribution Method 3: Connect any one conductor to sheath, measure capacitance between remaining two conductors Cc, =2C, Example 2.7.2: A 40 km long cable has a capacitance of 0.15 BF/km between any two conductors when the other conductor is connected to sheath. The supply voltage is 33kv, SOHz, 3-p. Determine the charging current Ipn. Sol: 1=40 km Coq = 0.15 pF/km Ciph =2 x Cog = 2x 0.15=0.3 uF/km 33K 13 X,/ph (1 2 —|/ph oc}? 33K | = = 2nfC/ph v3 a I, /ph = 33 x10° x 2nx 50% 0.3 uR/km, = 1.79 A/km ©. Liph = 1.79 x40 = 71.8 A 2.8 Over Head Insulators ‘The insulators for over head lines provide insulation to the power conductor from the ground. The insulators are connected to the cross arm of the supporting structure and power conductor passes through the clamp of the insulator. The insulator is to avoid leakage of current through the supports to the earth. Thus the insulators play important role in the successful operation of over head lines. Every insulator must have following properties: 1. Mechanically very strong in order to withstand the load due to the weight of the conductor wind load ete, 2. High insulating resistance in order to avi Teakage of currents to earth, 3. High relative permittivity in order to give high dielectric strength. 4, High ratio of puncture strength to flash ~over voltage. 5. The material should not be porous and there | should be no effect of change in Temperature 6. It must be free from intemal impurities, cracks. 7. It should be impurities to fluids and gases of the atmosphere. Materials used: 1. Porcelain 2. Glass 3, Steatite 4, Synthetic resin Porcelain: 1, Now a days it is used extensively for the manufacture of insulators, 2. It is produced by firing a mixture of Quartz, Kaolin and Feldspar at a high temperature. 3. It is mechanically stronger than glass. ERLE ected eat fare Dhnear | Vinod | Viniaparam | Tru | Pe | Chena ACE Engineering Academy 4. 5 6 & Power Systems Its surface is not affected by ditt deposits and it withstands temperature changes. Ithas dielectric strength of about 60 kV/em and acompressive strength of 7000 kg/cm? Itis used upto 33 KV. Glass: Glass is cheaper than porcelain in. simpler shapes. It gives high resistivity, Dielectric strength of about 140 kV/cm, It has higher compressive strength than porcelain. Its coefficient of thermal expansion is less. It is used upto 25 kV under ordinary atmospheric conditions and upto 50 kV in dry atmosphere, Steatite: \. 2, 3. Steatite is a naturally occurring magnesium silicate usually found combined with oxides in varying proportions. It has a much higher tensile and bending stress than porcelain. These insulators are used at tension towers or ‘when transmission line takes sharp turn, Classification of insulators: 1. Pin insulators 2. Shackle insulators 3. Suspension insulators 4, Strain insulators Pin insulators: 1 2, It can be employed for operating voltages upto about 25 KV satisfactorily, ‘An adequate length of leakage path is obtained by providing the insulator with two or three petticoats or rain sheds. Multipart pin type insulators are used upto 33 KV and beyond which their weight is more and uneconomical. The cost of pin insulators increases much more rapidly than the voltage, Costav' where x>2 Pin type insulators are not suitable for voltages higher than 33 kV. Suspension insulators: 1. Suspension type insulators are used for higher voltages (beyond 33k). 2. These insulators consists of one of more insulators units flexibly connected together and adopted to be tung for the cross-arm of the supporting structure and cary a power conductor at its lowest extremity is known as string insulators. 3. Bach insulator is designed for 11 kV and hence for any operating voltage a string of insulators can be used. For example for 132 kV transmissions, the number of insulators required is 12 4, In case of failure of one of the units in the string, only that particulars unit needs replacement rather than the whole string. 5. Low mechanical stress. 6. The operating voltage of the existing transmission can be increased by adding suitable number of dises in the string instead of replacing all the insulators as it is necessary in case of pin type insulators. 7. It is used horizontal plane rather than vertical plane. Voltage distribution across a string of suspension insulators: A string insulator may be treated as a combination of capacitances connected as shown in fig. Let V = operating voltage per phase en (4) es sf <—4 < est [SSRI Nea ane | hata | Vine | Vndtapne Tra | ne] Choa] 285: ‘Transmission & Distribution Vi, Va, V3, Va = voltage across various units SVIAVEEVS HVE oes som (1) Mutual capacity | Capacitance to ground Inthe fig., b=h+la = VimoC+V; oC = Vi @C (m+ 1) @ b=h+la => V3 moC = V; meC + (Vj; + V2) oC = Vsm=V2(n+1)+Vi From Equation (2), msimely m Similarly V, can be expressed as, =btls => VamoC = V3; m@C + (V1 + V2 + V3) oC 2 my +3m+1, V+Vilm+l) => VymoC = m m 2 me 43m41y m “yom tom tsmtly, m From the above four equations, values of potential across each unit can be determined. | * The voltage drop across the unit nearest the cross arn is minimum and it goes on increasing as we go towards the power conductor The voltage drop across the unit nearest the power conductor is maximum, * Mutual capacitance of each dise is same, © The current through the top most unit is minimum, Current go down the unit towards power conductor goes on increasing being the maximum, voltage across the string nx voltageacross the unit String efficiency = near the power conductor sov forthestring ‘nxsovof one disc Where ‘n’ is the number of insulators in the string, sov is spark over voltage. * String efficiency always less than one or unity. * The voltage across various units will tend to be equal in case the value of ‘m’ is large. * In the case of high voltage lines since the clearance between the conductor and the tower structure should be more to avoid flash over under normal operating condition, . ‘© The mutual capacity being fixed the ground capacitance goes on depending with larger clearanee and hence the ratio of the two capacitances goes on increasing. * The unit nearest the cross-arm should have the minimum capacitance maximum reactance and as we go towards the power conductor the capacitance should be increase by using grading equalizing the potential all units. Disc type insulators are used in different voltages: 66KV 132 kV 220K 400 kV 9-10 15-16 22-23 Voltage kV No. ofdises 4-5 Strain insulators: These insulators are special mechanically strong disc insulators used to take the tension of the conductors at line terminals and at point where the line is dead end. For example road crossing, junctions of overhead lines with cables, river crossing, at angle towers where there is change in line. Shackle insulators: These are used for LT lines, these are used in section poles, end poles, on sharp turn and for service connection, These can be used either in a horizontal or in a vertical position. [XCAR ad Ne Dai Dene inter] Voge | Vena | Toop Re | Cha Power Systems String efficiency can be improved: 1. By using longer cross ~ arms. 2. By grading the insulators. 3. By using a guard ring. Grading is served two purposes: 1. Equalization of voltage drop across the units. 2, When used with arcing hom it protect the insulator string from flash over whenever an over voltage (under normal or abnormal condition) appear between the tower structure and power conductor. Causes of Failure of Insulators: 1 2. Porosity. 3. Puncture of weak porcelain. 4. Shattering by power arcs. 5. Flash over caused by dust deposits 6. Failure from mechanical stresses. 7. Short citcuits by birds and similar objects. Example 2.8.1: Match List-I (Insulation Type) with List-I (Purpose or Configuration) and select the correct answer using the cades given below the lists: (IAS-Ot List-I (Insulation Type) A. Pin type B. Suspension type C. Strain type D. Shackle type List-II (Purpose or Configuration) 1. Low voltage distribution lines 2. String of insulators in horizontal position 3. String of insulator in vertical position 4, For voltage up to 33 kv Codes: A BOC D @4 2 3 1 m1 3 2 4 @4 3 2 1 @ 12 3 4 Sol: (¢) Example 2.8.2: If the voltage across the string of a string insulator assembly is 38 KV, number of insulation discs are 4 and valtige across. the lowermost dise is 12 kV, the string efficiency is Sol: Total voltage V = 38 kV Number of dises, n= 4 Last dise voltage, Ve= 12 KV. v8 4x12 =19.17% Example 2. In a three unit insulator string, voltage across the lowest unit is 17.5 kV and string efficiency is 84.28%. the total voltage across the string will be equal to Sol: n= V3=17.SkV 84.28% Mag SO, Ming = Vv Vv 0.8428 3x175 ‘Total voltage, V = 44.25 kV Example 2.8.4: A string insulator has 4 units. The voltage across the bottom ~ most unit is 33.33% of the total voltage, its string efficiency is, 33.33% of V =0.333V v Sy, v ~4x0333V = 75% RSET cain | New Daas | Bexglry | Bahar | Viiyaaa | Visiapatna | Tira | Pe | Chennai] 187: ‘Transmission & Distribution 2.9 Concept of Corona Corona: The ionization of Insulating material (air) surrounding the sueface of the conductor of @ transmission line (or) the disruption of the dielectric strength of air near the conductor of a transmission line. Description: R @e) a Y Transmission line B On economical basis, most of the transmission system is overhead system. There are certain free electrons which are available in the space in which some of the electrons are very near to surface of the conduetor and some of them are away from the surface conductor. The electrons near the power conductor will move at a faster rate than that of electron away from the surface of the conductor, because the electric field intensity is maximum at the surface of the conductor. The electric field Intensity wills proportional to the operating voltage of the system. Whenever the operating voltage is more than 270 Ky, the field intensity will also increase so that the mobility of the electron will increase. ‘The high mobility electron will collide with the other air molecules, which will dislodge the electrons from those molecules. The process is continuous and at certain instant the electron avalanche will takes place ice., an are will strike between the two lines. The arc is an ionized body, which will be treated as line to line fault, The reliability of supply is reduced. In order to eliminate the arc, the distance between ‘the two lines are increased in a non-linear manner as the operating voltage increase, so that the field intensity will be restricted to be maximum at the surface of the conductor and almost zero between the two conductors. However the conductors which are employed for transmission line are stranded conductors having rough surface. Due to rough surface certain impurities are to be deposited. So a self sustained field emission will be initiated around the surface of the conductor at certain points, The voltage at which the self sustained discharge will be initiated is called the critical disruptive voltage. The corona initiation can be identified as a) Hissing noise. byReleasing of ozone gas. ©) Occurrence of beds and Tufts Critical disruptive voltage of a 1-phase 2-wire transmission line: a(pr bor a —| Critical disruptive voltage Vs= gor (3) r = radius of conductor in em d= distance of separation in m, 0 = Dielectric strength of air 30 kV/em (peak) 1.1 kV/em (rms) at NTP However the existing temperature and pressure at the surface of the conductor are different from NTP. So at any other temperature and pressure Va=s't (2) kV/ms T ETI) certs | New Det | Bergan | Bhabancowar | Viera | Viakapatam | Tirupad| Pune Ost AC Kcsteay Power Systems g' = Dielectric strength at any temperature and pressure. 2 = B6 5 = air density factor eiazel UB+t 8 = air density factor h= Atmospheric pressure in em of Hg. t = temperature in C°. If the surface of the conductor is imegular then surface irregularity factor (m) will be introduced. Va= 211 mérin 5 kVirms r m = 1.0 for smooth conductor m = 0.93 ~ 0.98 for rough (or) stranded conductor ‘m= 0.83 ~ 0.87 for seven stranded cable ‘m= 0.9 for more than seven stranded cable Visual critical Disruptive voltage: when the corona is initiated it will not be visible. In order to visualize the corona, further ionization needs to be done at the surface of the conductor. 208 XX x The visual critical disruptive voltage W=21 1, _ }o(2) kV/rms m, = surface irregularity factor my = 1.0 for smooth surface of the conductor ‘my = 0.93 ~ 0.98 for rough conductor (ot) stranded conductor ‘The visual corona will be observed as white bluish colour. Example 2.9.1: ‘Under NTP conditions the dielectric strength of air is 30 KV/em (peak) what is the dielectric strength of air at a pressure of 72 m of Hg and at a temperature of 30°C. Sol: any t,h dielectric strength of air is ¢= go.6 3.92h 244 Ath= 72 omat Hg, t= 30°C =3 2x? _o o434 273430 0 > 0.9434 kV/em (peak) = 28,302 kV/em (peak) Critical Disruptive voltage for 3-phase system: GMD r Vaeata mse ( )eVimspase GMD = Mutual distance Local corona: lonization of air at certain points along the power conductor When the conductor is deposited with ice. The ‘m’ value for local corona is 0.72. Decided Corona: The visual corona at certain points when the conductor is deposited with ice. The ‘m’ value for decided corona will be 0.8. Corona loss: When the corona initiated on the transmission lines, certain real power loss will takes place. The amount of real power loss is called as corona loss, P=241 ao) Fv, ~ V2) KW /ph fkm £= supply frequency in Hz 8 = air density factor += radius of conductor in em. d= distance of separation in m. ‘pir operating voltage / phase/ rms. Va= critical disruptive voltage rms/phase Note: When the operating voltage is more than critical disruptive voltage, then only the corona will be initiated. Other wise there is no corona. 6) eagicetig Aademy 1595 ‘Transmission & Distribution. 7 loss will be same. In case of unsymmetrical Example 2.9.2 conductor configuration, the capacitance of the At S0Hz, operating frequency a HVAC transmission has a corona loss of 1.2 KW/ph/km. What is the corona loss of the same line operating at 60Hz. Se I: Phass © (£+ 25) At f)= SOHzZ, Plogs =1.2 kW/ph/km Ath= 60 Hz, Pioss2. =P % +25 5 O25 50425 #12 = 1.36 KWiph/km Factors Influencing corona loss: 1. Electrical factors: (a) Supply frequency increases the corona loss will increase, because P a (£125). (b) The corona loss of Ac Transmission line is more than DC transmission line for the same operating voltage. (©) The corona loss at positive polarity conductor at a DC transmission line is more than negative polarity conductor. At the positive polarity conductor the electron will move at a faster rate due to positive electric field Intensity and negative polarity electron. So the ctitical disruptive voltage is less and corona Joss will be high. (@) when the sinusoidal wave form is having distortion (i (©) lower the height at the power conductor, higher will be the corona loss. Due to lower height, the capacitance of the conductor will increase so that the operating voltage will increases. The increased operating voltage will provide high electric field intensity so that the critical disruptive voltage is less which in lum increases the corona loss. (® Conductor configuration: when the transmission lines are arranged in equilateral form, the capacitance / phase will be same so the operating voltage will be same, which in than the critical disruptive voltage will be same for the three conductors, So the corona middle conductor is high, so the critical disruptive voltage is less and the corona loss is, high. 2. Atmospheric factors: (a) Temperature and pressure: Both are related to air density factor. 5 392xh 27B+t Px 1/8 (or) Po (Vp—Va)” (or) Va ccd ‘As *®’ increases the corona loss will decreases. As 8 increases Vg will increases so (Vp-Va)” will reduce which will reduce corona toss. In hill area the corona loss is high when ‘compared to sea level area. In hill area the falling of pressure is very high when compared to falling of temperature. So “8? will be less in hill area and corona loss will be high. (b) Deposition of Dust, ice, snow and for: These are all impurities to the surface of the power conductor, which will reduce the dielectric strength of air so. that the critical disruptive voltage will reduce and the corona loss will increase. 3. Factors related to the size af the conductor: (a)-use larger diameter of the conductor: Poe vr... (1) Pot (Vpn ~ Va) (2) Vaer ...G) As the diameter increases, there is a possibility to increase the corona as per equation (1) However in equation (2) it will be reducing, because Vy will increase a per equation (3). The fall in (Vpi-Va)" is more predominant than that of raise in Vr term. (b) Use more than conductor per phase: By using more than are sub-conductor/phase the self [ER ee ca New De | eng | Distonervar | Vipera | Vinkapemam | Tira | Pe [ Cheaal_] O snscb ten Power Systems distance will increases. So the critical descriptive voltage will increases. The corona loss is reduced. (0) Profile of the conductor: By using cylindrical shape of conductor the field is more uniformly distributed throughout the cross section at the conductor. So the Vq will increases and the corona loss will be reduced, Disadvantages of Corona: 01. There is certain real power loss a part from ohmic loss. 02. When the corona is initiated on transmission lines, the overall diameter of the conductor will increase so that the capacitance will increase. The net charging flux which will link with the neighboring communication line is not zero. So there will be communication Interference. Apart from the charging flux, the zero sequence components during ground fault will also cause interference. The part of power system by which electric power is distributed among various consumers for their local use is known as distribution system. A low tension distribution system is shown in fig. below, | which comprises the feeders, distributors and service mains. Service mains @ ‘Consumers Distributor 1. Feeder: A line or conductor which connects the major station to the distributor is known as feeder. Advantages: Corona will acts as a safety value against direct lightning stroke, by dissipating the peak magnitude of lighting stroke in the form of corona loss. ‘Methods of reducing corona loss: (1) use larger diameter of the conductor (2) use hallow conductor 3) use bundle conductor Corona loss will be high on a stranded conductor and it will be less on hallow conductor, However the corona will exist beyond 275 ky transmission system only. So bundle conductors will be used in which each sub conductor is a stranded conductor. Example 2.9.3: Which of the following equipment placed at the terminal of HV bushing of a HV power transformer to reduce corona loss, (a) Bundle conductor (b) hollow conductor (©) stranded conductor (d) corona ring Ans: (d) 2.10. Distril bution System 2. Distributor: A line or conductor to which varios consumers are connected through service mains is known as distributor. Service Main : A line (conductor or cable) which connects the consumer to the distributor is known as Service main, 3. Classification of distribution system: 1. As per the type of current i) DC. distribution ii) A.C distribution 2. According to construction i) Overhead distribution system ii) Underground distribution system 3. According to the service i) General lighting and power ii) Industrial power ailway ED iets Kona tea iv) Street lighting ete Bnbanewar | Viivavada | Viakhapatnam | Tirupati | Pane | | ACE Engineering Academy 4. According to the number of wires i) Two wire i) Three wire iii) Four wire 5. According to scheme of connections i) Radial system ii) Ring system iii) Interconnected system DC Distribution : The electric power is almost exclusively generated, transmitted and distributed as a.c. but for certain applications (e.g for electro- chemical works, for the operation of variable speed machinery dc. motors ete.) is absolutely necessary. For this purposes, a.c is converted into dic. at the sub-station and is then distributed by i) 2-wire system ii) 3 - wire system, AC Distribution: The electric power (or energy) is invariably generated, transmitted and distributed in the form of altemating current. The main reason of adopting ac. system for generation, transmission and distribution of, electric power is that the alternating voltage can conveniently be changed to any desired value with the help of a transformer. Primary distribution: The system in which electric power is conveyed at 1IKV or 6.6 KV or 3.3 KV to different sub-stations for distribution or to big consumers (¢.g industries, factories etc) is called primary distribution system. Secondary distribution system: The system in which electric power is distributed at 400/230 V to various consumers (e.g residential consumers) is called low voltage or secondary distribution system. ii, Connection schemes of distribution system : Radial system: In radial system, separate feeders radiate fom a single substation and feed the distributors at one end only. Advantages: i) Its initial cost is minimum. ii) Itis the simplest distribution system. 261 ‘Transmission & Distribution Disadvantages: 1. The end of the distribution nearest to the feeding end would be heavily loaded The consumers are dependent on single feeder and distribution. Therefore when fault occurs on the feeder or distributor, the supply is cut off to all the consumers who are on the side of the fault away from the sub-station 3. The consumer at the farthest end of the distributor would be subjected to serious voltage fluctuations with the variation in load. Ring main system: In this system each consumer is supplied via two feeders. The arrangement is similar to two feeders in parallel on different routes. Advantages 1. Less copper is required as each part of the ring carries less current than in radial system. 2. Less voltage fluctuations 3. Itis more reliable. In the event of fault on any ‘one section, the continuity of supply t all consumers can be maintained by isolating the faulty section. Inter connected system: In this system, the feeder ring is energized by ‘wa or more than two generating stations or substations. Advantages. 1, Itincreases the reliability of supply. 2. During overload hours, the area fed from one generating station can be fed from other generating station. Thus it reduces the reserved plant capacity, improves the service reliability and increases the load factor and efficiency of the system. Voltage drop in distributors: A) Load at unity Power Factors : nat ioe bee hoy Feeding Point (Vs) h b L If], bs, Jy are length in meters. Ly, ba Ls are loads on the line in Amps. FRR 0 TT Ne Da ene Distance Vows | Ving | Tra | Pe | Came] 262: Power Systems ‘P is loop resistance of distributor in ohm per meter. Total voltage drop= [(I-+e+b).lr.1] + ((eth).L.1] + [(b).b.)] volts. Power-loss = {(r+Io+h) tra] + (a+) ox] + [(b)*.1)] watts B) Load at Different Power Factors: se Feeding Point (Vs) Leos; peasy Is cosds fl, 1, Js are length in meters. 11, fy fs are loads on the line in Amps. ‘r’ is loop resistance of distributor in ohm per meter ‘7 is reactance of distributor in ohm per meter Total voltage drop = (I(r.cos $5 + xin $)ln + [h(r.cos e+ xsin g2\(Ii+ 2)]+ [b(t.cos 63+ x.sin 45 ). (lish +b)|Votts Powerless = [cos $+ xsin Olin + [L?(cos G2 + x.sin $2)(/1+ 12)}+{L°(r.cos d+ x.sin $s ). (lis e+ by] Watts C) Uniformly loaded distributed fed at one end: ! > Va ———— ! Feeding Point (Vs) 1 total length of conductor; i= current tapped off per meter length ; r= total resistance of distributor in ohm per meter; x= reactance of distributor in ohm per meter Where i= I= Total current in amp ; Jr=R= total resistance in obm ke total reactance in ohm; + Total voltage drop = (12)IR cos 6+ X sin $) volts D) Uniformly loaded distributed fed at both ends — 1 / \ total length of conductor ; = resistance of distributor in ohm per meter x= reactance of distributor in ohm per meter where Ir=R = total resistance in ohm Ix= X = total reactance in ohm; The voltage drop upto middle = (1/8) [R Cos § + X Sin 6 ] volts Importance of capacitor in distribution system: 1. Voltage improvement 2. Loss reduction 3. Power factor improvement Series Capacitor oe Ve Feeding Point (Vs) Capacitor | Shunt ] h b h ‘© Series capacitor influence is down streamside and where as in case of shunt capacitor influence is upstream side. ‘© Series capacitor (AVB) is mainly used in voltage improvement and Shunt capacitor is ‘mainly used in loss reduction and power factor improvement. * Automatic Voltage Boosters are used for voltage improvement and these are low cost, marginally loss reductions. ESTEE) erat | New Dei | Bengsturs | Bhubaneswar | Viayavada | Visakhapatnam | Tipe | Pune | Chenna_] Per-unit system, Symmetrical Components &Fault Calculations . [actual values—units Electrical Quantities per unit values~ Nounits. Impedance ~ 20 ohms actual. Im above two cases can be studied electrically by using only one network so that only on network equation to be solved. Hence the taking will be less. -15 0 -PU Current — 100 amps—actual wey 220KV 33KV 66KV OS OO 20 -P.U f(D (2) at e841 ar) Absolute system (or) actual system 5 SLD UK Jiiyy ———— 4 synGen OHTL teins Per wnit system Inductive Joad 14 Ideal eases synGen —ONTL ‘Syn Gen "ACTUAL SYSTEM PERUNITSYSTEM ] 1-The given SLD is dealing with more than one operating voltage, the above SLD represented electrically with more than one n/w. So that there will be more than one n/w equation to be solved. Hence the time taking will be high 1-The given SLD is dealing with more than one operating voltage; the above SLD is represented electrically by only one n/w. So that there only n/w equation to be solved. Hence the time taking will be less. 2.In case of transformer the primary impedance is not same as secondary impedance, so it can be represented electrically by two networks. Ziper unit impedance, of transformer in primary will be same as per unit impedance of the transformer. in. secondary. So that it can be represented electrically by one network — IC™ 2p Q#Z,Q Zou In any per unit system, the given single line diagram will be replaced by an electrical simulation networks i.e, impedance (or) reactance and admittance (or) susceptasice networks in phase manner with respect to neutral point, under the assumption that 3 phase system is a balanced system, Y- Configuration and the associated electrical quantities will be expressed per unit values. ani) yHiyderabad | New Delhi | Bengalura | Bhubaneswar | Vijayawada | Visakhapatam | Tirupati | Pune | Chennai Electrical Simulation: Synchronous machines —> voltage source in series with impedance / reactance. Transformer —> Impedance / Reactance ‘Transformer line > Impedance / Reactance Induction Motor -> Impedance / Reactance Name Single line | Impedance! [ew diagrams _| Reactance ra Synchronous | #—| ae Generator | | bam | fo R<. MVA, Zr = Zq MA (KV) Conversion of one per unit impedance into another per unit impedance - (ii) ~ (iii) Zpunew = Zyots UN = Det a wit KV ¥ Example 3.1: Alternator is rated as 100 MVA, IL KV, x = 1.2 PU. ‘What will be the PU reactance at 250. MVA and 13.2 KV. Sol: X, a ( uy pune =12xox| | = 2.08 PU 100 "(13.2 Example 3.2: Alternator is rated as 200 MVA, 13.2 kV, X= 69 and what will be the per wnit reactance at 400 MVA and 18.2kV. Sol: Xyy =6x 200 = (13.2) 2 Xpuney = 6.88% 40 (132) 200 (18.2) =725 per unit ‘The per unit impedance of transformer will be same on both sides: ~O- Vp Vs Ip Is 2p Zs Lent Zsa Zoru = Zspu jymmetvical Components & Fault Analysis , 4 Zepy = Z, While calculating the primary side, use primary values are base values. 2m _Z, iene, Zppy = 20 a 20 2 7g tt oy ree lay ey. 4, Zend ve ey While calculating on secondary side, secondary values are base values, So that PU reactance in primary will be same PU in secondary. Xeu ay In case of both sides. A Y TF, reactance P.U. is same on os S-phase ¥ equivalent 1- Y equivalent FR NG a Rect | ner Vivo | in [Tie Re | Cl 166: Power Systems The A-connected winding will be converted into 1~ Va =Vnu + Via + Veo 4. Y equivalent, the phase voltage assumed as the Vy =Vyy +Vyg + Vyo (oS mmetsical original line to line A-connected system, The P.U. Components reactance will be same on both sides. Symmetrical Components In case of balanced load, the voltage at load p will be evaluated by using per phase reactance network along with PU values. ‘The time taking is less, because only one phase is sufficient to steady the system. Unbalanced Condition: The occurrence of fault in 3- system will result as the 3- system will be come unbalanced. The unbalanced load voltages will be evaluated by having — individual representation of phase, There will three network equations to solve in order to evaluate the unbalanced load voltage. The time taking will be high. In order to reduce the time taking for evaluation of unbalanced load voltages, it is proposed to express: any unbalanced electrical quantity by a set of 3 balanced electrical quantities and set of three balanced electrical quantities called symmetrical quantities. (or) symmetrical components namely, 1. Positive sequence components, -1 2. Negative sequence components ~2 3. Zero sequence components — 0 Vr = Vai + Veo + Veo Vy = VyitVy2tVyo. Ve = VartVe2* Vio Ira + Ino + Tro ivi + Ty2 + Tyo Ta: + Tee + Too The occurrence of fault will be unbalance in phases, so that voltage and current will be express by symmetrical components. However power can not be express by symmetrical components because, power calculation will be aiways 3-6. Three unbalanced (Ms = Vai + Ven + Veo In order to reduce no, of symmetrical components, it is proposed to replace the symmetrical components of Y-phase, B-phase, in terms of R- phase by using suitable notation which is denoted by K and it is defined as unity magnitude with 120° phase displacement in anti clock wise direction. K=12120° K?=K x K=12120° x 2120° 1.2240° =.0.5 —j0.867 K°=K x K x 1=12360° = 1.04j0.0 KAK?+K? = 0; L4K+ K? = K*KK=K KS = KK? KS=K°K?=K*=1 Faults Open Circuit Short circuit (On) (01) Series fault ‘Shunt Fault The open circuit faults are due to melting of the conductor because of over loading, or switching of the conductor due to wind effect or opening of the circuit breaker contact, Unbalanced (or) unsymmetrical faults i. Melting of the conductor in one phase or opening of the circuit breaker in one phase . Melting of the conductor in two phases or opening of the circuit breaker in any two phases, (Ea | Nev Deas | Bergan | Bhar | Vinyavada | Vibapamar | Tirupati | Pe | Ghat 67: PU System, Symmetrical Components & Fault Analysis The open circuit occurs in all three phases the system will work at no load condition then no load voltages same as rated voltages. Hence it will not be treated as fault. Short Circuit (or) shunt fault: A short circuit fault will ocewrs due to failure of insulation or falling tree branches on the wires Percent #( Cal KF Short circuit on R-L network: (Trans mission Tine) At t= 0 the short circuit is taking place in transmission line and the capacitance is shorted, ‘The transmission line is replaced by R-L series circuit with a constant excitation voltage of generator. A short circuit occurs in all three phases will result as there will be short circuit current in the phase. ‘The short circuit current is mote than load currents. The failure of insulation is mote common when compare to the melting of conductor, hence short circuit faults are more common when compare to open circuit Short circuit fault: ‘Transmission line: The transmission line is supplied by synchronous generator and it is working at steady state condition with a balanced load. The voltages and currents are expressed in rms values. ‘Name | Types of Faults | Diagrams R L | ate Me Att=0 (Short ciruit) i). Line to i Ground fault} [> 70 (LG) rad | Unbala [ii) Line to line} ) veVusin (ota) v= V2 V sin (atta) ‘ned or} — fault (LL) he 15 vuasym «-is the angle between fault point and zero cross metrical |jii)Line to line | p> ‘over point of a sinusoidal wave form, to ground | 10 L | fault (LLG) | a i) Line to Tine to line fault he (LLL) + | Balance | ii) Line to line De at d or to line to } Symme | ground | trical (LLLG) ss 1 2 Seyee kL cycle st —re-Steady> { Transient Transient state period petiod Instantaneous current is an unsymmetrical wave form, but sinusoidal due to the presence of de off set current Rt a = V2 Vsin(ot+a) Ina Sin(cotrcr-G) + ker“ (OEE od oD ei ‘Bhubaneavar | Viayanada | Viakhapatam | Tiropal | Pane | Chennai i= —Msin(ot + 0 - a) tke Bo Zz isi tiy Vsin(ot + 0.~ 6) + ke“ i, = symmetrical sinusoidal steady state current (ae component) ig = transient current (or) exponentially decaying component (DC effect) in one direction afX) L =Tan™| — |and t== ‘ lk R The magnitude of de effect ca be obtained by using the initial condition of the network ie. inductance can not allow sudden change of current at t= 0 i@)=0 0 2 vsin(o + 0-9) +k k= sin(a- 4) Case (i): «6 = 0 (or) No DC effect ixiy Case (ii): a= &, Maximum de effect ce Zz i aa sin(ot + 0-8) a sin(a-6)e* ‘The momentary current of the fault is having de off set value the heat that can be produce by de off set value is zero. So that the fault current which can be calculated is symmetrical short circuit current and it can expressed in rns. value, ER era Beatra | Bhabanewar | iar | Visapamara | Tira | Pe | Chena 268: Power Systems Synchronous generator before fault: a t Th (be) Stator Most of the loads are driven by inductive nature so that the nature of stator flux inductive the rotating flux of stator will influence the rotor flux which is called as armature is reaction and it will be slightly inductive nature. 1 is represented by armature reactance. There is a leakage flux in the stator winding and it is represented by leakage reactance. The electrical network is shown below. us E26 At t = 0, there is a short circuit fault on the terminals of generator will result as the current delivered to load is zero. But there will current upto fault point which is called as S.C, current and it is purely inductive so that the rotor air gap flux is totally demagnetized due to armature reaction. The magnetic decoupling between rotor and stator, which can be also called as asynchtonous (or) unstable, istiy i,=ac component i= de effect 6,= rotating at synchronous speed is. static flux In order to maintain stability itis require to opposes armature reaction by strengthen air gap flux with the help of rotor field winding and damper winding due to flux produced by de off set current. The electrical equivalent is as shown below to deliver the current. Xv Ai Subtrancient reactance X) =X) +X, HI) Koop) ‘The counter emf that can be present that depends ‘upon time constant of the elements. The damper winding having more resistance. So that time constant is less the counter emf will disappear from damper. It will be continued from to field winding, ‘The element equivalent is as shown below Transient period 2~5 cycles Transient reactance Xy =X, +(K, )IX, +69: PU System, Symmetrical Components & Fault Analysis ‘The synchronous generator will brought back to steady state condition by isolating fault with help of relay and C.B. X} period" state In case of synchronous generator the ac component is asymmetrical sinusoidal waveform with a variable current due to change reactance because of | Armature reaction. However, it is measured in rms | value. Fautt name | Occurrence | Severity of offault | fault (Isc) [Line to ground | Most common | Less severe (or) |_ More frequent Line to Tine | Line to line to ground | Line to line to | Less common | More severe | line (or) | ____| Less frequent | In three phases to ground fault the potential of ground zero. It can draw more current when ‘compare to three phase fault. Great [Nev Da | Benga | Bhubacovar | Vvawada | Viskhapatnarm | Tira | Pane | Chena _ ACE ‘Engineering Academy 70 Power Systems Positive Sequence Components: ‘These are the components equal in magnitude and | 120° phase displacement. The phase sequence of these components is same as that of original phase sequence of network. ‘The sequence at which the system is working before fault is the original sequence, Negative phase sequence components: These are the components equal in magnitude 120%phase displacement. The phase sequence of these components opposite to that of original phase sequence of network. Zero phase sequence components: ‘These are components are equal in magnitude and without any phase displacement. If there is no phase displacement there is no phase sequence to compare. Sequence: Direction of rotation of a rotor of a synchronous generator is the reference to decide the sequence. Original Sequence: It is assumed that the direction of rotation of rotor flux clockwise, so that there will be Clock wise induced emf in stator, which is having a sequence of RYB (or) ABC. The flux produced by stator current is also having direction of rotation in a clock wise ‘manner, which is same as the direction of rotation rotor flux. The corresponding electrical quantities are called Positive sequence components and steady state condition, ‘The reactance offered is Positive sequence steady state reactance i.€., Xq Ve=Ver Ir=Ini Ep = Eri Va=Ver ty=W Ey=Ey VW=Vvi In=Ie1 Ep= Ep, Tales eae ‘artViatVe0 BeEgrtEnatEns By= Byy+E ato Jy ty + Ha Hye Ja= Tos lect lon The occurrence of fault will make the system unbalanee and it will be represented by three symmetrical components, out of the three components one of the component is +ve sequence component. ie. the sequence which exist before fault will also exist during the fault but sub transient condition. There is an unbalance in stator winding but the direction rotation of rotor flux will be always in clock wise manner i.e, same direction as that of before fault. The rotating flux of rotor will be able to produce induced emf in stator in clock wise manner, The flux produced by corresponding current is having direction of rotation same as that of direction of rotation of rotor flux and corresponding electrical quantities are called +e sequence components sub transient condition. ‘The reactance offered is called x}. Significance of Positive sequence components: The Positive sequence components of components which are available before fault and also during any type of fault hence these components are bases for relay operation and circuit breaker operation, Tiyderabad | New Deli | Wengalura | Bhabanesvar | Visyarada | Vieuhapainam rapt | Pane | Chena ‘71: PU System, Symmetrical Components & Fault Analysis 100A = full load current ot Relay Coil ey Relay | ae Pick up value = 1.2 I (ve Sequence) (0) = 131 Set valve Nae ae sonnet The pickup value of relay will be expressed in terms of positive sequence steady state i.e. pickup current = 1210) 13 During the fault same sequence will exist in sub transient manner so that the relay will operate because the positive sequence sub transient current is more than positive sequence steady state current. ‘The relays are provided with min pick up value based on the line to ground fault so that it will operate all other faults. ) anc FC 2 During the normal operation the C.B, contacts are jue closed. If a fault occurs the moving contact is separated from fixed contact, so that an arc is developed, The circuit is isolated unless ate is break. The breaking of arc result as heat release. ‘The amount of heat release depends upon breaking current. ‘The heat released by are should with stand by C.B. contacts, The rating of CB depends on three phase fault of Positive sequence component. Negative sequence components: Whenever the direction of stator flux is opposite to that of rotor flux the corresponding electrical ‘quantities are called negative sequence components R, B, Y. (OR) anticlockwise components During the fault there will be a terminal voltage at fault point due to unbalance i.e, Vp, but there is no induced voltage in the stator because there is no anti clock wise direction of rotor ic. Ex, will be zero, Hence the corresponding current from the fault point towards the rotor. The flux produced by the correspond current opposite to that a direction of the totor flux and corresponding components are called negative sequence components. Significance of negative sequence components: The flux produced by negative sequence component is having speed of Ns however with respect to rotor, it has relative speed of 2Ns. So that negative sequence flux will able to cuts the rotor field winding which will able to produce an induced emf. Induced emf will once again produce circulating current in rotor field winding with double the frequency. The circulating current will result as the rotor field winding gets over heated. In order to protect the rotor due to over heating negative sequence relay is employed in a stator. ‘Zero sequence components: These zero sequence components will exist provided that fault must be ground fault and nearest neutral of the system should be grounded, Se 1 ' ( t ' I ‘ 1 1 t In case of grounded fault the current passes through ground and enter into the system from nearest neutral ground. The ground can provide the magnitude with out any phase. The reactance offered is called xo. Hyderabad | ii] Bengaura | Bhabaneswar | Viayawads | Viathepamam | Trupal | Pune | Chennai Power Systems Significance of zero sequence components: ‘The flux produced by the zero sequence component current are equally magnitude without any phase displacement. If there is no phase displacement there is no rotation of flux. So that the effect of zero sequence component flux will not be there on the rotor field winding. However these components are in the form of leakage components in stator winding. Ww Vor RYB ++ve Balanced Var Ver RBY -ve Balanced RY undalance d Va= VeotVertVe2 Vy = VyotVyrtVy2 Ve = VeotVer+Ve2 Vrot Veit Veo VeotK°Vai+KVRa VrotKVartKVRo 1 TVno KI V_ K? Ves + 4 un known known unbalanced balanced ‘The advantage of symmetrical components it to the reduce time taking while evaluating unbalanced electrical quantities with help of K notation. fe] 11 idk, Ik F 1K Ki], LJ lk if If a fault occurs, the system is unbalance and use unbalance quantities to calculate balanced symmetrical components. In] fi 1 17 [lp In [=|1 K? K| Ud, In| {1 KK?) Us) Zero Sequence} Vez Components | (Balanced) | 11 1 fTe] i K KUL 1, LL K? a The symmetrical components are associated with their networks which are called as symmetrical ‘networks (or) sequence networks. Symmetrical Networks (or) sequence networks: (Per phase Sub-Transient Impedance (Reactance) networks In case of positive and negative sequence network, neuttal is the reference. In case of zero components ground is reference. Tg + Tn + Ip # 0 (un balanced) TroHtTReHyo Hy t+y2t pote te2 = Irony Heo got K 2g teat eo Kgs Kf = Slots (1+K?#KJHR (I+ K+K?) =3lko =3IRo In positive and negative sequence the effect of neutral grounding is not required to include. In case of zero sequence the effect neutral grounding to be included. In Positive sequence Network: Tri Xeg=XY Eni Vai = Erle X tog Ve = xi — IiXieq is called positive sequence Network Terminal voltage equation. Negative sequence Network: Tee Xaeq=X2 Vea Epa-leaXeq Ta GRAN de | Nev Daa Berge | Bhubaneswar | Viayavaca | Vikapatnm | Trupat | Pune | Chena _ACE ‘Engineering Academy Vo = Ir2X20q | Vas = ~ IeoXoeqi is called as Negative sequence network terminal voltage equation. ‘Zero sequence Network: Consider the effect of neutral grounding, because the reference is ground. The effect of neutral grounding is in terms of neutral voltage Va=I4Ry (Resistance grounding) Va = 3IRoR, with negative sign Tro, Exo=0 Vpo= Eno-Vn~IpoXo : | a Xo lho v Tro ‘ko= —3Releo-Xolro E “Tea(BR4tiXo) —“IroZoeq ot z Varo = —InoZoeat Zero sequence network terminal voltage equation Reactance Neutral grounding: Tro Xo, 3Ra Varo = —TroX0eqi Opes Winding B 73: PU System, Symmetrical Components & Fault Analysis | Ina A connected winding the positive & negative | sequence component exists because the reference is, | neutral, The zero sequence components dves not exist the network not connected to ground. Y-winding is called open winding and phase ‘currents are same as line current, AN “Open winding Close winding B Xo ) | sermon In case of A connected zero sequence currents are circulated with in the phases but currents are not entered to line because there is no grounding of system. Zero sequence networks of 'T/Fs: ter Xr : Xo + ‘Neutral grounding In order to inchide neutral grounding effect of TF. The T/F can be represented mechanical equivalent as series parallel switches, ey —_| | — Series winding — Y winding Parallel wind > A winding Series Switch — Y (open winding) Parallel Switch — A (closed winding) GRRL eas | Nee Deh | Bese | Btencvar | Viravade | Vint | Tat [Pune | Chana ® ACE ‘Engineering Academy Power Systems Yr tah — Che 3x9 —Closed path — Close — LX A [S.No | Connection | Impedance/Reactance —0 a0 0 10 : om | YY PO Xe u AA 2 iv Po” Xqy 3X 8 3 ¥ We sve | Zee - name | Sequence | sequence | sequence —agp_ago 0 __. | Reactance | Reactance | Reactance 4 Y Pix S Tr. ine Ki | X=%X_| Xo=3X A Transformer |X; | Cylindrical x; | Rotor Syn a “Michin 5 yy ¥ PV Xn VS FSalient pole| x; | %< Xt | Xo—<—X% | Rotor syn | ‘machine lL Ea tad | New Deas | Bena | Bhtnesvar | Vinyonda | Vinay | Tira | Pane | Chena ®@ 75: PU System, Symmetrical Components & Fault Analysis The negative sequence reactance is same as that of positive sequence reactance except in a salient pole synchronous machine due to non uniform distribution of flux in the air gap due to projected poles. The negative sequence reactance is the average reactance of direct axis reactance and quadrature axis reactance, y= XUtXt 2 X< XY The zero sequence components are present when the fault is a ground fault, In any ground fault, the current passes through ground, so that the effect of ground impedance (OR) reactance is to be considered in a longer length of equipments like transmission Sine and the effect of ground is ignored in a shorter length of equipments. { the ground effect is consider in transmission line, the zero sequence reactance is a variable value due to different soils and change in moisture in the ground. The value is varied as: 2.5X; to 3.5X, and an average of 3X;. In case of synchronous wiachine, the zero sequence reactance is treated as Jeakage reactance which is very less when compared to. positive sequence reactance because armature reaction effect is zero so that X,=0. Example 3.3 ‘A generator is connected to a transformer which feeds another transformer through a short feeder. ‘The zero sequence impedance values are expressed in Pu on a common base and are indicated in figure. ‘The Thevenin’s equivalent zero sequence impedance at point B is Mo=O03 X=. X= 0.05 . ‘i gt Thos Sol: Per unit zero sequence reactance diagram of the given single line diagram is shown below. 01 50.05 j00 3x025 5 H ae XX 3zn X= 0.03 Ly ‘Thevenin’s equivalent impedance, Z at ‘B? is Ze, = j0.1+ j0.05+ 50.07 40.95 =0.75 +j0.22 Fault analysis: Line to ground fault: Altemator is working at no oad, rated voltage solid neutral and solid fault. Q ool Power Systems The Sault is unbalanced with ground so that the fault is having three components in any grounded fault, the fault current should be in terms of zero sequence currents, \ fac component The fault current which is calculated symmetrical sinusoidal current, that will be expressed in terms of" rms. value. pee $ Trew) = 1) i Ia) = IdP.U) I, A or KA S=VW,I, vv, i= (¥ system) Calculation of Ini : Ve=0 VestVaztVno=0 Epi ~ Int Xteq— Juz Xz €q—$ao Xe eq= 0 iat Xieq ~ Ina X2 eq + Ino Xo €q In positive sequence current Kea Keg + Xeq 3E, 1, =31, =>——__»—__ ewe” Xi eq+ X3eq + Xoeq The fault is having 3 components so that there will be three networks and the networks are connected in series because the sequence currents are same. Sofi fault Tn =Teamlo= Ins The-rating of C.B. will be expressed in terms of braking current, This is called as fault current (or) short circuit current. The rating of circuit breaker will also expressed breaking capacity or short circuit capacity = 3ViIr Potential of neutral: Va = foXa™ Slo Xa =1XnP.U. In any grounded fault the cusrent through neutral (oi) current through ground is same as fault current only. a P-U. xVp volts (or) kV. for solid neutral. Isolated neutral XGy + a = — in _ sop, Kegs X,egr0, In case of Isolated neutral fault current will be zero and the voltages of healthy phases will be same as rated voltage. Hence neither faulty phase nor hhealthy phase not damaged. However if altemator is connected to transmission fine and the fault occurs at end of the line, the neutral of altemator is Isolated. The fault current is zero but the voltages of healthy phase become V3Vwhich is called as Raa rata Nee Daa Mero | Bhubaneswar | Visvawa | Viekapatam | Tirpa | Pane | Chennai 277: PU System, Symmetrical Components & Fault Analysis arcing ground. The insulation of phase is selected for ¥2V. So that dielectric strength of insulation failed in healthy phases. The arcing ground due to capacitance of line Ie=3lk0 Oe ie, DD See ' Ch 0 {Bv3v grounds) Hee . (Phase to In order to eliminate arcing grounds the neutral of the system to be connected to ground through a coil which is known as Peterson Coil. The type of grounding is called resonance grounding. The value of inductance of coil L Example 3.4: A 10 MVA, 13.8 KV alternator has positive, negative and zero sequence reactance of 30%, 40%, and 5% respectively. What reactance must be put in the generator neutral so that the fault current for a line to ground fault will not exceed the rated current. Sol: Fault current = Rated current Uipu=10pu 3Eg, 1,0 (Xy +X2+ Xo +3 Xi) =3 10 0340.4 +0.05 +3 X,=3 Xa~ 0.75 pu Xnqa) = 0.75 pe MVA, = 0.75| 38 14.280 10 MVA Example 3.5 A 20-MVA, 6.6-KV, 3-phase alternator is connected toa 3-phase transmission line. The per unit positive sequence, negative-sequence and zero-sequence - impedance of the alternator are j0.1, j0.1 and j0.04 respectively. The neutral of the alternator is connected to ground through an inductive reactor of j0.05 p.u. The per unit positive, negative- and zero- sequence impedances of the transmission line are j0.1, j0.1 and j0.3, respectively. All per unit values are based on the machine ratings. A solid ground fault occurs at one phase of the far end of the transmission line. The voltage of the alternator ‘ueutral with respect to ground during the fault is ee x it Sol: %=j01re Xi, jute 403 nesters jours sx 005% Xim=)02R Rag] O49P The fault current for Tine to ground fault 1 3Ep, 105 + Xaey + Xoeg Where, E,,=Va=1L0p.u 3xh0 ___ 3.37 2 -90° l= 0.2+ j0.2+ j0.49 Hyderabad | New Dei | Benak | Bhubanenar | Vnywada | Vsaepamam | Tnupal | Pane | Chama @ ACE Engineering Academy ls Xn The voltage of neutral reactance =1X =3.37 x 0.05 = 0.168 pu Since the voltage of alternator is taken as base, neutral reactance Voltage, V,= 0.168 x £620 = 642.2V 3 L-L fault: During Fault: Unbalanced condition el +k Use the symmetrical components leo =the +1y +1,]=0 In =the +t, +414] = jlo+K, -K1)] av ere i =4fh, +KLy KI] At, | =41,[«?-K] The fault is unbalanced fault without ground, there is no zero sequence components the fault is having positive, negative sequence components. Vy= Va VyotVvrtVy2 = VeotVert V2 K°VprtK Vio = KVaitK’Vio (K-K)Vai = (®-K)Via Gi) 178: Power Systems The fault is having two networks and they connected in anti parallel, Positive sequence current is Vai = Va Exi —Iet Xieq =~HroX20 Eri = Iai(Xieq+X2€q) Calculation of Ie: I= ly= hotly 1e=0+K'lgr+Kle Ip= OFK"Ig-Kla Tr= (KK) Int Ip= [(-0.5-j0.867) - (-0.5+40.867)] 732 Inu = 51.732 Ira 1, =V3Iq PU. Tei = “Tee E, © Xeq+ Xyeq ~ PU. Jn T, = V3I_, PU. Ex Woe 1a) =1;P-Ux I, A (ot) K.A. Short circuit capacity =3Vols. Potential of neutral Va=IbXa Va = 3IRo Xn=0 In case of reactance neutral (or) isolated neutral the fault current remains same. tve, -ve sequence network does not depend upon neutral grounding for L-L faults. lyderaed | New [Bengalis | Bhubanesrar | Visjavad | Visakhapatnam | Tirupat | Pune | Gheaal ALG fault: During fault: Unbalanced keh +e k= Vy=Vp=0 The fault is unbalanced with ground the fault is, having three components le=ly +l [r= Tyotlyr#y24po torte Te=Tyot fpr Knot Teo Kgs 4K ln Jp= Wrwtla(K4K) He(K+K*) Te= 2Io-IiIea T= IkotHer*Hea = 0 Tro= Tile Ys= 2atleo = 3Tko PU. Calculation of Tai: Tra + Tea + Tho = 0 - Gi) Replace Ip2 & Ino in terms of Ie Vat = Var Epi — In:X1€q = —Ip2X20q 1 En-InXeq]) OL hed Vai = Veo Ena — Int X1eq = ~InoXoeq Hyderabad | New Dell | Bengalurs | Bhubaneswar | Viayavada | Veakhapatnam | Tiwpai | Pune | Chena PU System, Symmetrical Components & Fault Analysis ‘The fault is having three networks are connected in the parallel combination negative and zero sequence network are connected in series with positive sequence network. Tay => (nate) Xeq " Xyeq + Xoeq ie= Bho PUL Ia) =1{P.U) I, Short circuit capacity Potential of neutral V, Va = 3leoXn= 1X In case of solid neutral V, = 0. |, P.U, Current Divider Rule. In case of isolated neutral current into the ground is Zero there will be current in the phase so that it will be treats as L-L fault. Taye ACE, Engineering Academy . @ Line to line to line: (or) balanced During fault: Leh=h=In Int+ly+Ip+0 Va=Vy= Ve Fault is having only positive’ set component it can be proved by symmetrical components. Ino = Hh, Hy +1,]=0 1 2 Inu 5h + Kly + K*ly|= Ty I= glle +1 +KI,]-0 1=In= In P.U. > because balance fault, SRR TI acta Nov at | ayaa | iakancovar | Vinwads | Vowiapatam | Trwpad| Pane | Cheat] Power Systems In3 - 6 short circuit fault the +ve sequence network terminal voltage is zero. Vai =0 Ear x Tes Xieq=0 Vre= Eat h=In Ia) = IdP.U)I, Short Circuit capacity: 3Vul Va = TaXn = 3]R0Xn = 0 The fault does not depends up on neutral grounding, ° Line to line to line to ground: L-L-LG=LL-L i = ita sp Fault © een XEq Short circuit capacity = 3Vblc= 3V» IrP.U. Ip =3VobUPU. =3V\, =3,20 Short circuit capacity = ree Q nwt ‘Engineering Academy 281: PU System, Symmetrical Components & Fault Analysis For synchronous generator X)90° 5 P. is reducing OP, ti 5 negative a5 Practical case: VeZ8 V.Z0 OLED, Z5-|A|Zov, 18 =V,I," val 1B) 2B PY costs a) als Aly P cod - «) |B) Special ease: short transmission line iB Be cosp p= WDE VR? WRX? X=v3R (0%) 4-8 Uncompensated Transmission Line: In short line the total value of x (reactance) is less so the altemator will deliver. maximum power without loosing synchronism. Compensated Transmission Line: In long transmission line the total value of reactance is very high so that alternator may fall out of synchronism, while delivering maximum power. To reduce the net reactance, series capacitor is placed. Such a ‘transmission line is called compensated line. Methods of improving steady state stability limit: (i) Operating the system at higher voltages ii) Reducing the net reactance of the system. This can be obtained by using parallel lines double circuit bundle conductors series capacitors mid point compensation (SAND) Hs derstad | New Debs | Bengaluru | Bhihancowar | Viayawad | VisKhapamam | Tirupai | Pane | Chea stem Stability ‘A generator with constant 1.0 p.u. terminal voltage supplies power through a step-up transformer of 0.12 p.u reactance and a double circuit line to an infinite bus bar as shown in the figure. The infinite bus voltage is maintained at 1.0 p.u. Neglecting the resistances and susceptances of the system, the steady state stability power limit of the system is 6.25p.u. If one of the double- circuit is tripped, then resulting steady state stability power limit in p.u. will be @o ®@ .@ CHO Sol: Before fault Steady state stability power EV 625 Limit = 1.0x1.0 (o12+%) 2 After fault steady stability power limit 10x10 = 0.12-+0.08 625 > X=0.08Pu ‘Transient Stability: It is the maximum amount of power that can be delivered to the load without loosing synchronism for sudden and large variations of the load due to 3- phase short circuit fault occurred for a time of 5 cycles only, Otherwise the system will loose synchronism. For the given network, the power transfer before fault is higher than that of power transfer fault Some times the power delivery becomes zero for a fault at the generator bus terminal (or) fault on only one of the transmission line near bus bar 1. Mechanical and Electrical powers of Synchronous Generator: T. oP \ »—DTt w Synchronous Generator T, ia Pe p we Te Synchronous motor Ifthe Input is more than output Pe (acceleration power) 4x W_ (W=angular velocity) w=2mn 60 N= actual speed p.m P,=Ioc W[I= Inertia kg/m? ] a= angular acceleration P,=Ma_ [M-= Moment of Inertia (or) angular momentum] M=J-sec/Mech.'rad M=MJ~see / Mech.rad x p/2 M=MI —sec / Mech.rad x p/2.x 57.11 =MJ—sec/ elec. degrees __ Rotor rotating field Stator rotating (aot Held Reference 00) = 8() + ot STN erated | New Demi | Benelare | Bhubaneswar | Viwyowada | Vihapatam | Tiupa | Pune | Chennai] Power Systems [w, = Relative angular velocity between stator V3 vZ0 and reference axis] fi E en y Py se sin 3 Xea, Swing equation which is a non-linear differential equation. It describes the relative position of rotor with respect to stator fixed as a function of time. Inertia Constant: (i) = Kinstic eneray stored Rating of themachine (w) ieee [=i] 7 ae SH Seteaael (radians) SH (de ~Fgop “eBres) Mek ‘The transient stability can be analyzed by using The swing equation, M23..p, EV sing x The solution for the above swing-equation can be obtained as follows: Swing Equation It is used to decide the stability of a synchronous machine in a Transient condition. Solution of Swing equation is required. Tt depends on number of syn machines, Single Machine System: (i Synchronous Generation is connected to an infinite bus by a loss less network. Sa eat | New Dat Benga | Bhbanesvar | Vijawada | ViaBhapamam | Teal | Pe | Chama Equal Area Criteria is used to get the solution of swing Equation, It is a Graphical study (ii) Two machine System: ‘Synchronous Generation is connected to syn ‘motor by a loss less network & £52 £81 to" + — -O# ELE s in(5, -8, Sea sins~8.) Ina two machine system, synchronous Generator is connected to syn motor, by a lossless network. Initially both the machines in a stable condition. ‘When the load on the motor increases suddenly, the electrical input cannot change simultaneously so that the motor will get oscillate, When the oscillations are very high when compared to natural frequency of oscillation the motor as become unstable, When the motor has become unstable, the load on the synchronous generator is zero and the generator will make oscillations. These oscillations are also more than natural frequency of oscillations so that the generator also becomes unstable. ‘The above two machine system has become single machine system because the two machines are stable (of) unstable at any point of time. The equal Area Criteria is applicable, (iii) Multi Machine System: More than two machines are involved. The solution of swing equation can be obtained by point by point method (or) step by step method. It is a ‘mathematical solution, Euler solution (or) RKU method is used. Note: Though the equivalent area is stated as single-machine system it is applicable for two machine system Ca w Power System Stability ‘Two machines are swinging together Two altemators are connected to common bus, any change of load will change the rotor position of two altemators is called swinging of two machines, G G M, and M; are angular momentum of two generators, 8)=5)= 5 and Pa gi = Pa; + Pay 5) yy Bo di? * dt? Ma Two machines are not swinging together When two alternators are not connected to common bus, only change of load will change the rotor position of one altemator only, then the machines are not swinging together. My and M; are angular momentum of two generators. 8= 8-5) 46O-4 a5 7, dB, eae ae ais) at? im -r)-f) ota a Swinging of the machines: en Hog Hy + Hy +... Hy Do not swinging of the machines: 1 L Hy H, In general swinging of the machines will prefer so that the equivalent inertia will increase which will reduce the swinging of the machines. Bg: Generator ~ | having inertia constant Hy & Sz Generator ~ 2 having inertia constant Hy & Sp Equivalent inertia constant HH y= * tH, Itis not correct, because H; # Ho. Choose a Base i.e., any generator having rating as Shae. To get the swinging SyoaH, Bggy = Hey = Hey + Hoegt For n machine system y BaaS.as t's Example 4.2: A 50 Hz, 4-pole, 500 MVA, 22 KV. turbo- generator is delivering rated megavolt- amperes at 0.8 power factor. Suddenly a 3 — phase fault occurs at generator terminal, which will be reduce the electrical output by 40% and assume constant power input to the shaft. The accelerating | torque of the generator in MNm at the fault will be Sol: Before fault Mechanical input (P, ) = cleetrical output (P.,) = 500 x 0.8 x 10°=400 Mw During fault electrical output = 0.6 x 400 x 10°= 240 MW Fiyderabad | New Delhi | Bengalura | Bhubanesvar | Vinyavada | Viathapatram | Tirupad | Pune | Obeanal Ce 188: Power Systems, During fault mechanical input (p,,) =400mw Accelerating power, P,= P,,~ P,, = 400 ~ 240 = 160 MW 500rpm 1500 o= = 157 rad /see 0 Accelerating Torque ye ON © Loe MNm ©, 157 Calculation of steady state stability of alternator connected to infinite Bus: va EL Infinite Bus It involves the study of dynamies of rotating body. When the rate of change of load is quite small as compared with the natural frequency of oscillation of the system, The basis is swing equation, ‘The solution to the above equation can be obtained by lineralizing the non-linear differential equation around its initial operating point ( Peo, 5, ) Mm (6) +48) = (P., +A?) K jis pure imaginary conjugate. The roots are undamped oscillations. When compared to angular momentum (M) of the synchronous generator the change of load is less. So the undamped oscillations becomes damped oscillations due to activating of damper windings. damped The system is stable Case (ii): If | ay K becomes positive. The system unstable Sve 5 8) > 90° Example 4.3: A 500 MW, 21 kV, 50 Hz, 3-phase, 2-pole synchronous generator having a rated pf = 0.9, has a moment of inertia of 27.5 x 10° kg-m’, the inertia constant (H) will be Sol: Inertia constant (H) __ Kimetic energystored in rotor (MJ) MVA rating of alternator (S) EST) ected [Noy Daw [Berga | Watancovar | Vievarada | Vinapamam | Tropa | Pune | Chena Q ric Finn Power System Stability 1 Kinetic energy stored in rotor = a Ta? 2nN, o- 60, 120. =3000:pm 2 i oS 257000. 314.15 mad /seo oKE= ; «27.5 x 10? x (314.15)* = 1357 MI n= 2357 2244 Ms/MVA 500 09 Equal area criteria: 1, Itisa graphical solution to the swing equation The basic is swing equation. Muhiply both sides the equation 2 d&/dt and integrate with respect to time. | at ls = 0 where (o=,) at For transient stability ! For the system to be stable Ai Az 3. During the swinging of the rotor the maximum swing may exceed 90°. It is only area stability, but not angle stability ES deta Nor Daas | Benesrn | abaevar | Vinyard | vEsapanam [Tiopa | Pune | Chena 4. For the stability it only consider the first swing of the rotor after disturbance, If it maintain the stability during the first stability it will also maintain the stability during the succeeding if swing. So the chances of loosing synchronism is only first swing to calculate Assumptions stability (1) The resistances, the shunt capacitance of generator and transmission lines are ignored. The shunt elements like shunt capacitor (or) shunt inductor at load Bus or generator bus are ignored. The remaining network is represented as transfer reactance. (2) The mechanical input is assumed to be constant. (3) There is no change in the speed of alternator. (4) The damping force provided by damper winding is ignored, (5) The voltages behind the reactance’s of the ‘Machines are ignored to evaluate the transient Applications of equal area eriteria: (A sudden increased mechanical input to the synchronous generator. Gi) A sudden increased mechanical output on synchronous motor ii) Fault occurs in middle of the line in a parallel ‘vansmission s/m (iv) Fault occurs, on a line near to bus bar on a parallel transmission s/m (v) Fault occurs on the bus bar connected to infinite bus of a generator connected to infinite bus by a parallel transmission line. Assumptions of equal area eriteria: (i) P.rO and « = @, ; there is no change of rotor angle ie it is called a stable point Gi) P.=0 but a+ @, (or) P, 40 but ©= w, there is a change of rotor angle, it is called swinging point. ii) Consider the moment of inertia of synchronous machine on the swinging ® web Zire A Synchronous motor NZosace >, Ref. axis deo W acceleration o>0, The excess mechanical input will be stored in the form of K.E in the rotor in an instantaneous manner, ‘© The acceleration of synchronous generator will result as the actual speed will be more than synchronous speed which will result as the rotor angle of synchronous generator willbe relatively increasing from the previous position. i.e the swinging of generator starts from ‘a’ © The angle will increase beyond ‘b’ in order to get @ Gy. If the angle will increase, output is more than mechanical input so that the deceleration will short. The area of deceleration depends on area of acceleration and acceleration. depends on mechanical input. *__ With the above assumptions it is assumed that point ‘c’ will be the another swinging point where « = ©, is obtained. + The swinging of m/c beyond “b’ will result as the output will be more than input ice, stored KE in rotor will able to convert into electrical output * Though synchronous machine is swinging, there will be relatively; change in rotor position ie while it is changing from ‘a’ to ‘c’, the swinging will start from a new point that is a! ‘* Consider the first swing of the m/c for stability analysis. It @ = @ is obtained before boundary point of Aa. Ai o 8 will increase - Ate [Acceleration | @ = Opa: ‘According to (ji)&(iii) assumptions | is. zero: no| x0, of equal area criteria the rotor angle deceleration | further increased ["B'to ‘ec deceleration |@<@na, | The machine is having dec but > | but@>o, | os so that rotor angle further increasing Ate’ deceleration | @=0, It is a swinging point. Further deceleration @ <@, the angle star decreasing, [Foto bY Deceleration [o Onin Even though synchronous generator =P, -P, sind But <«, | having acceleration, the angle will | 7 Ra 7Po, sin further decreased because speed is |__ BaGve) less than synchronous speed. ERR [en | nse | Vr | Vinten | Trp | Pe | Coma] Power Systems Ps Bs © The purpose of equal area criteria is to obtain angle made by rotor at the time of fault cleared by CB which is known as critical clearing angle by considering critically stable condition ie Ay = Ad 2. Sudden increased mechanical output on synchronous motor — oO Pet Pa Xy Pe fimmrP" Pe Py = EY sind, = pa, sity eq Pa increased electrical input p., = 2Y-sind, =p,, sind, 26 Pay =Pmy > Xoeq = Xteq P,, =initial mechanical output p,,= increased mechanical output Acceleration Electrical input stable oF Onax, © The equal area criteria synchronous motor is an image of equal area criteria of synchronous ‘generator =Pe~Ps=—(Ps —Pe) motor = opposite of generator © The synchronous motor is initially supplied with electrical i/p of pz and the corresponding mechanical output of pj, it is represented by point ‘a’ on the power angel curve. SSR TIN rata | New at | Dent] baer | Vora | Winkpatam [Tia [Poe | Chena — 193: Power System Stability Disturbance: | le will be slowly increases so The load om synchronous motor is increased that the swigging of mie starts from ‘a suddenly from pu to pa where as there is ne ‘The angle will increase beyond ‘b’ in order to corresponding change in the electric input , get o=@,. As the angle increases, because the inertia of motor does not permit a po ; sudden change intherotor angle, (i) The acceleration will start i 7 (ii) he area covered beyond ‘b’ does depends ‘on the area already covered from ‘a’ to “b’ Ps, With the above assumptions the swinging of = ~ve = decelaration machine is mode up to ¢ to get « = os If the actual speed is not same as syn speed, After completing all the oscillations the there will be a relative speed which will result synchronous machine will be stable at ‘b” so a that © =<, _ -Positio ‘Equation Behavior Speed Remarks. {a _ _ Ata Neither ace [@= 0, ‘there is no change in rotor angle. Itis nor dec astable point “ato 8 | p=p, -p,, dee @nn | Even though itis an ace but rotor ee buta8) Ate | p=p,-p, acc 0-0 It is a swinging point for further sep acceleration > «as . The angle start decreasing the rotor 7 _ will swinging back. 7 ‘tod | p=p, -p,, ace o> 0 8 will decrease | Ath Accelerati | @= Omar inging point. The angle will oniszero | 4@, | decrease due to effect of moment of and no dee inertia, dec ©e, | @>a, so that the rotor, angle will | further decreased. Hyderabad | New Deli | Benglara | Bhuk nam | Tirupati | Pane | Checsai | Power Systems * The purpose of equal area criteria is to evaluate the angle made by the rotor at the line of fault “cleared by CB which is called “as critical clearing angle by considering critically stable condition ie., Ai = A> (SSSR $e Dak Deira [cna Woah | Vetapann | Tal Re [Chana] 3. Fault occurs at middle of transmission line in a parallel transmission network:~ vZo 0 es Oar] » FG-$) p= mechanical input 1m; sin 8) —> before fault asin 6—> during fault pes = pm sin 8 — after fault pm) < pms < pmy X20q> Xseq >Xieq OO, OF Ope OF O, ‘The synchronous generator is supplied initially with a mechanical input of p, and it is represented by point a on power angle curve. Disturbance: © A 3-@ short circuit fault has been taken place in a parallel line at the middle of line suddenly, so that the electrical output is reduced where as the mechanical remain same. It is represents by ‘b” P=Ps~Pa= Ps~Pna sin 5, = Hye =ace (w>w,) ACK So that the rotor angle will be starts increasing slowly ie, the swinging of m/c takes place from rr © The fault can exist for few cycles, before the fault is cleared by circuit breaker so that the swinging of m/e will be continued till the fault is cleared. * Iris assumed that fault is cleared by breaker at ‘c’ and the corresponding angle made by rotor is called as critically clearing angle 5, Engineering Academy 295: Power System Stability * The faulty line is isolated ty breaker so that there will be a sudden change in electric out put due (0 change in the reactance, the mechanical input is same and itis indicated by point ‘d’ due to moment of incletra of rolor the angle will increase future in order to get @ = @ in @ critically stable manner that is point ‘e” on pes curve. Calculation of 5.: fete [p.d6 =0 : Sma fds + [p.d6 =0 f(P.—Pq.sind\46+ [(p,~p,.sind)45=0 [P.5 + Pyy cos6 |S +[p,5 + ps cos6 f."" =0 (P8.— PA, + Pea 0086, ~ Ps C085.]+ Pn ~ PB. Put C88 yu ~ Pas °085, = 0 Ps(Smax— So) + Pid COS Bmax ~ Prod COS &5= Pas COS Se ~ Pm2COS 5. cos Atta’ P= pei = Pmisin 6, Bocca = Atte’ Ps= Pes = Pra $i Bras Snax = sin" (24) <90° + Ina critical stable condition the max swinging angle will be more than 90° «3 Pms Sin (180 - max) =sin(180-3,,.) Ea ta [New Daa Haars | Batancvar | Vinyorads | Viskhaparom [Tropa | Run | Cheat Ps x ~ 50) + Pgs 2085 rae ~ 1 Pua ~ Poa cos, lelecdeg 180-sin“| (2-rcue ma J = ban xt Bmax (rad) = Bas XT + For any given system there will be a critical clearing angle which is calculated with the help of equal atea criteria under the assumption that the s/m is critically stable. How ever, in order to ‘maintain stability the actual angle made by breaker should be always ‘ess than critical clearing angle. Other wise the s/m will loss the stability. He Power Systems * If the are will be quenched at a faster rate, the area covered will less before fault is cleared so that the actual angle will be less than critical clearing angle hence the s/m will maintain stability. ie Peo dt = A, ace > oo, Due to the acceleration of alt the rotor angle will increase slowly so that the swinging of the m/c will start from °b” & it will be continued till the fault is cleared by breaker after few cycles and corresponding angle is called critical clearing angle 3.. Even though it is assumed as bus fault but the faulty line will isolated by breaker so that the electrical output is increased suddenly due to change in the reactance, where as the mechanical input will remains same and it is represented by point “d°. Due to movement of inertia the rotor angle is further increased even fault is cleared in order to got © = @, in a critical stable manner i.e, point fe Calculation of + ‘festo+ p.a= 0 * ke = oo. -0)8s 0, =Pas sin 6)d5=0 3 =r [an =A Paton Li es Pas Conversion of 6, into t.: Pa=P. Pa =O] t > b= Fe +A ~-constant 0 = the synchronous machine is at initial condition 8.=5)=0tA > A=3, ND) 1 si Noa e| ntnerar| Vida | Weapanam | Tiga | Ps | Goa —i 297: Power System Stability MG, —5,) Calculation of 5.: Semon rs P. fo.d5+ [p,ab ' wo & 1, VM; to ; fom vp. fie.-045+ f(p,-p,osindo5=0 5. Fault occurs on bus bar: ° = : 4 va & oe Sa 5, +Pas 0086 H Pes Y Pna = Put Example 44: A generator delivers power of 10p.u. fo an - . infinite bus through a purely reactive network. P.= Mechanical input The maximum power that could be delivered by Pet = Pisin 89 the generator is 2.0p.u. A three-phase fault Pea= 0 5 Pao =O ‘occurs at the terminals of the generator which Po = passin eduees the generator output to zero. The fault is Pras= Pmt cleared after (, second, The original network is then restored. The maximum swing of the rotor Pe angle is found to be Snax = 110 electrical degree. i Pa and Pes Ten the rotor angle in electrical degrees at t= Sol: If a fault occur on bus bar so, that electrical output will become zev0, where as. the mechanical input will remains same ait will bbe represented by point ‘b’ on 8 axis Hence p= P.— pe = ps -0 =py= tre = ace => © >0, i.e, rotor angle start increasing. ‘© When the fault is cleared by breaker, the alt is isolated from parallel lines, and it will be closed as early as possible, so that the entire system will be restored. When the total system is restored there will be a sudden increase in the ‘output due to change in the reactance of the net work, where as the mechanical input will be remains same, it is represented by point ‘d’, ms Infinite bus Before fault Px = 2.0 P.u # P, B= sin (2). 30°= 0.523 rad During fault P,, =0= P,, = MOoxn wax = 110°= 919 rad. i 180 Pa = Pa { Where P,, =Pua] Critical clearing angle, 8, =c08" [= 50) + Pra, 08 Bar ~Pry =) 6, =c0s"] [sae 91 (EA ed | Nee Dats | Benga | Bhabanewvar | Vaan | Vinpamam | Tropa | Pa | Chama ACE ‘Engineering Academy 198: Power Systems 8 = 69.14" Factors Effecting Transient stabi To maintain the transient stability the area covered during acceleration (A2) in case of ‘generator and nice versa in case of motor. (@ Higher value of “M? Fora given accelerating power, the higher value of ‘M’ will result in slower rate of change of | rotor angle. So Aj Power Systems Case (ii): If the added element from any one of the bus does not create a new bus to the existing bus system, itis treated as link and the size of Zu Will remain same. All the elements of original Zxy, are to be modified to consider the effect of the impedance of added ‘ink element. 3 link Power if Sym 2 Noor i 3 L__] Procedure for modification of Zs, Connect a voltage source in series with the added element having the magnitude of 1.0P.U.So that a temporary bus is created, which is called as link bus. The size of Zpus is increased temporarily by ‘one. The impedances of the temporary are to be calculated x@) 10 Power ‘System Link bus) Network ‘(Link bus) 20) Tt ° Zy Zy 2Zy Ly Zy Ly Ly Ly Zee Zy Ly Zy Zy Zy Zn Zy 2a J Mutual impedance: Z,=Zy-Zy Self Impedance: Zy=Ly-Zy t2p %1 is impedance of added link element. The above equations are correct if bus ‘p’ is not a reference bus. In case bus ‘p’ is a reference bus ‘Mutual impedance: zy ge Zi =O Seif impedance: Zy=-Zy+Z, at Zh ‘The original size 2, can be obtained by eliminating the link bus for which the voltage source is short circuited. [12s] Zev nat = Zou is Case3: Removal of transmission line between any two buses. If the transmission line is removed between any two buses, the size of Zu, can remain same i.e. the lines which are permitted to remove are link elements. When the link element is removed, the entire Zeus is to be modified. The procedure for modification of Zu, is same as the procedure for addition of link element. Mutual Impedance: Ly = Zu —Zay Self Impedance: Zy =Zy 7 Zqu 2 i Zi the impedance of removed link element, Iz.) 4 Zeus mot * Zoe — Example 5.3: The Zann of a system is oy, TOL 04 OF Zou {01 0.2°-0.1!pu LOL 010.3) 5 If 4 S-phase fault occurs at Bus-2, + euirent in each phase is) Sol: In =e Xleq ate PSSST ND aad Ne Dati | enatra | Bhbaesvar | Viawada| Viakapamam | Tiupa | Pune | Chea ACE Engineering Academy 2105: Load flow Studies Exaniple 5.4: The positive sequence bus impedance matrix of a 4- Bus power system network is given helow: If-a symmetricai fault occurs on Bus-2, then positive sequence component of fault eurrent Is OTA j0.620 30.656. 0.644)."} sot: 1, rg. [10620 50738 90.642 0.660 Be" | §0.656.j0,642. j0.0702 - j0.676), j0.664 * j0.660 j0.676.. j0,719 —Yaus E iE Zaus () Memory requirement to store the elementis ess | (@) More memory requirement to store two | z elements (ii) Itis useful for load flow studies (i) Kis useful short circuit studies (jii)Shunt element are included Gauss-seidal method [Gii)Shunt elements are ignored Newton Raphson method (i) It gives the solution of non linear simultaneous equation in rectangular Coordinates. @ Tt gives the solution of non linear simultaneous equations in rectangular (OR) | polar form. (i) Convergent criteria is a quadratic Gi) Convergent criteria is a quadratic | AP) = Ppspec ~ Ppcai(OR) AQp = Qraps ~ Qpeat | Gi) Tt is easy in programming load flaw equations so that the memory requirement is less to store the data It is difficult in programming load flow equations so that the memory requirement is high, (iv) The time taken for each iteration is less (¥) No of iteration required: are more to get the convergent criteria and the no of iterations does depends on the size of power system. Gv) Time taken for éach iteration is high (v) No of iterations required are less to get the | convergent criteria and no of iteration are not depends on size of power system (Wi) Total time taken to get the convergent criteria is high | (vi) Total time taken to get the convergent eriteria is less | (vii)Selection slack will affect the convergent criteria (viii)Un reliable convergent (Wii)Selection slack Bus will not affect the convergent criteria (viii)Reliable convergent | (ix) Applicable for small power systems (ix) Applicable to larger power systems RE deat | Nov a | Benet | Bhabaneovar | Viervada | Vinkbapaiara | Trop | Pane | Cher Economics of Power Generation Load Curve: It is plot of load in kilowatts versus ime (usually for a day or a year.) in the order in which they cccur, ie, chronologically. The consumer consumes the electrical energy when he needs. This means his load on the system will keep on changing with time. The curve drawn on a daily basis is called daily load curve or the curve dawn on a annual basis is called Annual load curve. The average load on the supply system will be less than the maximum demand. Characteristic 1, The toad can be determined at any particular hour of the day. 2. The maximum and minimum values of the daily load can be found out from the load curve. 3. The area under the daily load curve gives the energy supplied in a day (month or year if considered monthly or yearly load curves). 4, The daily average load can be found out by caleulating the area under the load curve (i.e. energy in kWh) and dividing it by the number of hours (ie. 24 hours), 5. Information whether the power system is working efficiently or not, Load Duration Curve: It is the plot of load in kilowatts versus time duration for which it occurs in the descending order of magnitude, irrespective of the time of occurrence Load Factor: The ratio of average load to the maximum demand during a given period is known as load factor. Load factor = —Averazeload _ Maximum demand Average Load: The average of loads occurring on the power station in a given period is known as average load or average demand. (ESSER 0a aT a | Viva | inka | Taj | Re Cha] ee ee bat angio « NOt en (AWa) sete dey ome units generated in month Ne Monthly avg toad = —~ No.of hours ina month No.of units generated ina year Yearly avg load = 8760 hours Maximum Demand: \t is the greatest demand of load on power station during a given period. Demand Factor: It is the ratio of maximum, demand on the power station to its connected toad, Demand factor = Meximum Demand ‘Connected Load Connected Load: It is the sum of continuous ratings of all the equipments connected to the supply system. Diversity Factor: The retio of the sum of individual maximum demands to the simultaneous ‘maximum demand on the power station is knawn as diversity factor. Diversity factor Sum of indi Simultaneous Maximum demand on power station al_max imum demands ersity factor is higher, the energy is cheaper. + Load factor, diversity factor higher the energy per unit is less. + Diversity factor has direct effect on the fixed cost of unit generated. Coincidence Factor: It is the reciprocal of diversity factor and is always less than ‘1” Plant capacity factor: It is defined as the ratio of average demané on the station to the maximum installed capacity. ACE Engincering Academy 107: Economics of Power Generation Capaity factor= Actual energy produced Maximum energy that could have been produced Plant capacity factor will be the ratio of the total energy generated over the year, to the plant capacity (including reserve capacity) multiplied by 8760 hours. Plant capacity ~ Maximum Operation Factor (Service Factor): It is given by the ratio of number of hours the plant is in service to the total number of hours: in a given period (usually a year) Servicehours Operation factor Total duration Plant Use Factor: It is the ratio of kWh generated to the product of plant capacity and the number of hours for which the plant was in operation. Station oxtput in kWh Plant capacity x Hours of use Plant Utilization Factor: It is defined as the ratio Plant Use Factor of maximum demand to the plant capacity. Plant Utilization Factor Firm Power: itis the power intended to be always available. Cold Reserve: Itis that reserve generating capacity which is available for service but is not in operation. Hot Reserve: It is that reserve generating capacity which is in operation but is not in service. Spinning Reserve: It is that generating capacity which is connected to bus and is ready to take load. Base Load and Peak Load on Power Station: Base Load: The unvarying load which occurs almost the whole day on the station is known as ‘base load, Peak Load: The various peak demands of load over and above the base load of the station is known as peak load, Nuclear power stations are used as base load stations operating at high load factors of over 80%, These meet the ‘block loads’ which are at the at the bottom of the load curves. Hydro ~ electric stations with ample storage of water are also used for base toad operation, Similarly sun-off river plants during river flow periods are also used to meet the base load requirements. Hydro-cleettic stations with limited storage of water are used to meet the peak loads of a system. Steam — stations are capable of operation both as peak load and base load stations depending upon the co-ordination required with other type of stations in the system. These can operate at load factor’s varying from 40 to 80%. Example 6.1 A. generating station has a connected foad of 43MW and 2 maximum demand of 20MW the units generated being 615 x 10° per annum, Calculate (O the demand factor and (ii) load factor. Maximumdemand Connected load 20 Sol: (i) Demand factor = 0.465 B Average demand Units generated / annum ‘Hour ina year 61,5x10° 7020kW 8760 Average demand ii) Load factor a ee Maximum demand 351=35.1% ACE Engineering Academy 2108: Power Systems Example 6.2 ‘The maximum demand of a hydro station is 200 ‘MW, the annual load factor being 60%. Calculate tthe total electrical energy generated per year. Sol: M.D =200 MW = 200 x 10° kW LF=06 Annual load factor = noof units generatedina year MDX noof hrs.ina year -. No of units generated / year = 0.6 x200 x10° x 8760 1.2 x 876 x 10° 1051.2 x 10° kWh Example 6.3 ‘A generating station supplies the following loads to various consum Industrial consumer = 1500 kW 5 Commercial establishment = 750 kW Domestic power = 100 KW ; Domestic light = 450 kW. If the maximum demand on the station is 2500kW and the number of kWh generated per year 45x 10°, Determine (@) the diversity factor and i) annual load factor, 1500%750+100+450 Sol: (i) Diversity factor mae 2500 12 kWh generated /annum hoursina year _ 45x10* 8760 Averageload Max.demand Average demand. = = 513.7 kW (i) Load factor = .205= 20.5% [XCAR deatad | Nov Dai] Bengdare | Bhubaneswar | Viayavads | Viathapamam | Tirupati | Pune | Cheanal Example 6.4 A Generating station has a maximum demand of 15000kW. The annual load factor is 50% and capacity factor is 40%. Determine the reserve capacity of the plant. Sol: Energy generated = Maximum demand xL.F xHours ina year 15000)»(0.5)«(8760)kWh =65.7 x 10°kWh Capacity factor __ Units generated annum Plant capacity x Hoursina year x10¢ 0.4x8760 «+ Plant capacity = 18750kW Reserve capacity = Plant capacity — ‘maximum demand 18750 - 15000 750 kW | 1. Economic Load Dispatch The economic load dispatch involves the solution of two different problems. These are unit commitment problem and economic dispatch problem. Unit commitment problem: Select optimally out of the available generating sources to meet the expected load and provide a specified margin of operating reserve over a specified period of time. Economic dispatch problem: ft is required to distribute the load among the generating units actually parallel with the system in such manner as to minimize the total fuel cost minute-to-minute requirements of the system. The economic load dispatch problem applicable for fuel based units rather than hydro electric stations. @ i SF cadens Fuel cost of a thermal power plant + Itgives the relation between fuel cost and power ‘output of the plant + Itis an empirical formulae + The fuel cost of an i plant is expressed as 1 CAR) =saP? +R +6 Rs/ hr Where, a, bj, & ¢; are loss coefficients of i* unit P, = power output of i* plant in Rs/hr Fuel cost curve: + It is a curve between fuel cost in Rs/hr versus power output unit in “MW? + Pinig is fixed by boiler and thermodynamic + Pinas is fixed by thermal limit of the generator Incremental fuel cost of the unit The incremental fuel cost is obtained by determining the increased cost of fuel for a definite time interval during which power output is increased by a small amount Mathematically, 1 = ays Where, IC; is incremental cost of i* unit IC, =4,P +b, Rs) MWhr 2109: Economics of Power Generation Incremental cost curve AC{Rs/MWhr) Pontw) 6.4.4. Economic Dispatch neglecting losses: Objective: To minimize generating units, the total fuel cost of thermal ‘Let us assume that total number of units are The total fuel cost is given by Gaycir) ‘Such that p, (Equality constraint) And Pimin $ Pi S Pimax ‘nequality constraint) Using Lagrangian, method, the augmented cost function F cab be written as r=c,+i(Sa-n) \ Where, is a Lagrangian multiplier Minimization can be achieved by the condition or aR, By solving the above equation we get, ai a Where, Nv ic, 2, PE sa | New Dis | Bens | Dhabanesva | Vinyavad | Vietlapatam [Tip | Fane | Chena @ rath ‘Engineering Academy 2110: Power Systems £.~i¢; is the incremental cost of i® generator. P, 12020 -333.33MW Pp =P, + Pz = 225 + 333.33 = 558.33 MW | Beample 6.7: Conclusion: To get the minimum fuel cost solution, the incremental fuel cost of all the units must be same. Example 6.5: ‘The cost function of 50 MW generator is given by F(P) = 0.02 P? + $3 Pi + 225 Rs/hr When 100% load is applied, Calculate ineremental fuel cost and fuel cost. Sol: F(P,)=0.02P/ +53P, +225 aE 0.04P, +53 a, = 0,04(50) + 53 = 55 Rs/MWhr Fuel cost = 0.02(50)"+53(50)+225 = 2925 Rs/hr Example 6.6: The fuel cost of 2 units are as follows. FI=02P) +30P, +60 | a F2=0.15P? +20P, +80 Calculate the generation schedule of the each unit for a Lagrangian multiplier of Rs.120 per MWhr Sol: F,=0.2P? +30P, +60 F, =0.15P; +20P, +80 2.= 120 RoMWhr di =04P, +30 aP, ar, dP, =0.3P, +20 From co-ordination equation: IC; = IC) =% IC\=2=9 04P, +30=120 p, 120-30 _ 900 04 4 IC, = 2. => 0.3P, +20=120 225MW. The incremental cost characteristics of the two units in a plant are given by Ty=01P+80 — ReMWhe 12=0.15P,+3.0— Rs/MWhr The optimum sharing of load when the total load is 100 MWis Sol: [ei = 0.1 P; + 8.0 RVMWhr Jez = 0.15 P) +3.0 RS/MWhr Pp= 100 MW From co ordination equation: IC = IC2 0.1 P;- 0.15 Pp =-5 1 From equality constraints: Pp =P, =Pr+ Py | Prt P= 100 - Q | From eq (1) and eq (2) | P,=40MW, — P, =60MW Example 68: The Incremental fuel cost of three plants are given as To)= 0.12 P, + 16 Rs/MWhr, To. 0.2 P +24 RMWhr, To= 0.15 P+ 18 Rs/MWhr Determine the generation schedule of the 3 plants for the cost received for each plant is Rs. 30/MWhr. Sol: From co ordination equation: IC) =1C,= IC; =A ICy=2=> 0.12P, +16=30 P, =116.66 MW IC2=h=> 0.2P, +24=30 P,=30 MW ICp=2 => 0.15P, +18=30 ?,=80 MW CGRP) dea | New Dati | Benga | Bhubaneswar | Viyanada | Véeiopamam | Tropa | Pune | Chennat ® Engineering ACK cademy Example 6.9 Incremental fuel costs in Rs/MWhr for a plant consisting of two units are Assuming maximum and minimum loads on each unit to be 125 MW and 20 MW respectively, the optimal lead sharing by G; while delivering 50 MW load is Sol: Given IC, =0,2 Pi, +40 IC,=0.3 P2=30 20 Pi <20MW So P, = 20 MW ; P.= 30 MW Example 6.10: ‘The Fuel inputs of plant 1 & 2 are given as F, =0.2 Py’ +40P, +120 Rs/hr F, = 0.25 P;’ + 30P, + 150 Rs/hr ‘The minimum & max load on each plant is 25 P,< 100 Where i= 1,2. Determine the economical operating schedule and corresponding cost of generation for a total demand of 180 MW. IC = 0.4 P; +40 R/MWhr IC2=0.5 P2 +30 R/MWhr 25SP) & Pps 120MW 5 Pp= 180 MW JC, =IC, = 0.4 P; -0.5 P= 10~ P, +P. = 180 Solving i & ii P)= 1111 MW Engineering lead Economics of Power Generation P)= 68.89 MW Fr=F) (111-11) + F2 (68.89) = 0.2 (I111N)2 + 40 (111.11) +120 + 0.25 (68.89)° + 30 (68.89) + 150 = 10,436.64 Ryhr 6.1.2. Economic Dispateh with power loss: Objective: To minimize generating units. the total fuel cost of thermal Let us assume that total number of units are 4 The total fuel cost is given by Saw Such that, 2 Where, Py. is the power loss in ‘MW’ and it is expressed as ’» + P, (Equality constraint) A= Lear MW For N = 2, the power loss formulae expressed as PAB R + BaP 2B RE MI And Pisin S Pi S Pinax (Inequality constraint) Using Lagrangian method, the augmented cost function F can be written as ) F=6, o(Sa-n-n] aa Where, 4. is a Lagrangian multiplier Minimization can be achieved by the condition Fog oF, By solving the above equation, Powe Case 2: If load is located at plant -2 oF, G & 2. is the incremental transmission loss the i unit © re ar and it is expressed as, Py Ri 0;R; By x (Ry +R:)=> Bu ¥0 Byz oc (Ret Rs) => Bro =0 (Biz=Bz) a Rs => Bn =0 6.1.3 Special eases Case : 1 if the load is located at plant - 1 G Po Ri=0 Rr#0Rs=0 By oc (Rr +R3)=> Bu =0 Ba x (Ro +Rs) > Ba #0 (B= Bx) aRs => B= 0 PL=By PP + By Pp +2.By Py P2 MW Example 6.11: PL=By Py ‘The penalty factor of the system shown below is sen Py=4s MW Penalty Factor, Fld 1-ITL 1-01 SS eT rand | New Dei | engtorn]| Bhabaeswar | Vinywad | Viakbapaaen | Tropa | Pane | Obes Ce 2113: Economics of Power Generation Example 6.12: ‘A two bus power system is shown below. The incremental fuel cost of the two plants are OF ne Te; = 0.35 P; +41 R/MWhr Te. =0.35 P2 +41 R, MWh ‘The loss expression is P= 0,001 (P;~ 70° MW ‘The total incremental cost of the system is 117.6 Rsy/MWhr (@ Calculate the: optimal scheduling of generators (Gi) Calculate the total power loss. Sol: (i) From co-ordination equation A=LIQ=L IG - P, = 0.001 (P= 70" MW (0.35 P, +41) = 117.6 P, = 218.85 MW LAG=% te _ ((0.35P, +41)=117.6 114=0.002P, P, = 159,029 MW (i) PL=0.001(P2-70)" (001(159.029-70)" 92 MW 1, Py, = 369.966 MW ESRI ser We Dati | Benes | bance | Visyorach | ViaRapatue | Tropa | Pane | Chennai Circuit Breakers 7.1 Introduction ‘The protective switch gear is designed based on fault Analysis * Circuit breaker is designed based on most severe fault at its location. (3 - § Fault). © If there is a fault on any part of the power system, the magnitude of fault current is very high. (Approximately 2 to 10 times the full load current). The fault current is not just limited to faulty section of the system but even extended to each and every healthy sections of the system. © The relay is a device which senses the abnormal operation of power system and issue signal to corresponding circuit breaker and cicuit breaker disconnect faulty section from healthy system. 7.2 Types of protective switches used in the system i). Fuse ii). Load interrupter iii). Line isolator iv). Circuit breaker @ Fuse Operates only under abnormal conditions Whenever heat energy developed by the fuse is more than its thermal limit, then the fuse will melt down or blown out, fault element can be disconnected, * Fuse is a primary protective device in the distribution system * In substation, fuse is a backup protective device. * The fusing current, Je d%, diameter of the wire. * The fuse element is generally made up ‘material having the following characteristic. (@) Low melting point Eg: tin, lead, zine is the (b) Free from deterioration due to oxidation, (c) low cost Gi) Load interrupter * This will be operated only at full load conditions. * Fuse is operates at abnormal conditions (itself), © Load interrupter is used for energy management, load shedding/curtailment ‘* Breaking capacity of load interrupter = full load capacity. [> 100 MW (feeder-1) [> 250 MW (feeder-2) ©—— 150 Mw (feeder) Rating 50 MW (feeder-4 sow t= (feeder-4) “> 50 MW (feeder-5) = Total load = 600 MW The above diagram represents over load on the system e100 MW. The load interrupter operates and disconnects feeder-4 and feeder-5 (ii) Line Isolator © It will be operated only under no-load condition, * The necessity of Line Isolator is to provide ‘more clearance. * Sequence of operation is: First load interrupter has to operate and next Line Isolater. * Breaking capacity of Line Isolator is zero cp Bator Ge Mecof space Hyderabad | New Dali | Bexgalura | Bhulanesvar | Vipyavada | Viakapamam | Tirupati | Paoe | Gbeanai eee Circuit Breaker (iv) Circuit breaker # It will be operates under all conditions i.e., at no-load or at fulf load or at abnormal condition, Fuse and load interrupter can be replaced by cireuit breaker. * Breaking capacity of C.B is three phase short circuit MVA at breaker location. * CB is a protective switch and which wil make the circuit and break the circuit during normal and abnormal conditions respectively. 7.3 Purpose of cireuit breakers ‘The main purpose of circuit breakers is to: * Switch load currents © Make on to a fault © Break normal and fault current © Cary fault current with out damage till the interruption. The important characteristics from protection point of view are: * It should open main contact as quickly as possible once it receives trip signal from relay © The capacity of circuit that-the main contact is capable of interrupting The first characteristics referred to as. the tripping time of circuit breaker (CB open time), which is expressed in cycles. The opening time of modem high-speed cireuit breaker is 4 cycle to 8 cycles. Opening time: It is the time between the instant of application of tripping power to the instant of separation of main contacts, Areing time: It is the time between the instant of separation of main CB contacts to the instant of Arc extinction, CB operating time = opening time + Arcing time | 7.4 Are phenomenon Circuit breaker 7.4.1 ARC Initiation AC t= 0: There is No trip signal to C.B From relaying system. ref | \uc WY Att= 0: There isa trip signal to C.B from Relaying system & CB contacts are started separating. FC f | \ MC fu, Att=0° CB contacts little separated. FC Mc Ie a Ave Ex: 132 kV CB Ax- Distance between fixed contact and moving contact (which is very less Ax = 0.1 wm) AV- Voltage across contact spacing (which is very less compared to operating voltage AV = 100 V) Electric field intensity between the contacts, AF p= tO _ to00ev fem O.1nm “Bis very high and it is definitely greater than the breakdown strength of the medium between contacts. So gap breakdowns and are is initiated. E= ‘ar Conclusion: arc is initiated at the instant of contact separation due to high field gradient (or) field ionization, (EGRET stad New Dai | Berwatrw | Bhubaneswar | Vivavads | Veakapatnam | Tropa | Pune | Channa ® rnsab Beste 7.4.2 ARC Maintenance EX: | ; cycle C-B; Open time = 30 ms. Att = 30 ms: Contacts are fully separated. FC MC Vince X ~ Contacts Spacing (itis in c.m) =1om Vac~Are voltage (it is 2 to 5% of system voltage) Vex _0.03X132KV 1 x iB Jem = 2.286 kV/em. So field gradient is very’ less compare to the dielectric strength of the medium between the contacts. Conclusion: * The Arc is maintained not because. of field grailient * It is maintained because of thermal ionization due to high amount of heat produced during Arcing process. 7.4.3 Properties of the Arc: Arc is column of ionized gasses. Conductance of their Arc Number of free e generated during ionization. e< Cross section areas of the Arc et lengthoftheAre * The path of the Arc is purely resistive and it is negative temperature coefficient Temperature increases=Rye decreases [EGS ata |New Dat engine | Banewvar | Viavads | Viskapatan | Tropa | Poe | Cheat] 2116: Power Systems = Conductance of the Arc increases. * The resistance of the Arc is highly non-linear. 7.5 ARC interruption ‘There are two methods of arc interruption: (i) High Resistance method ii) Current zero interruption method. 7.5.1 High Resistance Method In this method, ARC resistance is increased so as to reduce the current to a value which is insufficient to ‘maintain the Arc. Heat energy = I Rare t — Yaw Re 2 Heat energy = —“= xt 1 Heat energy & <— p= Resistivity of the Arc I= Length of the Are A= Cross section area of the Arc The Arc resistance can be increased. (Lengthening the Are Gi) Constraining the Are Git)" Cooling the Are (iv) Reducing cross section area of the Arc. © When current is interrupted the energy associated with its magnetic field appears in the form of electro static energy. So a high voltage appears across the breaker contacts. If the voltage across the breaker contact is very high and it is greater than the with stand capacity of the gap then gap will breakdown and Are will restrike. * Therefore this method of Are interruption is not suitable for high current interruption. * This method is employed for how power A.C & D.C circuit breakers. M7: 7.8.2 Current zero interruption * This method is applicable only in case of A.C circuit breakers + For a SOHz system, the current wave passes through natural ze10 100 times per second. This feature of Arc is utilized for A.C interruption. * In this method the current is not allowed to rise again after zero current occurs. © The current is not interrupted other than zero current instant, a high transient voltage appearing across the breaker contacts which restrike the Are again. ‘There are two theories which are explaining current zero interruption method (i) Recovery rate theory. (Slepain’s theory) Gi) Energy balance theory. (Cassies theory). Recovery rate theory (Voltage race theory) * Arcis a column of ionized gasses. ¢ The ionization is minimum at current zero instant. So to extinguish the arc, ions and electrons removed from the gap immediately after the current reaches to a natural zero, © Tons and electrons can be removed either by recombining them into neutral molecule or by sweeping them by inserting an_ insulating medium between the contacts. * The Arc is interrupted if rate of removal of ions from the gap is greater than that of rate of ionization. * Rate of ionization is depends on restr voltage. * So in this method, the rate at which dielectric strength recovered is compared with rate of rise of restriking voltage, ing, © If dielectric strength increases more rapidly than restriking voltage the arc is extinguished. © If restriking voltage raises more rapidly than the dielectric strength, the ionization persist and breakdown of the gap occurs, resulting in an are for another half cycle, Circuit Breakers Voltage Restiking Volz Dielectric voltage Strength Dielectric Restikin Strength \ a voltage Time ‘Are not extinguished Tine ‘Are extinguished Fig. 7.1: Recovery rate theory Energy balance theory © The C.B contacts are about to open its resistance is zero and heat generation is zero. * When contacts are fully opened and are is extinguished resistance is infinity and heat generation is again zero. * Between these two instant heat generation goes to amaximum value. Heat generation R=O Rem Fig.7.2 Energy batance theory + The circuit breaker is designed to remove this generated heat as early as possible by cooling the gap, giving a blast of air or flow of oil at high velocity and pressure. ‘© If the rate of heat generation is less than the rate of heat dissipation, the arc is extinguished. * If the rate of heat generation is more than the rate of heat dissipation, The space break down and resulting in an arc for another half cycle. 7.6 Terms regarding Arc phenomena (i Are voltage (Vare) Voltage across circuit breaker during arcing period is known as are voltage [EN Dats enc [Wher | iby | Voalapaam | oat | Poe | Chena 118 ve vat ee RY Mc The © Are will be modeled as a non linear resistor, where Rye is variable according to temperature. Vare = TarcRare As Jue increases, temperature imereases and Rare decrease, So Vare ~ constant # Vaz Tag are in phase © Are power factor is unity but fault power factor is almost zero lagging. Recovery voltage Axe Phenomenon Fig. 73: (ii) Active Recovery Voltage The instantaneous voltage at the instant of Arc extinction is called active recovery voltage. Active recovery voltage depends on following factors (The power factor of the fault current. (ii) The Armature reactions _ synchronous machine (ii)The circuit conditions [ype of fault, System grounding] © If operating voltage of the system is132 kV and all factors are neglected then Vow = Maximum value of phase voltage of system (Vin) * To account for power factor of the fault “Vmn’ should multiplied by a factor sing Where ‘$’ is the power factor angle of fault current. © To account for effect of armature reaction ‘Vn’ should be multiplied by a factor K, Kish. © To account for circuit conditions *V,’ should be multiplied by a factor ‘K2” Kz is called first pole clearing factor Ky = _Vitageactoss first pole which iscloared the fault VoltegeacrossC Batterallpolesarecleared the fault Type of | Rand ve | uw | ue | un | ute rowing Grounded . = et | Kels = womided sat | eis | ket | Kel | Kind ‘Ungrounded orl lete ee ones Keets [kets | kes | emus Varv= Ki KV q sing Note: To calculate line value of active recovery voltage the above expression should be multiplied by a factor V3 Ex: Fora 132 KV system Van BAY 3 v3 RAN cates | Now Det | Benzaura | Bhubaneswar | Vinyavare | ViaKiapataim | Tirupal | Pane | Chena 19: Circuit Breakers ‘The transient voltage across breaker contacts at the instant of arc extinction is known as restriking voltage. ‘ault on terminals of a circuit breaker Where E is the system voltage at the instant of are interruption (also called active recovery voltage). As the transient oscillation is a fast phenomenon, E can be regarded as a constant for a short duration. Restriking voltage = Voltage across the Circuit breaker at the instant of are extinction = Voltage across the Capeitor at the instant of are extinction ‘Substituting these values in the equation given above, we get tee yy. =E dt Taking Laplace Transform on both sides of the equation, we get E LC8'Ve(3) +Ve(8) | Where ve(S) is the Laplace Transform of v. Other terms are zero as initially q= 0 at t= 0, of Ve(s)[LC3? +1) E V-(s)=——=,— = SA oeriLcaen tof ; therefore, ‘Taking the inverse Laplace, we get OE s(s? +02) +0, or V()= V.0)=0,£ {sino =o As Ve(t) = Oat t= 0, constant = 0. Veit) = B(1 cos eyt ) Restriking voltage ‘The maximum value of restriking voltage = 2 Epc =2 x peak value of the system voltage + The Rate of Rise of Restriking Voltage(RRRV) da “econ “te0-so,} =0,Esino,t ‘The maximum value of RRRV = «,E. = OE yeax Frequency of natural oscillation, rad/sec Hz * Ome ER) raid | Ne Dat | Benga | Bhanesnar | Vita | VisRtapaam | Tapa | Pane | Chena 7.7 Current chopping When interrupting a low inductive current (shunt reactor or magnetizing current of transformer), the current become abruptly zero well before natural zero instant. This phenomenon known as current chopping. Due to current chopping the voltage across circuit breaker under goes to huge oscillation and may results in restrike. During current chopping, Apel 2 < 2 Prospective voltage v * A current chopping phenomenon is very severe during the interruption of low magnetizing current. * A current chopping phenomenon is very severe in air blast circuit breaker. System voltage | With standing Vs capacity of CB Va | vi Fig. 7.6: Current Chopping © If prospective voltage > with standing capacity of circuit breaker then gap will restrike. © Arc restriking is successfully done up to the point where the prospective voltage become less than withstinding capacity of circuit breaker. 2120: Power Systems 78 Resistance Switehing The resistance switching is employed to reduce the effect of restriking voltage and current chopping. PT 1 ite Fig. 7. Circuit for analysis of resistance switching ‘The voltage equation is given by div Lo4—ficdt=B and i=ic tig at ol : : Therefore, the above equation become ilctil yap a dic | di racy tingy, aa NC Or ig =f = KC¥e) a dt ic _ d(CVe) _o4?Ve Therefore, Se LC cA a at & dig 8Ve/R)_ 1 We a at R Substituting these values in the main equation, We get Taking Laplace Transform, we get oP Ve(6)+E8Ve(8)4Ve@)== uc Vesta Ve (s)+Ve(s): 5 Other terms are zero, as Ve= 0 at t= ims nyt cVelo|et+ ae or Lc cto +e +l or Ve(s) Tiydrabed | New Dali | Benn | Bakanenar | Vinjwada | Viaapatam | Tira | Pune | Cheon 2131 Circuit Breakers For no transient oscillation, all the roots of the equation should be real. One root is zero, i.e. s = 0 which is real. For the other two roots to be real, the roots of the quadratic equation in the denominator should be real. For this, the following condition should be satisfied, 2 : ) -L] 20 or —L 2RC} LC 4R*C? ‘Therefore, if the value of the resistance connected across the contacts of the circuit breaker is equal to or less than [E » there will be no oscillations ‘The frequency of damped oscillation is given by ere 2«VLC aR? 7.9 Ratings of cireuit breakers 1, Breaking capacity of circuit breaker 2. Making capacity of circuit breaker 3. Short time current rating: This is based on thermal and mechanical considerations. The circuit breaker must be capable of carrying short circuit current for a short period without damaging its contacts, Rated voltage > L - L rms voltage, generally ‘more than the nominal system voltage. 5. Rated current or continuous current rating: It is ms value of current caried by CB under normal operating condition = 6. Rated frequency -> System frequency Lc System | Vstue of R | Envelope of Restriking || 7. No. of operation, : : damping Voltage No, of operation is specified on name plate of CB after which the CB requires maintenance. . ‘One operation = One break + one make ae 8. Operating duty | i) Cireuit breaker not used for auto reclosing. B-3~MB-3-MB Where, B ~ breaking operation, 3 — Duration in minute Over , damping ‘Mi — Making operation Gi) Circuit breaker with auto reclosures L_ B—D,~MB | Where, D, - Det time of circuit breaker in cycles (approximately 300 ms). Under damping Ex: Fault on power system G i © isa BE 7 | No ead Un. Rew \ R ‘ L 0" damping | _ _ Mo Irault Hyderabad | New Delhi | Bengalura | Biabanswar | Viayavada | Veakhapamam | Tirupal | Pune | heal 2122: Power Systems LA = sin( or a9) Z| Transient period ‘Steady state period (08) symmetrical period 7.9.1 Symmetrical breaking capacity It is the rms value of the ac component of the fault current that the circuit breaker is capable of breaking, oA v2 * Lys the fault current comesponds to 3-6 fault at breaker terminal + Symmetrical breaking current, Iyy = ly= 3 2 © Symmetrical breaking capacity V3 xrated system voltage symmetrical breaking current = 1,,(psu.)x Base MVA = [SOMA ur 7.9.2 Asymmetrical breaking capacity It is the rms value of the total current comprising of both ac and de components of the fault current that the circuit breaker can break. Asymmetrical current (For First cycle) [RGR ected Nev ba | Haste | keno | Viren | Vinton [Tepe [Pee Ge] foo = xxly x= 1.6 for first cycle Asymmetrical breaking capacity: _ VB xrated systemvoltage x Asymmetrical breaking current xxsymmetrical breaking capacity Note: Default is always symmetrical breaking capacity. 7.9.3 Making Capacity The possibility of a circuit breaker to be closed on short circuit is also considered. The rated making current is defined as the peak value of the current (including the de component) in the first cycle at which a circuit breaker can be closed onto a short circuit. © The capacity of a circuit breaker to be closed onto a short circuit depends upon its ability to withstand the effects of electromagnetic forces. ‘Making current = the peak value of fault current corresponds to first cycle = oc -J2 1.8% symmetrical breaking current 2.55 x symmetrical breaking current Making capacity = 2.55 x symmetrical breaking capacity 7.10. Classification of Circuit Breakers Classification is based on Arc quenching medium (or) insulating medium used between contacts i) Air break circuit breaker ii) Air blast circuit breaker Oil circuit breaker iv) SF gas circuit breaker v), Vacuum circuit breaker In circuit breakers, the solid insulating materials will not be used for are quenching purpose ® 7.10.1. Air break circuit breaker ‘+ Also called AB switch. © Atmospheric air available at room temperature and normal pressure will be used as Arc quenching medium * Dielectric strength of air = 30 kV/em (peak) © There is no external action taken for the Are interruption in the air break circuit breaker. So Are interruption process is poor * This circuit breaker used for operating voltages (400 V to 1900 V) 7.10.2. Air blast circuit breaker High pressure air will be used as arc quenching medium + High pressure air will be blasted into the Arcing chamber for Arc interruption. * High pressure air will have good cooling properties ‘© The high pressure ait will be maintained up-to condensation point where the gaseous material will be converted into liquid material. © Air pressure is maintained at 20 — 30 kg/em? © Operating voltage levels:of Air blast circuit breakers is12 kV to 525 kV * According to blasting direction of air there are three types of Air blast circuit breakers. i) Axial blast ii) Cross blast ii) radial blast (i Axial blast circuit breaker: ON High pressufe—, Air FC Mc are Noe © High pressure ait will be exested on to the are axially ERS od New Dai | Benes | Wher | Vioranod| Viana | Trpal| Pae| Chal ii) Cross blast circuit breaker: are FC Mc eee (01) forced cool air ‘+ High pressure air will be exerted on to the are perpendicularly. So arc resistance will increase and heat energy developed by the arc will reduce. So it will reduce the thermal ionization process. (Gil) Radial blast circuit breaker ee sir FCF*S5|MC Double blast CB aie High pressure Air © High pressure air will be exerted on to the arc perpendicularly and It will leave arcing chamber axially. © This method is: used for modem extra high voltage circuit breakers. 7.103 Oil Circuit breaker + Insulating oil or transformer oil is used for Arc quenching medium, + AE ihe contacts are separated there is an are struck between the contacts. ‘The heat of the are evaporates the oil and it is decomposed into hydrogen gas at high pressure. + The volume of gas produced is one thousand times that of oil decomposed. + An are energy decomposes oil into 70 % hydrogen, 5 % methane, 22 % acetylene and 3% ethylene + Arc is in a bubble of gas and surrounded by the oil. The are extinction is facilitates mainly by two process. 124: Power Systems (i) Hydrogen gas cools the arc. (ii) Hydrogen gas pushes the oif in between the contacts and then removes the arcing products from the Arc. Advantages: + Itabsorbs the are energy to decompose the oil into gases which is an excellent cooling property + It acts as an Insulator and permits smaller clearance between live conductors and earthed components. Disadvantages: + Itis inflammable and there is a risk of Fire. + Possible to form an explosive mixture with ar. + Arcing products (carbon particles) remains in the oil. These carbon particles increases with number of operation which will deteriorate the oil, so frequent maintenance of oil is required. According to the amount of oil used in oil circuit breaker there are two types of circuit breaker. 1. Bulk oil circuit breaker 2. Minimum oil circuit breaker Purpose | Bulk | Minimum oit rp oil CB CB ‘Are quenching | Oil Oil Insulation | Solid provision with ingulatin respective to | Oil r live parts in aN ‘the system. (osrceai) : - Gaseous Cooling the arc | Oil coolant * In bulk oil circuit breaker, the contacts are placed inside a rigid mechanical chamber * Whenever, the contacts are about to separate, a small amount of oil will be sent to the space of contacts, so that, the oil will be converted into 25 which will form a gas bubble around the Are to cool the Arc, or for heat dissipation, gaseous coolant will be sent on to gas bubble (SRR) ati | New Dah | Benin | Bhubaneswar | Vieanada | Vieapatam | Tirupati | Pane | Cheol 7.10.4, SF gas cireuit breaker : © SF6 gas is used as are quenching medium * Dielectric strength of SF gas is 2 —3 times of air dielectric strength, Dielectric strength of SF gas = 80 kV/em = 10 Fs gas is « good coolant and it is having electroregative property. * Electronegative property is a property of material which will attract electrons or electron density under the presence of SFs gas. The electrons from the arc will be removed so that the arc strength gets reduced * By using cooling property of SF¢ gas, the heat will be dissipated + Atcurrent zero point, a turbulence force, will be created, inside the circuit breaker chamber, such that the heat dissipation will become more effective, there by the arc will be interrupted at current zero point, © Operating voltage levels 6.6 KV to 800 kV + For all voltage levels, this SFs gas circuit breaker will be used 7.10.5. Vacuum circuit breaker © Ideally vacuum means zero pressure (or) ‘0” torr (or) ‘0° mm of Hg pressure Practically high vacuum is created at 10° torr Dielectric strength of vacuum = 107 Vem = 10000 kV/em «As a dielectric’strength of vacuum is very very high ‘The spacing require between the contacts is very Jow and it is about mm. ‘* There is no gaseous atoms or molecules presents in vacuum, * When ever the contacts are about to separate a small amount of metal vapour will be released from contact bodies, due to mechaaical friction between the contacts ‘© On the metal vapour there are two forces (i electromagnetic forces due to current flow through contacts Gravitational force (mg) Circuit Breakers | Atcontact separation time, the electromagnetic Voltage ‘Are forces will dominate the gravitational force, Range and Type Quenching such that the metal vapour will be stayed Breaking Medium between the contacts and it forms a conducting Capacity path between the contacts. This conducting path Miniature ‘Airat ——] 400-600V; for | is called as plasma. Circuit atmospheric | Small current |» At current zero point, the electromagnetic farces Breakers Pressure taking rating | 7 ‘Air Break ‘Air at become zero and the metal vapour will reach to i Bes | soveitk circuit . revi atmosphere the bottom of circuit breaker chamber due to pl eA gravitational forces. There is no a gaseous atom Breakers or metal vapour are available for arc restriking. || Minimum Oil S.34N - 220 . ‘Transformer So arc will be interrupted at first current zero Circuit = KV; 150- itself Breakers 25000 MVA | Operating voltage levels up to 66 KV. For Vaca a eT above 66 KV levels the creation of vacuum is Cu pee | eee difficult as well as itis costly. rceeees | sr, SFeats | 3376S kV; cireuit 7.11. Comparison Of circuit Breaker: eee kg/om? 1000-50,000 ke 1. Insulating Methods. Breakers pressure MVA iE zi ~ Compressed Property Breakers Oil | SF6 Vacuum: Air Blast 66 kV — 1100 Nese air at high Nees Medium | Low | Medium | High Cireuit KV; 2500- |_ operations | pressure (20- ‘soft’ break | Gooa | Good | Good |) Fair Breakers >, | 60,000 MVA ; ability i 30 kg/m?) Monitoring of Manual Not | Medium NA | “rest_| Automatic | possible _ Fire hazard Choice of ' xowe | High | None | None it ua Rated Voltage circuit Remark a ett | None | Low | tow | None ben a Economical | Uptol [33-22 | 33 33-36 ir Oreak © | Vacuum is voltage | kV | kv | gooey | ev Below 1kV | vacuum CB, | TAs : SFCB, |? Minimum oil 5 132KV-220KV | CBSF6CB) St | airblastC.B, | PS SF, C.B., ait SFe is 400 KV- 760KV | Blast CB preferred. (Ea rtd | Nev Dati | era | Btneswar | Viyawada | Vsaapawaca | Tira | Pane | Ghana 2196: Power Systems 2. Features of CB: __Factor Gil Breakers Air Breakers VaccumiSK6 Safety Risk of explosion and Fire due to | Emission of hot air and | No risk of explosion increase in pressure during multiple | ionized gas to the operations surroundings Size Quite large Medium Smaller Maintenance | Regular oil replacement Replacement of arcing | Minimum lubrication - contacts for control devices Environmental | Humidity and dust in the atmosphere Since sealed , no Factors can change the internal properties effect due to and affect the dielectric environment Endurance Below Average ‘Average Excellent Example 7.1: Solution: ‘An LL fault occurs at the terminals of a 34, (i) Symmetrical breaking capacity (SBC) = KV synchronous generator which is solidly 2000 MVA grounded. The fault current lags the system 7 voltage by 60° and voltage of the system is Vit = 33 KV decreased by 5% due to demagnetizing effect of SB.C= V3x¥,,x1, synchronous machine. Calculate active recovery voltage. = 2000510" Solution: ra ‘Vary = K; Kp Vm sing =35KA (ii) Making current = 2.55 x symmetrical 60° breaking current = 2.55 x 35 kA, = 89.22 kA (Due to demagnetizing voltage is decreased by 5%) K 5 (LL fault) Vary = K, KV sing 951.5 Hk, v3 = 1LO8KV. Example 7.2: Seo A3+p, 33 KV oll circuit breaker is rated for 120A 200 MVA and 4 see (The symmetrical breaking current is Gi) Us making current will be pp Hydcabad | New Dali] Benge | Dharwar | Vinvavodh | VinKipatam | Tinga | Pane | Chena Example 7.3: A344 fault on a 33 kV, SOHz system will give a steady state fault current of 20004. A 33 kV C.B 4s placed for this interruption. The phase ground ‘capacitance and inductance up to C.B location is 0.01 UF and 5:mH respectively. If arc is chopped at peak value of fault current then voltage across CB contacts is Solution: fault current = 2000 A ‘Maximum value of fault current = 2000 x J2 iE “Ne z = 2000 x Jef SA = 2 Mv dono Example 7.4: In 2.220 KV system, the reactance and capacitance up to the location of cireuit breaker is 8Q and 0.025 AF, respectively. A resistance of 600 ohms is connected across the contacts of the circuit beaker. Determine the following. Natural Frequency of oscillation, Damped Frequency of oscillation. ii) Critical value of resistance which will give no transient oscillation, (iv) ‘The value of resistance which will give damped frequency of oscillation, one fourth of the natural frequency of oscillation. ‘Sobution: = _ 8 _oonssan 2n50 100% () Natura Frequeney of oscilation = L [ aces IVC a = 6.304 kHz ~ 2 V0.02544% 0.02510 (ii) Frequency of damped oscillation is given by 1fi_ i ax lOmsian0oasAI0* aoonsai0*F lon} 1 [10 10" 2m V636 9 (iii) The value of critical resistance 1 2544 Le If 2VC 2\0.025xi0* (iv) The damped frequency of oscillation is 6.304 kHz = 1576 Hz 413KHz. = 504.352, ist6= 2a 2a [aaseaxaoDRI0" solomasci0*P aR or 1576 = 28 =10"25R 2nV6.36 Circuit Breakers 2 R=5208Q Example 7.5: For a 132 kV system, the reactance and capacitance up to the location of the circuit breaker is 3 ohms and 0.015 pF, respectively. Calculate the following. @ The frequency of transient oscillation, Gi) The maximum value of restriking voltage across the contacts of the circuit breaker. (ii) The maximum value of RRRV Solution: (The frequency of transient oscillation, L= et = 50, the system frequency .00954H © Ynef0.0095430.015%10 3 : uo tO = 13,201kH 2mxi.1962 7.524] (i The restriking Voltage = Ef cost] The maximum value of the restriking voltige = 2 Epeak 2132 V5 -215.56kV F (iii)The maximum value of RRRV = ©y Eyeat = 20, 132 RV Is a =9.0104547 / ps SERRE I si | New Det eg | lca | Vind | Venap | Twa [Fe | hss Protective Relays 8.1 Introduction: * An clectrical power system consists of generators, Transformers, transmission and distribution line. The system equipments are likely to damage during fault or abnormal condition in any part of the power system, if proper protection system is not provided. ‘© Ifa fault occurs in any part of the power system, an automatic protective device is needed to isolate the faulty element as quickly as possible. * A protective system includes, circuit breakers and protective relays to isolate the faulty section of the system from healthy sections. * The function of a protective relay is to detect an abnormal condition in the power system and issue trip signal tothe circuit breaker to disconnect the faulty element * The basic electrical quantities likely to change during abnormal condition are voltage, current, frequency and phase angle (Direction) * The protective relays utilize one or more of these quantities to detect abnormal conditions in a power system. Back-up Relay: a back up celay operates after a slight delay, if the main relay fails to operate. Back up protection: The back up protection is designed to clear the fault if the primary protection fails. It acts as a second line of defense. Primary protection: If a fault occurs, itis duyy of the primary protective scheme to clear the fault and it acts as a first line of defense. If it fails, the back — up protection clears the fault Stability: This is the ability of the protective system to remain inoperative under all load conditions, and also in case of external faults. The relay should remain stable when a heavy current due to an extemal fault is flowing through it. Pick up level: The value of actuating quantity above which relay operates, Reset level: ‘The value of actuating quantity below which the relay is de-energized and retums to its original position, Relay: Relay is a sensing devices, this will sense the abnormal condition such as short cireuit fault in the system and gives tip signal for circuit breaker Operating Time: The time which elapses between the instant when the actuating quantity exceeds the pickup value to the instant when the relay contacts get closed, Reset Time: The time which elapses between the instant when the actuating quantity becomes. less than the reset value to the instant when the relay contact returns to its normal position, Primary Relays: The relays which are connected directly in the circuit to be protected. Secondary Relays: The relays which are connected in the circuit to be protected through current and potential transformers. Auxiliary Relays: Relays which operate in response to the opening or closing of its operating circuit to assist another relay in the performance of its function, This relay may be instantaneous or may have a time delay. 8.2 Basic objectives of system protection The fundamental objective of system protection is to provide isolation of a problem area in the power system quickly, so that the damage to the rest of the system is minimized and as much as possible is left intact. Thus protection does not mean prevention, but rather, minimizing the duration of the trouble and limiting the damage, outage time, and related problems that may result otherwise. The basic objectives of system protection are (i) Speed of operation Gi) Sensitivity ii) Selectivity (iv) Reliability EE) | New Deli | Benglura | Bhubaneswar | Viayarad | Visakhapatnam | Tropa | Pane | Cbenah 129 Protective Relays (®) Simplicity (vi) Stabitity (vii) Economies Speed of operation ‘The protection system should ensure minimum fault duration, consequent minimum damage 10 equipment and system stability. If the fault clearing is delayed, the healthy equipments of the system will damage due to excessive thermal and mechanical stress. So the protection system should isotate faulty section from healthy system as quickly as possible. Sensitivity The protection system should able to detect minimum possible value of foult current and issue trip signal to circuit breaker. Selectivity (Discrimination) The protection system should ensure maximum, continuity of service’ and minimum. system interruption The relay should able to discriminate the fault in its ‘own zone of protection or outside the zone of protection. Reliability It is the assurance that the protection gystem will perform 100% correctly thas two aspects 1. Dependability 2, Security Dependability: It should operate when itis required Security: It should not operate when it is not required Simplicity The protection system should have minimum number of equipment and associated circuitry 10 achieve the protection objectives Stability Relay operation must be in stable mode even it is carrying the fault current The relay should able to carry the fault current with out damaging it self till the point of interruption. Economics Protection should provide maximum protection at minimal total cost. 83 Classification of relays ‘* Classification based on technology Electromechanical Relays: These relays were the earliest forms of relay used for protection of power system, They work on principle of ‘mechanical force causing an operation of relay. Static Relays: These relays don’t have moving parts. The design of static relays is based on the use of Analog electronics instead of coils and magnets to create relay characteristics Digital Relays: To “implement the relay functions, Analog circuits used in static relays axe teplaced with microprocessors and micro controlters in digital relays ‘Numerical Relays: These relays can be viewed as natural developments of digital relays as a result of advances in technology. These relays use DSP processor as a computational hardware, together with the associated software tools. ‘© Classification based on application 1. over current relays 2. Directional relays 3. Distance relays: 4, Differential relays 84. Operating principle of Electro mechanical Relays Basically there are two different operating principles of relays (@ electro magnetic attraction (ii) electro magnetic induction FER Ne Ba Tce | anwar | Vga | Vea | Tea Pe | Clea] Power Systems Fig. 8.1: Attracted armature Relay ‘A rrestraining force has to be produced exactly in opposite direction of operating force to restrict relay operation under normal condition. ‘+ Field energy stored in electro magnetic circuit is function of air gap length (x) w=? 2 p= asa T I= length of magnetic path Jo. of tus of coil a= area of cross section of coil © The inductance ‘L’ changes with respect to contact gap = WO) $10) ? ‘© Torque produced by electro magnetic circuit, a 1 : yp dL lasinot, Fo= —(1,, sina)’ sino Fo= 3 (fy sinax) Jak & sin? ot 2dx 1h (Loastot) 2a" 2 aL Fora constant ‘x’, “is constant ix Tope T° Tres = Ki? Resultant torque, T= Top ~ Tres =KP-K Where K is the torque produced by spring ‘© This type of relay will be used for both AC and DC relaying purposes. 84,2. Induction type relays: Applicable only for AC relaying purpose. There are ‘Wo types of induction relays 1. Induction dise type element (j) Shaded pote structure ii) Watt hour meter/energy meter structure 2. Induction cup type element Shaded pole induction disc type Shaded pole Induction Relay For the relay application, continuous rotation of dise is not required. To restrict the rotation of disk during normal operation, a restraining torque is ET csi [Rew Daa | Hei | haber | nana | Vicky | Toa | Pane | Chea ACE 84 Engineering Academy 18h: Protective Relays produced by using spring. To damp out oscillations} made by disc a permanent magnet (PM) is provided. Tox pypsina. T=Fsina T=KiP — K=constant Watt-hour meter (energy meter) disc type element fer magnet ef 220; Fig. 83: Watt-hour meter type Induction Relay Upper magnet coil Lower magnet coil V2 20" V2Z0° + + v.20 + _ v.20 "2,28; + 12-0; = he + ¥ 6 2-8) hp ZO Torque produced on Aluminum dise Tc dy.gasin(®; ~ 02) © If; — 82 =0 (or) 6; = 6, then torque produced by Aluminum disc is zero and disc will not rotate. © To get rotation Aluminum disc 0; # 2 ot x ratios of upper coil and lower coil maintained different Induction cup type relay: © Operated based on induction motor principle, ive, the torque produced is due to interaction of two rotating magnetic fields. © Rotating body is a hollow Aluminum cup * This is constructed either for 4 pole or 8 pole. Zi Rotating Al cup ‘Stationary iron core Contact to tip r Fig. 83: Watt-hour meter type Induction Relay © Size of cup is very small and it’s weight and inertia is very less * Torquehveight rat ‘very high, * Typical operating time = 0.01 sec * This is high speed relay * Sensitivity of relay is very high and the ratio between reset level to pickup level is around 0.9, * Cup type relays are used in high speed relaying applications Applications of induction type elements: 1. All the three elements will be used as inverse time over current relays. 2, Watt-hour meter and cup type elements will be used as directional relays 3. In the distance relay both operating forces and restraining forces will be produced by system quantities by reconnecting the coils. PG tas [Ne Dati | Wears | Bhbanevar | Vinvavada | Vayama | Tre | Pane | Chena ® vest Tian 2182: Power Systems 8.4.3. Thermal relay: Thermal relay used to protect the rotating machines ftom over heating. (Cause for the over heating is over load on the system) ‘Types of thermal relays 1, Thermo couple/Thermistor 2, RTD (Resistance temperature detector) 3. Bimetallic strip Universal Torque Equation TKDE 4 KV? + KV c0x0-1)+K yoy 4 4 Current Voltage Directional Spring unit unit unit torque 8.5. Over current Relays From universal torque equation T=K, 0 + Kz V? + Ky VI.cos(6 -1) +K Ky =#ve, K; = 0,K3=0, K=-ve T=KiP-K Under threshold T=05 Top = K\P-K=0 Ki P=K During Operation T>0; Top > Tres Ki P-K>0 KiP>K i Je K ee . If cmment flowing through the relay operating coil is greater than the pick up value of current Relay operates. SRN) certs [New Dot | Benes | Bicone | Viana | Vieapaam [Ti 1> 1 pictap > Relay operates 1 Relay Not operates T= Ipictup > Relay is on Threshold. Current Setting: It is defined as the ratio of pick up current of relay to the relay current rating 1 EE 100 1 Note: Current settling is also expressed in Amperes If current setting is in Amperes then, Pickup current = current setting (A) lug setting (A) = Set value of current (A) Current setting (C.S) % 8.5.1 Plug Setting Multiplier (P.S.M) primary fault currentof CT _ Ps : SM™ CF Ratiox Relay current seting (A) secondary felt current ~ Relay current senting A) Relay current setting (A) = CSC) 5 Relay current rating Note: If relay current secondary current of C. current settling. iting is not given, the is reference to calculate 8.5.2 Characteristics of over current Relay It is a curve between P.S.M versus time o operation of over current relay Top =KF; Tres = K (constant) T=KP-K © If fault current increases, P.S.M increases. net torque (T) increases, speed of the disc increases and time of operation decreases. * So There is a inverse relation between P.S.M and time of operation of relay Fig, 8.5: Characteristics of Over Current Relay 8.5.3 Time Multiplier Setting (T.M.S): * To control actual operating time of over current relay, Manufacturers provide a provision for changing the operating time. ‘© Operating time of relay = T.M.S x Operating time of relay with 100 % T.M.S © To adjust current setting, the number of turns in relay coil is to be adjusted, * By adjusting back stop position operating time of relay is adjusted i} cr : (ROC) : ——tisg 70% : r——* 80% : -— 85% ' tC sar, : + 95% ' soo ; lamas 105% i -—< 110% i -—* 113% i }+—¢ 120% i L_§_< 125% i eee 3133: Protective Relays 8.5.4 Types of Over Current Relays Depending upon the time of operation the relays are categorized as (i) Instantaneous over current relay, (ii) Definite time over current relay ii) Inverse definite minimum time (DMT) over current relay (iv) Very inverse relay, (v) Extremely inverse relay. Instantaneous over-current relay Here there is no intentional time delay is provided for the operation of Relay. The time of operation of such felays is approximately 0.1 sec. This characteristic can be achieved with the help of hinged armature relays. Definite time-current relay Here the operating time is constant through out the. operation. ‘The operating time of Relay is independent of PSM Operating time in sees 10 0 10 100 PSM Fig. 2.6: Characteristics of Over Current Relays Inverse Definite Minimum Time Relay In IDMT Relay the operating time is approximately inversely proportional to the fault current near pick up value and becomes substantially constant slightly above the pick up value of the relay. This is achieved by using a core of the electromagnet which gets saturated for currents slightly greater than the pick up current. RR erat | New De | Bensalara | Bhubancovar | Vinyavada | Vikiapsinar | Tovpat | Pane | Chea Power Systems Very inverse relay In Very Inverse Relay the saturation of the core occurs at a later stage. The characteristics relay is as shown in Fig, 5.6. The time current characteristic is inverse over a greater range and afier saturation tends to definite time. Extremely inverse relay In Extremely Inverse Relay the saturation of the core occurs at a later stage, the characteristic assumes the shape as shown in Fig. 5.6, The equation describing extremely inverse relay is approximately of the form I?t = K, where I is the ‘operating current and / the operating time. 86 Directional Relays From Universal torque equation T=Kj\P +K,V? + K;VIcos(0-1) +K For directionat relays Ki =0, Kp=0, Ky = +ve, K=-ve T=K3VIcos(@- 1) —K so... @ The restraining torque ensure that for operation of directional relay a minimum current is required in ‘he set direction Fig. 8.7: Zero degree connection of directional relay ‘V—rms value of voltage fed to the voltage coil of directional relay ‘I<: value of current fed to the current coil of directional relay 0 - angle between V & 1 ‘t~ maximum torque angle Fiyderabad | New Delfi | Bengal | Bhubanesvar | Visyawada | Viskhapanara | Teupai| Pune | Obeomai ‘+ By changing the current coil connection the set direction of directional relay can be changed. If @-0) = constant cos(0-1) = constant equ. (i) => K3VI~ |K| Under threshold T=05 Top =Tys K3VI-|K|=0 IKI WD cece % (i) Equ. (ii) gives constant product characteristics for directional relay. During operation T>0, Top> Tres KVi-K 70 K VI>— K, Ne fi fi t No operation Fig. 88: Constant product characteristics of direction al relay Protective Relays Conclusion # IFT Inin (in set direction) and for any value of voltage the relay will not operate(This is expected) # If V < Vaig and for any value of current relay not operates (This should be avoided) ‘If there is a fault very near to relay location, the voltage fed to the relay is less than the minimum voltage required for the relay operation and directional relay does not operates. Dead zone The maximum distance upto which the voltage fed to the relay is less than the minimum voltage required for relay operation is called dead zone of directional relay. * If there is a fault any where in the dead zone the relay will not operate. * To avoid the dead zone problem the voltage coil of directional relay should not be connected to the same phase where current coil i Phasor diagram Top = KsVixos(0-1) or Phasor diagram of directional relay * The MTA line is always in quadrature with voltage coil current or voltage coil flux. Fijderabad | New Deli | Bengaluns | Bhubaneswar * Angle between voltage coil voltage and voltage coil current is adjusted with the help of phase shifting network. So it is possible to adjust the maximum torque angle. © If6,=90°, maximum torque angle, ¢ = 0° © 1f6,=0°, maximum torque angle, t= 90° © 1f0,=45°, maximum torque angle t= 45” Polar characteristics If restraining torque assumed as zero then total torque T= Top T=K;Vicos(0-1) Under threshold T=0= K3VIcos(0-1) = 0 relay. Region of eent flowin opposit. direction ties of directional relay Fig. 8.10: Polar character © Zero torque line is in phase with voltage coil current or voltage cail flux. © Zero torque line and maximum torque lines are orthogonal to each other. 8.7: Distance Relays (@ Impedance Relay (i) Reactance Relay (ii) Mho’s Relay sad ‘Veakigpanam | Trupa | Pane | Chennai MCE. HEE? Engineering Academy 8.7.1 Impedance Relay From Universal torque equation T=K.P +K,V’ + K:VIcos(0-1) +K For impedance Relay Ki = #ve, Kp =~ve, Ks =0,K=0 T=KP-K,Vv? (iv) © Current unit is providing operating torque Voltage unit is providing restraining torque + This relay is also called voltage restrained over current relay. Under threshold T=0, Top = Tres KiP~KV?=0 K,P = K,V? (W) When relay is operating T>0 Top = Trex KiP> KV? Wages PK, v f& ¥ i T VK, Lees < Leet Fig. 5.11: Characteristics of impedance relay on RX diagram Conclusions * If the impedance seen by the relay is less than the preset value of impedance, the relay operates. * The operation of relay depends on both resistance and reactance seen by the relay 1136: Power Systems * The impedance relay don't have inherent directional properties. * Itis moderately effected due to Arc resistance © Occupies moderates space on RX diagram. © Moderately effected due to power surges * Moderately effected due to load encroachment 8.7.2 Reactance Relay From Universal torque equation T=K,P +K,V? + K;Vicos(0~t) +K For Reactance Relay Ki =+ve, Ky =0, Ks =-ve, K=0 T=KF -K3Vicos(8— 1) © Current unit is providing operating torque * Directional unit is providing restraining torque © This relay is also called directional restrained over current relay. For Reactance Relay maximum torque angle is 90° So Net torque, T = Kil? ~ K3VIsin0 .... (vi) From equation (vi), the reactance relay also called Reactive power restrained over current relay. Under threshold. K.P K3VIsin0 = 0 K,P = K3Visind zsino= Ai K, ‘When relay operating Top > Rees K.P ~ K,VIsind > 0 Zsino< At (RRR era | New Dati | Henle | Bhubaneswar | Vieyarada | Visakhapatnam | Tropa | Pane | Chennat Academy 3137: xk K, XeXy x NOP op | R op Xo NOP Fig. 8.12: Charactetisties of Reactance relay on RX diagram Fig. 8.13: Characteristics of Mho Relay Conclusions © Ifthe reactance seen by the relay is less than the preset value of reactance, the relay operates © The operation of’ relay depends only. on reactance seen by the relay * The reactance relay don’t “haye inherent directional properties © Mis not effected due to Are resistance © Occupies more space on RX diagram. © Highly effected due to power surges © Highly effected due to load evicroachment 8.7.3 Mho Relay: . From Universal torque equation T=K,F + KyV?+K3VIcos(0-t) + K For Mho Relay K,=0,K; =—ve, Ks=+ve, K=0 T=K3VIcos(0 - t)-K,V” Direction unit is providing operating torque © Voltage unit is providing restraining torque * This relay is also called voltage restrained directional relay. ‘When Relay is operating T>0 K;Veos(@~t) ~ K,V? > 0. ‘on RX diagram Conclusions * If the impedance seen by the relay is less than the preset value of impedance, the relay operates * The operation of telay depends on both resistance and reactance seen by the Felay © The Mho relay having inherent directional future, © Iis highly effected due to Arc resistance © Occupies least spdice on RX diagram. «Less effected due to power surges © Less effected dueto load encroachment 8.74 Application of distance Relays Short transmission line © The impedance measurement by the relay is effected by arc resistance because arc resistance is comparable with transmission line impedance ‘« Reactance relay is suitable for the protection of short transmission {ine because it’s operation is independent on are resistance © Power swings on the short transmission line is very less and less severe. Long transmission line © Power swings are more common in long transmission line because of load variations Hyderabad | New Dai | Bengalans | Bhubancovar | Visyavada | Vinapamam | Trap | Pane | Cbennal ACE Engineering Academy +138 Power Systems * The relay which is selected for long | Case (i): transmission line should be less effected due to cr APparaUS is SE orwinding power swings. S pe -Mho relay is preferred. + Impedance of long line is very high so effect of are resistance is very less on impedance measurement. Medium transmission line: * Are resistance is with line impedance * Power swing occurrence is appreciable comparable Impedance relay is suitable. 8.8 Current differential relay @ a a Inocthrm PROC i) i Roc. Undesirable during sormal operating conditions cai) ior i Conditions: 1. C.T secondaries should be connected in additive polarity. Such that there is some current circulation in the C.T secondaries 2. ROC should be located exactly at mid point on pilot wire or at equal potential point. 3. Both the CT should have magnetization characteristics. 4. Ratio error and phase angle error should be same for both CT’s identical # Apparatus fed from only one side Input and output currents of apparatus are same L=h=|k, Iroc= 0. Case (ii): ROC Fault occurs in between 2 CT's Toc = Ir (telay operates for internal faults) * As relay is operating only for internal fault, this protection is called unit protection. * This relay can also be called as circulating current differential relay. * Differential relay will have highest selectivity property compared to over current and distance relays. ‘Applications: 1. stator winding of alternator 2. ‘Transformer 3. Transmission line protection 4, Bus bar protection Causes for mal-operation of differential relay: 1. Mislocation of ROC from equipotential points So Iroc # 0, under normal conditions relay operates. Fiyderabad | New Delhi | Bengalora | Bhubaneswar | Vinyavada | Visakhapatram | Tiupat | Pane | Chena @ rach Finsen Protective Relays «By placing balancing resistors on the pilot wire the ROC current will be maintained at zero under normal conditions. 2. Mismatch CT characteristics (BH curves) 3. Difference in ratio and phase angle errors. To avoid mal operation: © The mal-operation of the relay is depends’ on system conditions. © To restrict the relay ‘operation, a restraining force has 10 be produced which should depends on system conditions: + As operating torque depends on i; and iz the restraining torque should depends on ir and iz: * Biased differential ‘relay (or) %slope differential relay: Th System ch i aoeon 22 ‘Restraining coil (n.) Troe it ~in ‘Torque produced by an electro magnetic coil o (MMF produced by coil)” Operating torque, Top & [no(is ~i2)]* Top = Kola ~ i) Restraining torque T, c [3 +e SAS Na [ag | ce | | Veen [Tee] Go] Resultant torque T = Top ~ Tres -Ts Ts is inbuilt spring torque produced by over current relay element ; T= Keni -6[ (28 } -K, ‘ASK, is very very ess a5 compared (4) 2 T= Kelno(ir~ia] - w[a(2 “4 2) * IE'T.p> Tees then relay operates, #: If Tep< Tres then relay will not operate © If Typ=Tren then relay is at verge of operation. At verge of operation, F,es= 0 ACE, Bagicerg Academy © Always slope provided by biased differential relay is the ratio between restraining coil turns to ‘operation coil turns () Ny * Slope is adjusted by changing restraining coil tums Example 8 A 10 A relay is connected to the supply circuit through 1000/10 A C.T and it is set at 50 % if the fault current of the system is $000 A, The P.S.M_=? Sol: CS (A) =0.5x 10=5 A, L PSM = ————i_=— CI RatioxC.S(A) 10S 10 Example 8.2: Calculate the plug setting and. plug. setting ‘multiplier for the relay as shown below. ptt kV ee eae taal Hi Relay |< ‘aut CS(%) 100 5 2125 100 x1, ty Sol: Plug Setting 11x10° Fault current I;= Vpn = 1100 eee eee CTratioxPS(A) 1100 aos PSM =88 ESAT erated [New Dati | Benelona | Bhubaneswar | Visovada | ViaBbapaisam | Trwpa | Pune | Chennai 2140: Power Systems Example 8.3: Determine the time of operation of a relay of rating 5 amps, 2.2sec IDMT and having a relay setting of 128% TMS = 0.6. It is connected to a supply eireuit through a C.2. 40/5 ratio, The fault cucrent fs 4000 amps, TMS=1.0 20) 10 Time in 2.0) H scents 10 a 10 2030 1020 Sol: The pick up value of the relay is 5 amps but since the relay setting is 125%, therefore, the operating current of the relay is 5x1.25 = 6.25 amps ‘The plug setting multiplier of the relay, secondary current ~ Relay Currentsetting From the characteristics the operating time for PSM = Bis 3 sec. Since the TMS is 0,6, the actual operating time of the relay is 1.8 sec ACE Ni? Engineering Academy 141 Example 8.4: ‘A 6600 volt 3-phase turbo-alternator has a maximum continuous rating of 2,000 kW at 038 pf. and its reactance is 12.5%. It is equipped with Merz Price circulating current protection which is set to operate at fault currents not less than 200 amperes. Find what value of the neutral earthing resistance eaves 10% of the windings unprotected? Sol. Let the earth resistance be ‘r’ ohms Full load current of the alternator 2.000%I,000 _ 916 ar 0.8x V3 x6,600 Ps Let the reactance per phase of the alternator be x ohms. 12,5 — BXX*219. 99 9,600 x= 2.19 ohms Reactance of 10% of the winding 0.219 ohm ‘Voltage induced in winding is 6,600 3x10 =381.volts, The protection operates at a current = 200 amps. Hence r= 1.89 ohms. : Protective Relays Example 8.5: The section of balanced power system is shown below and a 34 fault occur at point ‘p’. The current following through relay is 40/5 Sol: System is balance so [I, | |1,|=Heland phase angle between the currents is 120° Secondary curfent of three C.T’s are same (balanced currents) So Trewy = Ty +1Y FILE 0 Example 8.6: ‘The connection of relay operating coil from two ‘C21"s is shown below the current flowing through relay operating coil is Booa ly 4005 1004, 005 SSRN END po veratad| New Dei | Benga | banca | Vinyard | Viakapatram | Tirwpal| Pune | Chena @ wit Fonien Sol: 1, A005 2008 109A , 40005 fall} <3.754 Example 8.7: A distance relay connection is shown below and there is a fault at point ‘p’. The impedance seen by the relay is, 4005” son P 10,000/1000 [SSR ea New Det | Benet] Bhar | Vinyard | Vip | Trop | Pane | Chena

You might also like